You are on page 1of 202

Mathematical Literacy Notes

Margaret Lubczonok and Sizwe Mabizela


Department of Mathematics (Pure & Applied)
Rhodes University
Contents

1 Mathematics in everyday life 1

1.1 The metric system (SI) . . . . . . . . . . . . . . . . . . . . . . . . . . . . . 1

1.2 Ratio, Proportion, and Percentage . . . . . . . . . . . . . . . . . . . . . . . . 7

2 Real Numbers 12

2.1 Classification of real numbers . . . . . . . . . . . . . . . . . . . . . . . . . . 12

2.2 Graphical representation of real numbers . . . . . . . . . . . . . . . . . . . . 15

2.3 Operations on real numbers . . . . . . . . . . . . . . . . . . . . . . . . . . . 18

3 Algebraic Expressions 22

3.1 Factorisation . . . . . . . . . . . . . . . . . . . . . . . . . . . . . . . . . . . 24

3.2 Algebraic Fractions . . . . . . . . . . . . . . . . . . . . . . . . . . . . . . . . 26

3.2.1 Simplifying algebraic fractions by cancelling out common factors . . . . 27

3.2.2 Multiplication and Division of algebraic fractions . . . . . . . . . . . . 29

3.2.3 Addition and Subtraction of algebraic fractions . . . . . . . . . . . . . 32

3.3 Partial Fraction Decomposition . . . . . . . . . . . . . . . . . . . . . . . . . 38

4 Linear and Quadratic Equations 43

4.1 Linear Equations . . . . . . . . . . . . . . . . . . . . . . . . . . . . . . . . . 43

4.1.1 Simultaneous linear equations . . . . . . . . . . . . . . . . . . . . . . 48

4.1.2 Applications: Problems leading to systems of linear equations . . . . . 53

4.2 Quadratic Equations . . . . . . . . . . . . . . . . . . . . . . . . . . . . . . . 55

4.2.1 Solving Quadratic Equations . . . . . . . . . . . . . . . . . . . . . . . 55

i
4.3 Equations leading to Linear or Quadratic Equations : Rational and Irrational
Equations . . . . . . . . . . . . . . . . . . . . . . . . . . . . . . . . . . . . 67

4.3.1 Irrational Equations . . . . . . . . . . . . . . . . . . . . . . . . . . . 70

4.4 Applications: Problems leading to linear/quadratic equations . . . . . . . . . . 74

5 Linear and Quadratic Functions 79

5.1 Concept of Function . . . . . . . . . . . . . . . . . . . . . . . . . . . . . . . 79

5.2 Properties of Functions . . . . . . . . . . . . . . . . . . . . . . . . . . . . . 84

5.2.1 Increasing and Decreasing Function . . . . . . . . . . . . . . . . . . . 84

5.2.2 Even and Odd Functions . . . . . . . . . . . . . . . . . . . . . . . . . 85

5.2.3 Zeroes of functions . . . . . . . . . . . . . . . . . . . . . . . . . . . 86

5.3 Linear Function . . . . . . . . . . . . . . . . . . . . . . . . . . . . . . . . . 88

5.3.1 Sketching a graph of a linear function . . . . . . . . . . . . . . . . . . 89

5.4 Quadratic Functions . . . . . . . . . . . . . . . . . . . . . . . . . . . . . . . 98

5.5 Quadratic Inequalities . . . . . . . . . . . . . . . . . . . . . . . . . . . . . . 104

6 Polynomials 113

6.1 Addition and Subtraction of Polynomials . . . . . . . . . . . . . . . . . . . . 114

6.2 Multiplication of Polynomials . . . . . . . . . . . . . . . . . . . . . . . . . . 115

6.3 Polynomial Division . . . . . . . . . . . . . . . . . . . . . . . . . . . . . . . 116

6.4 The Remainder Theorem and Factor Theorem . . . . . . . . . . . . . . . . . 120

6.5 Inequalities . . . . . . . . . . . . . . . . . . . . . . . . . . . . . . . . . . . . 126

7 The Absolute Value (Modulus) Function 128

7.1 Absolute-value Equations and Inequalities . . . . . . . . . . . . . . . . . . . . 129

7.2 The functions y = a|x p| + q . . . . . . . . . . . . . . . . . . . . . . . . . 133

7.3 Algebraic and graphical solution of equations and inequalities . . . . . . . . . 137

7.4 Geometric representation of Absolute Value . . . . . . . . . . . . . . . . . . 139

7.5 Miscellaneous Examples . . . . . . . . . . . . . . . . . . . . . . . . . . . . . 140

8 Trigonometry 146

ii
8.1 The radian measure . . . . . . . . . . . . . . . . . . . . . . . . . . . . . . 146

8.2 Trigonometric ratios in a right-angled triangle . . . . . . . . . . . . . . . . . 148

8.3 Trigonometric function of any angle . . . . . . . . . . . . . . . . . . . . . . 153

8.4 Trigonometric Identities . . . . . . . . . . . . . . . . . . . . . . . . . . . . . 156

8.5 Periodic Relations . . . . . . . . . . . . . . . . . . . . . . . . . . . . . . . . 158

8.6 Parity relations . . . . . . . . . . . . . . . . . . . . . . . . . . . . . . . . . . 159

8.7 Reduction Formulae . . . . . . . . . . . . . . . . . . . . . . . . . . . . . . . 160

8.8 More Trigonometric Identities . . . . . . . . . . . . . . . . . . . . . . . . . . 163

8.9 Graphs of Trigonometric Functions . . . . . . . . . . . . . . . . . . . . . . . 166

8.9.1 Some important transformations which are applied to the trigonometric


functions . . . . . . . . . . . . . . . . . . . . . . . . . . . . . . . . . 168

8.10 Trigonometric Equations . . . . . . . . . . . . . . . . . . . . . . . . . . . . . 171

8.10.1 Solving Equations: . . . . . . . . . . . . . . . . . . . . . . . . . . . . 173

9 Exponential and Logarithmic Functions 179

9.1 Exponential Laws and Definitions . . . . . . . . . . . . . . . . . . . . . . . . 179

9.1.1 Exponential Laws . . . . . . . . . . . . . . . . . . . . . . . . . . . . 179

9.1.2 Definitions . . . . . . . . . . . . . . . . . . . . . . . . . . . . . . . . 179

9.1.3 Surds Laws . . . . . . . . . . . . . . . . . . . . . . . . . . . . . . . 180

9.2 Operations on Exponents and Surds . . . . . . . . . . . . . . . . . . . . . . . 180

9.3 Exponential Equations . . . . . . . . . . . . . . . . . . . . . . . . . . . . . . 182

9.4 Equations with Rational Exponents . . . . . . . . . . . . . . . . . . . . . . . 182

9.5 Number Systems . . . . . . . . . . . . . . . . . . . . . . . . . . . . . . . . . 183

9.6 Scientific Notation . . . . . . . . . . . . . . . . . . . . . . . . . . . . . . . . 184

9.7 Exponential Functions . . . . . . . . . . . . . . . . . . . . . . . . . . . . . . 186

9.8 Logarithms (Definitions) . . . . . . . . . . . . . . . . . . . . . . . . . . . . . 189

9.9 Logarithmic Laws . . . . . . . . . . . . . . . . . . . . . . . . . . . . . . . . 190

9.10 Change of base . . . . . . . . . . . . . . . . . . . . . . . . . . . . . . . . . . 192

9.11 Logarithmic Equations . . . . . . . . . . . . . . . . . . . . . . . . . . . . . . 193

iii
9.12 Logarithmic Functions . . . . . . . . . . . . . . . . . . . . . . . . . . . . . . 196

9.12.1 Definition of Logarithmic Function . . . . . . . . . . . . . . . . . . . 196

iv
Chapter 1

Mathematics in everyday life

1.1 The metric system (SI)

Designed during the French Revolution of the 1790s, the metric system brought order out of
the conflicting and confusing traditional systems of weights and measures then being used in
Europe. Prior to the introduction of the metric system, it was common for units of length,
land area, and weight to vary, not just from one country to another but from one region
to another within the same country. As the modern nations were gradually assembled from
smaller kingdoms and principalities, confusion simply multiplied. Merchants, scientists, and
educated people throughout Europe realized that a uniform system was needed, but it was
only in the climate of a complete political upheaval that such a radical change could actually
be considered.

The metric system replaces all the traditional units, except the units of time and of angle
measure, with units satisfying three conditions:

1. One fundamental unit is defined for each quantity. These units are now defined precisely
in the International System of Units.

2. Multiples and fractions of these fundamental units are created by adding prefixes to the
names of the defined units. These prefixes denote powers of ten, so that metric units
are always divided into tens, hundreds, thousands, etc. The original prefixes included
milli- for 1/1 000, centi- for 1/100, deci- for 1/10, deka- for 10, hecto- for 100, and
kilo- for 1,000.

3. The fundamental units are defined rationally and are related to each other in a rational
fashion.

The metric units were defined in an elegant way unlike any traditional units of measure. The
Earth itself was selected as the measuring stick. The meter was defined to be one ten-millionth
of the distance from the Equator to the North Pole. The litre was to be the volume of one

1
cubic decimeter, and the kilogram was to be the weight of a litre of pure water. It didnt turn
out quite like this, because the scientific methods of the time were not quite up to the task of
measuring these quantities precisely, but the actual metric units come very close to the design.

The metric system was first proposed in 1791. It was adopted by the French revolutionary
assembly in 1795, and the first metric standards (a standard meter bar and kilogram bar) were
adopted in 1799. There was considerable resistence to the system at first, and its use was
not made compulsory in France until 1837. The first countries to actually require use of the
metric system were Belgium, the Netherlands, and Luxembourg, in 1820.

Around 1850 a strong movement began among scientists, engineers, and businessmen in favor
of a international system of weights and measures. The scientific and technical revolution was
well underway and a global economy was developing. The need for uniformity in measurement
was becoming obvious. Furthermore, the metric system was the only real choice available.
The only possible competitor, the British Imperial system, was so closely tied to the British
Empire it was not even acceptable to the Americans, let alone to non-English speakers.

Between 1850 and 1900 the metric system made rapid progress. It was adopted throughout
continental Europe, in Latin America, and in many countries elsewhere. It became firmly
established as a key part of the language of science.

In the 1870s the French made a crucial decision to turn control of the system over to an
international body. In 1875, most of the leading industrialized countries (including the United
States, but not Britain) signed the Treaty of the Meter. The treaty established the International
Bureau of Weights and Measures, which has presided ever since over what we now call the
International System of Units. It also provided for distribution of copies of the metric standards
throughout the world and for continuing consultation and periodic revision and improvement
of the system through regular meetings of a General Conference of Weights and Measures.
The 22nd General Conference met in October 2003.

Since 1875 the eventual triumph of the metric system in science and international commerce
has been assured, despite continuing popular opposition in Britain and the United States. In
fact, the metric system has met popular opposition in every country at the time of its adoption.
People dont want to change their customary units, which are part of how they see and control
the world. It is naturally disturbing to do so. This opposition has been largely overcome
everywhere, except in the U.S., by economic necessity: the need to participate fully in the
global economic system. Even in the U.S., economic needs assure the continued creeping
adoption of the system in one area and then another.

Those Americans opposing adoption of metric units often argue that the metric system is
abstract and intellectual or that its use would embroil us in calculations. This is not true. The
metric system has been the customary measurement system in France for almost two centuries,
in the rest of continental Europe for at least one century, and in the rest of the world for a
least a generation or two. Most people in the world know exactly how long a kilometer is, how
large a litre is, how much a kilogram weighs, and how warm 25 C is, because they use these

2
units every day of their lives in the same way Americans use miles, gallons, and pounds.

Outside Britain and the United States there is almost no need to convert metric units into
something else. In fact, the way to avoid conversion formulas is to adopt the metric system.
As long as Britons and Americans continue to use traditional units, they will have to remember
how these units relate to the metric units.

Widespread use of the metric system has not meant the complete elimination of traditional
units, nor has it stopped the continuing creation of new units, many metric but some not, to
meet new needs.

All systems of weights and measures, metric and non-metric, are linked through a network
of international agreements supporting the International System of Units. The International
System is called the SI, using the first two initials of its French name Systeme International
dUnites. The key agreement is the Treaty of the Meter (Convention du Metre), signed in Paris
on May 20, 1875. 48 nations have now signed this treaty, including all the major industrialized
countries. The United States is a charter member of this metric club, having signed the original
document back in 1875.

The SI is maintained by a small agency in Paris, the International Bureau of Weights and
Measures (BIPM, for Bureau International des Poids et Mesures), and it is updated every
few years by an international conference, the General Conference on Weights and Measures
(CGPM, for Conference Generale des Poids et Mesures), attended by representatives of all
the industrial countries and international scientific and engineering organizations. The 22nd
CGPM met in October 2003; the next meeting will be in 2007. As BIPM states on its web site,
The SI is not static but evolves to match the worlds increasingly demanding requirements
for measurement.

At the heart of the SI is a short list of base units defined in an absolute way without referring
to any other units. The base units are consistent with the part of the metric system called the
MKS system. In all there are seven SI base units:

the meter for distance,

the kilogram for mass

the second for time,

the ampere for electric current,

the kelvin for temperature,

the mole for amount of substance, and

the candela for intensity of light.

Other SI units, called SI derived units, are defined algebraically in terms of these fundamental
units. For example, the SI unit of force, the newton, is defined to be the force that accelerates

3
a mass of one kilogram at the rate of one meter per second per second. This means the newton
is equal to one kilogram meter per second squared, so the algebraic relationship is N = kg
ms2 . Currently there are 22 SI derived units. They include:

the radian and steradian for plane and solid angles, respectively;

the newton for force and the pascal for pressure;

the joule for energy and the watt for power;

the degree Celsius for everyday measurement of temperature;

units for measurement of electricity: the coulomb (charge), volt (potential), farad
(capacitance), ohm (resistance), and siemens (conductance);

units for measurement of magnetism: the weber (flux), tesla (flux density), and henry
(inductance);

the lumen for flux of light and the lux for illuminance;

the hertz for frequency of regular events and the becquerel for rates of radioactivity
and other random events;

the gray and sievert for radiation dose; and

the katal, a unit of catalytic activity used in biochemistry.

The SI does not allow use of any units other than those listed above and their multiples. In
particular, it does not allow use of any of the English traditional units (the horsepower, for
example), nor does it allow the use of any of the algebraically-derived units of the former CGS
system, such as the erg, gauss, poise, stokes, or gal. In addition, the SI does not allow use of
other traditional scientific and engineering units, such as the torr, curie, calorie, or rem.

Certain scientific fields have defined units more or less compatible with the SI, but not part
of the SI. The use of the jansky in astronomy is a good example. There is always the chance
that future meetings of the CGPM could add these units to the SI, but for the present they
are not approved.

For multiples of approved units, the SI includes a list of prefixes. This list has been extended
several times, most recently by the 19th CGPM in 1991. Prefixes now range from yotta- at
1024 (one septillion) to yocto- at 1024 (one septillionth). There seems to be some need for
another extension, but this question was not addressed at the 1999 CGPM. The SI does not
allow these prefixes to be used for binary multiples, such as the use of kilobit to mean 1024
bits instead of 1000. For binary multiples a new list of special prefixes has been established
by the International Electrotechnical Commission.
Metric Prefixes

Here are the metric prefixes, with their numerical equivalents:

4
Prefix Symbol Multiplier Numerical Exponential

yotta Y 1 000 000 000 000 000 000 000 000 1024

zetta Z 1 000 000 000 000 000 000 000 1021

exa E 1 000 000 000 000 000 000 1018

peta P 1 000 000 000 000 000 1015

tera T 1 000 000 000 000 1012

giga G 1 000 000 000 109

mega M 1 000 000 106

kilo k 1 000 103

hecto h 100 102

deca da 10 101

no prefix means: 1 100

deci d 0.1 101

centi c 0.01 102

milli m 0.001 103

micro 0.000 001 106

nano n 0.000 000 001 109

pico p 0.000 000 000 001 1012

femto f 0.000 000 000 000 001 1015

atto a 0.000 000 000 000 000 001 1018

zepto z 0.000 000 000 000 000 000 001 1021

yocto y 0.000 000 000 000 000 000 000 001 1024

5
Commonly used metric system units and symbols

Quantity measured Unit Symbol Relationship


millimeter mm 10 mm = 1 cm
Length, width,
centimeter cm 100 cm = 1 m
distance, thickness,
meter m
girth, etc.
kilometer km 1 km = 1000 m

milligram mg
1000 mg = 1 g
gram g
Mass (weight)* 1 kg = 1000 g
kilogram kg
1 t = 1000 kg
metric ton t

Time second s

Temperature degree Celsius


C

square meter m
Area hectare ha
1 ha = 10 000 m2
square kilometer km2
1km2 = 100 ha
milliliter m`
1000 m` = 1 `
cubic centimeter cm3
Volume 1cm3 = 1 m`
litre `
1000 ` = 1 m3
cubic meter m3
meter per second m/s
Speed, velocity
kilometer per hour km/h 1 km/h = 0.278 m/s
Density kilogram per cubic meter kg/m3
Force newton N
Pressure, stress kilopascal kPa
watt W
Power
kilowatt kW 1 kW = 1000 W
kilojoule kJ
Energy megajoule MJ 1 MJ = 1000 kJ
kilowatt hour kWh 1 kWh = 3.6 MJ
Electric current ampere A

6
1.2 Ratio, Proportion, and Percentage

Definition 1.1. (Theorem 3.2.1[2])A ratio is a comparison of two quantities.

3
Ratios are written with a colon between the quantities (e.g., 3 : 4) or as a quotient (e.g., ).
4
The ratio a : b is read as a is to b.

NB: The order in which the numbers appear in a ratio is important: a : b is NOT the same
as b : a.

Examples 1.2. 1. In a certain school, the ratio of boys to girls is 3 : 2. This means that
there are three boys to every two girls in this school. Put differently, three out of every
five (= 3 + 2) pupils in this school are boys and two out of every five are girls.

2. A box contains oranges, apples and bananas in a ratio of 3 : 2 : 5. This means that out
of every 10 = 3 + 2 + 5 fruit, we have 3 apples, 2 oranges and 5 bananas.

Dividing a quantity in a given ratio

Examples 1.3. 1. Divide 1360 in a ratio 2 : 3.


2
Soln: A ratio of 2 : 3 means that every 5 units are split as 2 and 3. The first number is of
5
3
1360 and the second is of 1360. That is,
5
2 3
1360 = 544 and 1360 = 816.
5 5
The number 1360 is divided into 544 and 816.

2. John, Jim and Dave have agreed to divide their bonus of R2 300 in a ratio of 2 : 3 : 5,
with John getting 2 parts, Jim 3 parts, and Dave 3 parts of the bonus. How much will
each get?

Soln: A ratio of 2 : 3 : 5 means that every 10 = 2 + 3 + 5 units are split as 2, 3 and 5.


Therefore
2
John will get 2300 = 460 rand;
10
3
Jim will get 2300 = 690 rand, and
10
5
Jim will get 2300 = 1150 rand.
10
3. In a park, the ratio of ducks to geese is 12 to 8. How many of the 250 birds are ducks
and how many are geese?

7
Soln: The ratio of 12 : 8 means that, of every 12 + 8 = 20 birds, 12 are ducks and 8
12 8
are geese. That is, of the birds are ducks and are geese. Therefore there are
20 20
12 8
250 = 150 ducks and 250 = 100 geese.
20 20
Definition 1.4. A proportion is an equation stating that two ratios are equivalent.

Proportion may be written with a double colon (::) or with the equality sign (=). The
proportion a : b :: c : d is read as a is to b as c is to d. The proportion a : b :: c : d is also
a c
expressed as = .
b d
For example, 1 is to 4 as 3 is to 12, is written written as
1 3
1 : 4 :: 3 : 12 or = .
4 12
In a proportion a : b :: c : d the first and last terms (a and d) are called extremes and the
second and third terms (b and c) are called the means.

In a proportion, the product of the extremes is equal to the product of the means; i.e.,
a : b :: c : d if and only if ad = bc.

Examples 1.5. 1. The ratio of 5th graders to 6th graders at a school is 2 to 3. If there
are 440 5th graders, how many 6th graders are there?

2. If 3 out of every 5 people will vote in an election, how many people will vote in a
population of 40 000?

Soln: Let n be the number of people out of the population of 40 000 that will vote. Then,
the given information says that

3 : 5 :: n : 40 000.

Equivalently,
3 n
= .
5 40 000
3
Therefore n = 40 000 = 24 000.. That is, 24 000 people out of the population of
5
40 000.

Ratio and proportion are used to solve many real-life problems:

1. Maps, blueprints, and scale models- An appropriate ratio, or scale, is chosen. Large
objects or areas can then be shown in a considerably smaller size or space.

2. Medicine dosages are frequently related to the weight of the patient, so the dosage must
be increased/decreased proportionally to weight.

3. Recipes or formulas may be increased/decreased by using proportions.

8
Definition 1.6. Two quantities x and y are said to be

1. directly proportional if one of these is a constant multiple of the other; i.e.


y = cx, where c is a constant. If x and y are directly proportional to each other, we
write x y.

2. inversely or indirectly proportional if their product is a constant; i.e., xy = k,


where k is a constant. If x and y are inversely proportional to each other, we write
1
x .
y
Definition 1.7. A percent is a ratio that compares the number to 100.

The word percent means per one hundred. We use the symbol % to denote a percentage.
15
For example, 15% means or 0.15.
100
17
Examples 1.8. 1. Express as a percent.
40
Soln: We are asking the question: what number compares to 100 as 17 compares to 40? Let
n be that number. We set up the following proportion
n 17
n : 100 :: 17 : 40 equivalently = .
100 40
17
It follows that n = 100 = 42.5.
40
2. Figures from Makana Municipality show that 22 570 people visited Grahamstown during
the 2006 Arts Festival. This was an increase of 12.85% on 2005 figures. How many
people attended the Arts Festival in 2005?

Soln: Let n be the number of people who attended the Arts Festival in 2005. From the given
information, the number of people who attended the Arts Festival in 2006 is

n + 0.1285n = 22 570, or equivalently 1.1285n = 22 570.

Therefore, solving for n, we have that


22 570
n= = 20 000.
1.1285
That is, 20 000 had attended the Arts Festival in 2005.

3. A recent poll has found that 36% of those asked planned to vote for one particular
candidate for President of the SRC. If 4 350 students vote, how many students vote for
the particular candidate?

Soln: We need to find 36% of 4 350. We first convert 36% to the number 0.36. Since we
want 36% of 4 350, we multiply 0.36 and 4 350 to get

0.36 4 350 = 1 566.

That is, 1 566 will vote for the particular candidate for President of the SRC.

9
Definition 1.9. A scale drawing is a drawing that has the same shape but different
size to the actual object.

The scale of the drawing is a ratio of the size of the drawing to the actual size of object. That
is,
Objects model size
scale = .
Objects actual size

To solve a scale problem, set up a proportion

Objects model size Scale information of the model


=
Objects actual size Scale information of the actual object
Examples 1.10. 1. A stadium is 90m long and 60m wide. If 1cm represents 10m, what
are the dimensions of the stadium if it is drawn on a sheet of paper?

Soln Let x be the length of the stadium on paper and y its width on paper. Let us establish
proportions:
x 1 1
= or equivalently x = 90,
90 10 10
and so x = 9cm. That is, the length of the stadium on paper is 9cm.

y 1 1
= or equivalently y = 60,
60 10 10
and so y = 6cm. That is, the width of the stadium on paper is 6cm.

2. Cartographers use scale extensively to draw maps on paper. Scales on maps relate the
distance on the map to the actual distance on the ground. A scale 1:10 000 shows that
one measurement unit on the map represents 10 000 of the same units on the ground.
For example, if units used are centimetres, then a scale of 1:10 000 means that 1cm on
the map represents 10 000 cm on the ground.

What would 7cm represent if the scale is 1:1 000?

Soln: From the given scale, 1cm represents 1 000 cm. Therefore, 7cm represents 7 000 cm
(or 7km).

Exercises

1. A car travels a distance of 144 km on 12 litres of petrol. How far will it travel on 50
litres of petrol?

2. If four loaves of bread feed three people, how many loaves will you need to feed tewlve
people?

10
3. In a rectangle, the ratio of length to width is 5 : 2. The perimeter of the rectangle is 28
cm. Find the dimensions of the rectangle.

4. An amount of R1 750 is to be divided between two people in the ratio of 3 : 4. How


much does each person receive?

5. The ratio of alcohol to water in a mixture is 1 : 9. Find the percentage of alcohol in the
mixture.

6. A ratio of males to females in a school is 5 : 4. If there are 1 800 pupils in the school,
find the number of each sex.

7. If 4 out of every 7 people are going to vote in an election, how many people will vote in
a population of 210 000?

8. A mixture of alcohol and water is in the proportion alcohol : water = 1 : 10. How much
water must be added in the mixture in order for the mixture to have 5% alcohol?

9. A sale price of an item is increased by 10% in December to a new price. In January,


this new price of the item is reduced by 10%. Is the January sale price more, less, or
the same as the November sale price? Justify your answer.

10. Is a price increase of 10% followed by a further increase of 10% equivalent to a one-off
20% price increase on the original price? Justify your answer.

11. The ratio of female to male shoppers at a department store has been found to be 9 to
5. If 5 706 female shoppers were at the store one Saturday, how many shoppers were
there in all that day?

12. On a map, 1cm represents 15km. If two cities are 7cm apart on the map, what is the
actual distance between the two cities?

13. To make green paint, one has to mix 2 parts of blue paint and 3 parts of yellow paint. If
a painter has 60 ` of blue paint and 100 ` of yellow paint, what is the maximum amount
of green paint can the painter make?

14. The results of a survey show that 23% of TV viewers in South Africa watch the sit-
com Generations. If there are approximately 7 000 000 viewers, how many people
Generations?

15. Suppose that we have 10 litres of a 20% solution and we want to add a 5% solution to
it to make a 10% solution. How much of the 5% solution do we need to add per litre
of the 20% solution?

16. Find the number of litres of 5% solution that must be added to 1 litre of a 40% solution
to obtain a 30% solution.

17. Find the number of litres of water that must be added to 1 litre of a 60% solution to
obtain a 10% solution.

11
Chapter 2

Real Numbers

2.1 Classification of real numbers

We start this section by reviewing various types of numbers that make up the real number
system.

The set of natural numbers is the set {1, 2, 3, . . .}. We use the symbol N to denote
the set of natural numbers. That is,

N = {1, 2, 3, . . .}.

The set of whole numbers is the set {0, 1, 2, 3, . . .}. We use the symbol N0 to denote
the set of whole numbers. That is,

N0 = {0, 1, 2, 3, . . .}.

The set of integers is the set {. . . , 1, 2, 1, 0, 1, 2, 3, . . .}. We use the symbol Z


to denote the set of integers numbers. That is,

Z = {. . . , 1, 2, 1, 0, 1, 2, 3, . . .}.

A rational number is any number that can be written as a quotient of two integers.
The set of rational numbers is denoted by Q. That is,
a
Q = { , where a and b are integers, with b 6= 0}.
b

We can also define a rational number as a number whose decimal representation either
terminates or repeats.

An irrational number is any number that cannot be written as a quotient of two



integers. Some examples of irrational numbers are: 2, 3 7, , 0.101001000....

12
The decimal representation of an irrational number is nonterminating and nonrepeating.
There is no standard notation for the set of irrational numbers.

The set of real numbers is the union of rational numbers and irrational numbers.
That is, R = {Rationals} {Irrationals}.

It is clear that every natural number is a whole number. This we denote by

N N0 .

The set of integers includes natural numbers and their negatives. Therefore

N0 Z.

Every integer is a rational number in the sense that one can write an integer m in the form
m
m = . It therefore follows that
1
Z Q.
We therefore have the following containments:

N N0 Z Q R.

We can represent these containments using the following diagram


' $
' $
' $
' $
'$

N N0 Z Q R
&%
& %
& %
& %
& %

Properties of Real Numbers

Let a, b, and c be real numbers.

13
Property Name of the property

(i) a + b is a real number Closure property for addition


(When we add two real numbers we get
a real number)
a b is a real number Closure property for multiplication
(When we multiply two real numbers we
get a real number)

(ii) a + b = b + a Commutative property for addition


ab = ba Commutative property for multiplication

(iii) (a + b) + c = a + (b + c) Associative property for addition


(a b) c = a (b c) Associatative property for multiplication

(iv) a(b + c) = a b + a c Distributive property


(a + b)c = a c + b c Distributive property

(v) a + 0 = 0 + a = a Identity property for addition


a1= 1a= a Identity property for multiplication

(vi) a + (a) = (a) + a = 0 Additive Inverse property


1 1
For a 6= 0, a = a = 1 Multiplicative Inverse property
a a

Note: The commutative and associative properties do not hold for subtraction or division.

a (b c) is not equal to (a b) c
a (b c) is not equal to (a b) c
ab is not equal to b a
ab is not equal to b a

Use some examples to convince yourself that commutative and associative properties may fail
for the operation of subtraction or division.

One of the main aspects of Mathematics is the ability to construct a mathematical argument
to justify (prove) a statement that one makes. A Proof is a sequence of logical statements
aimed at establishing the truth of some statement.

Let us prove that 2 is not a rational number.

Theorem 2.1. The real number 2 is irrational.
a
Proof. Assume that 2 is a rational number. Then 2 can be written in the form where
b
a and b are integers, with b 6= 0. That is,
a
2= . (?)
b

14
We assume that a and b have no factors in common except 1. Squarring both sides of (?),
we have that
a2
2= 2 or equivalently a2 = 2b2 . (??)
b

This means that a2 is an even number. But if the square of a number is even, then the number
itself must be even. That is, a is an even number. This means that a can be written in the
form a = 2k, where k is an integer. Therefore, we can write (??) as

a2 = 2b2 or equivalently (2k)2 = 2b2 or equivalently 4k 2 = 2b2 . (? ? ?)

It follows from (? ? ?) that b2 = 2k 2 , which shows that b2 is an even number. This, in turn,
implies that b is an even number.

Our argument has established that both a and b are even numbers. This, of course, means
that a and b have a common factor of 2. This contradicts our assumption that a and b have

no common factors except 1. It now follows that our original assumption that 2 is rational

is false. Therefore 2 is irrational. 

Note: The method of proof used in the above Theorem is called Proof by Contradiction.

2.2 Graphical representation of real numbers

Real numbers can represented as points on the number line. To every real number corresponds
one and only one point on the number line, and to every point on the real line corresponds
one and only one real number.

The number line is such that all the positive and negative integers are represented by a set of
equally-spaced point on a line. All the rational numbers which are not integers also have their
appropriate places on the number line.

>
3 1 5
4 3 2 2
1 0 2
1 2 2
3

It is clear that between any two rational numbers we can insert another rational number,
a+b
namely their average. Between a and b there is the number . There are infintely many
2
rational numbers between any two rational numbers.
3 31
Example: Find a rational number between 100
and 1000
(i.e. between 0.03 and 0.031)

Solution: One such number is


 
3 31 61 305
+ 2= = = 0, 0305
100 1000 2000 10000

15
There are still points left on the number line which represent irrational numbers.

Let us consider the irrational number 2:

1< 2< 4 or equivalently 1 < 2 < 2.

Where does the irrational numbers 2 lie on the number line?
We have methods for finding geometrically irrational numbers.


2 1

B
O 1 2 2

If the above triangle was constructed, the hypotenuse would be 2 (Theorem of Pythagoras).

OA = OB = 2. Thus point B on the number line respresents 2.

Real numbers are ordered. Let a and b be real numbers. We say that a is less than b, and
write a < b, if b a is a positive real number. Geometrically, this means that a lies to the
left of b on the real number line. The symbol a b means that either a < b or a = b. For
example, 2 7.

Exercise

1. Identify each of the following statements as True or False. Justify your answers.

(a) Every real number is a rational number.


(b) All integers are rational numbers.
(c) Some irrational numbers are rational numbers.
(d) Every rational number is a real number.
(e) All rational numbers are integers.
(f) Zero is a rational number.
(g) All whole numbers are integers.
(h) All integers are natural numbers.
(i) The product of two rational numbers is always a rational number.
(j) The sum of two rational numbers is always a rational number.
(k) The product of two irrational numbers is always an irrational number.
(l) The sum of two irrational numbers is always an irrational number.

16
2. Complete the following table:

4 1 22 3
0.121221 . . . 2 0. 3
2 7 2
N X
Z X
Q X
R Y

3. Write down any three numbers which lie between 0.032 and 0.033.

4. Find the rational number which lies half-way between


1 2
(a) 2
and 3

(b) 0.25 and 1.25

and place all three numbers on the number line.

5. The average of n numbers is


the sum of the numbers
.
n
Find the average of the following numbers.

(a) 1, 12, 13 (b) 2, 4, 6, 8 (c) 1, 2, 3, . . . , 10

6. Convert each of the follwing fractions to decimal form (NC)


1 3 2
(a) 2 (b) (c)
3 11 15
7. Express each of the following as a common fraction and then check your answer by using
your calculator.

(a) 0. 3 (b) 0. 03 (c) 0. 36 (d) 0. 1 2 3.

8. Use < signs to write the following triples in order of size from the smallest to the largest.

(a) 0.0 4; 0. 04; 0.04
(b) 3.45; 3.445; 3.4445

(c) 1.2 3; 1.23; 1. 23

9. Prove that 3 is irrational.

10. Construct 5 and 3 on the number line. Explain your method.

11. Locate (approximately) the following


numbers on the number line without the use of a
1 3
calculator: 5, 1 + 2, , .
2

17
2.3 Operations on real numbers

In this section we briefly discuss some algebraic operations involving rational numbers and
those that involve surds.

Let a, b, c, and d be integers. We define the sum, difference, product and quotient of two
a c
rational number and by
b d
a c ad + bc
+ =
b d bd
a c ad bc
=
b d bd
a c ac
=
b d bd
a c a d ad
= =
b d b c bc

1 1 1
NOTE: 6= + .
a+b a b
Examples:
2 4 27+43 14 + 12 26
+ = = =
3 7 37 21 21
2 4 2743 14 12 2
= = =
3 7 37 21 21
2 4 24 8
= =
3 7 37 21
2 4 2 7 27 14 7
= = = = .
3 7 3 4 34 12 6

Let a, b, c, and d be real numbers such that a and b are nonnegative. Then


(i) a b = a b.
r
a a
(ii) = , b > 0.
b b

(iii) c a + d a = (c + d) a.

(iv) c a d a = (c d) a.


NOTE: a + b 6= a + b.

18
Examples


1. (a) 23 2+5 2 =3 2


(b) 3 3 + 2 2 ( 3 + 2) = 3 3 + 2 2 3 2 = 2 3 + 2


(c) 2 3( 3 + 3 2) = 2 3 3 + 6 3 2 = 2 9 + 6 6 = 6 + 6 6


(d) 50 +20 125 = 25.2 + 4.5 25.5 = 25 2 + 4 5 25 5

=5 2+2 55 5= 5 23 5

(e)

5 2 2 5 2(3 + 3) + 2(3 3)
+ =
3 3 3+ 3 (3 3)(3 + 3)

15 2 + 5 6 + 3 2 6
=
93

18 2 + 4 6 9 2+2 6
= = .
6 3

p q q
(f) 3 5(3 + 3) = 5(3 + 3)2 = (3 + 3) 5
3+
p
2. Let x = 4 3 and y = 19 8 3.

(a) Find x2 and y 2.


(b) Is x equal to y?

Solution:

(a) x2 = (4p 3)2 = 16 8 3 + 3 = 19 8 3

y 2 = ( 19 8 3 )2 = 19 8 3

(b) x = y since x2 = y 2 and x > 0 and y > 0.


Note: x2 = y 2 if and only if x = y or x = y.

3. Express each of the following numbers with a rational denominator. [The process of
rewriting a number with a surd in the denominator so that the denominator is a rational
number is called rationalising the denominator.

1+ 3 1 1 7 2
(a) (b) (c) (d) .
2 2+1 1+ 2+ 3 7+ 2

19
!
1+ 3 1+ 3 2 1+ 3 2+ 6
Soln: (a) = = 2 =
2 2 2 2 2

1 1 21 21 21
(b) = = = = 21
2+1 2+1 21 ( 2)2 12 21
(since (a + b)(a b) = a2 b2 )
(c)
" #
1 1 (1 + 2) 3
=
1+ 2+ 3 1 + 2 + 3 (1 + 2) 3

1+ 2 3
=
(1 + 2)2 ( 3)2

1+ 2 3 1+ 2 3
= =
1+2 2+23 2 2

1+ 2 3 1+ 2 3 2 2 1+ 2 3
But = = .
2 2 2 2 2 4

1 2 1+ 2 3
Therefore = .
1+ 2+ 3 4

(d)
!
7 2 7 2 7 2
=
7+ 2 7+ 2 7 2

72 7 2+2 9 2 14
= 2 2 =
7 2 72

9 2 14
= .
5
Exercise

1. Simplify the following.


1 1 1 2 5 2 1
(a) + + (b) (c) 2 1
2 3 4 3 6 3 2
0.128
(d) (e) 0.03 0.4 0.6 0.01
0.02

2. Show that
3
(6 53 3 14 ) 5 65
(a) = 2.5
(21 1.25) 2.5

20
(b) (7 19 2 14
15
) (2 23 + 1 35 ) ( 34 1
20
) ( 75 5
14
) = 35
48
.

3. Simplify the following.



(a) 50 + 98 8

(b) 16 + 9 + 20 45

75 + 48
(c)
12

2 18 32
(d)
8+ 2
p p
(e) 5 5 + 5 5 + 5
p
4. Let x = 21 4 5 and y = 2 5 1. Is x equal to y?

5. Rationalize the following.



10 1 2 1
(a) (b) (c) .
3 2 2 3+1 2+ 3 5
6. If 7 books cost R105, what will 5 of these books cost?

7. If petrol costs R4.20 per litre and the price is decreased by 10%, what is the new price
of petrol?

8. A car costing R50 000 depreciates at a rate of 12% per year. What is the car worth
after 2 years?

9. Determine what sum must be invested at 10% p.a. to yield R5 500 after one year?

10. An amount of R1 000 is invested at 10% p.a. (CI). Find the amount in the account
after

(a) one year (b) two years (c) three years.

Compound interest investment is a method of building up capital (C.I).

21
Chapter 3

Algebraic Expressions

We start this chapter by defining some common terminology used in Mathematics.

A variable is a letter used to represent a generic real number. We often use the letters
x, y, and z for variables. Other letters of the alphabet are also used.

An algebraic expression is a number, variable or combination of the two connected


by some mathematical operation like addition, subtraction, multiplication, division, ex-
ponents, and/or roots. Some examples of algebraic epressions are: 2xy, xy 2 x2 y,
x3 + 2xy 4.

The terms of an algebraic expression are the elements that are separated by the plus
(+) or minus () signs. For example, the expression 3xy 5x2y 3 + 2 has three terms.

- A monomial is an algebraic expression containing only one term (mono means


one). For example, , 3xy is a monomial.
- A binomial is an algebraic expression containing two terms (bi means two). The
algebraic expression 2xy + 5x2 y is a binomial expression.
- A trinomial is an algebraic expression with three terms (tri means three). The
algebraic expression 2xy + 5x2 y 3 is a trinomial expression.
- A multinomial is an algebraic expression with two or more terms.

Coefficients are the number part of the terms with variables. For example, in the
expression 3xy 5x2y 3 + 2, 3 is the coefficient of xy, and 5 is the coefficient of x2 y 3.

A constant term in an algebraic expression is a term that contains only a number.


A constant term is not multiplied by any variable. For example, in the expression
3xy 5x2 y 3 + 2, the number 2 is a constant term.

Like terms in an algebraic expression are terms with the same variables raised to
the same powers and differ only in numerical coefficients. In the algebraic expression
3x2 y 2xy 2 + 3xy + 6x2 y + 7xy 2 xy, the terms 3x2 y and 6x2 y are like terms. So
are the terms 2xy 2 and 7xy 2 ; and so are the terms 3xy and xy.

22
To simplify an algebraic expression means writing it in a compact or efficient manner,
without changing its value. This mainly involves multiplying out brackets and adding
together like term.

Special Products

The following products occur so frequently in Mathematics that one has to be able to quickly
recognise them.

1. (x y)(x + y) = x2 y 2 (Difference of squares).

2. (x + y)2 = x2 + 2xy + y 2. (Square of sum).

3. (x y)2 = x2 2xy + y 2 . (Square of difference).

4. (x + a)(x + b) = x2 + (a + b)x + ab.

5. (x + y)3 = x3 + 3x2 y + 3xy 2 + y 3. (Cube of sum).

6. (x y)3 = x3 3x2 y + 3xy 2 y 3 . (Cube of difference).

7. x3 y 3 = (x y)(x2 + xy + y 2) (Difference of cubes).

8. x3 + y 3 = (x + y)(x2 xy + y 2 ) (Sum of cubes).

NOTE: (x + y)2 6= x2 + y 2 [ i.e., (x + y)2 is NOT equal to x2 + y 2 ],

(x y)2 6= x2 y 2 [ i.e., (x y)2 is NOT equal to x2 y 2 ],

(x + y)3 6= x3 + y 3 [ i.e., (x + y)3 is NOT equal to x3 + y 3 ],

(x y)3 6= x3 y 3 . [ i.e., (x y)3 is NOT equal to x3 y 3 ]

Examples 3.1. 1. Simplify the expression 7y 2 2xy 2x + 8 + 5xy 4y 2 + 2y 3.

Soln:

7y 2 2xy 2x + 8 + 5xy 4y 2 + 2y 3 = 7y 2 4y 2 2xy + 5xy 2x + 2y + 8 3


= (7 4)y 2 + (2 + 5)xy 2x + 2y + (8 3)
= 3y 2 + 3xy 2x + 2y + 5.

2. Simplify the expression 3(x 1)2 + (x + 2)(x2 2x + 4) (x + 1)3

Soln: 3(x 1)2 + (x + 2)(x2 2x + 4) (x + 1)3 = 3(x2 2x + 1) + x3 + 8 (x3 + 3x2 + 3x + 1)


= 9x + 10.

23
3. Simplify the expression (x 1)3 4x(x + 1)(x 1) + 3(x 1)(x2 + x + 1).

Soln:

(x 1)3 4x(x + 1)(x 1) + 3(x 1)(x2 + x + 1)


= x3 3x2 + 3x 1 4x(x2 1) + 3(x3 1)
= 3x2 + 7x 4.

4. Simplify the expression (x2 1)(x4 + x2 + 1) (x2 1)3 .

Soln:

(x2 1)(x4 + x2 + 1) (x2 1)3 = (x2)3 1 (x6 3x4 + 3x2 1)


= 3x4 3x2 .

5. Simplify (x + y + z)2.

Soln:

(x + y + z)2 = [(x + y) + z]2 = (x + y)2 + 2z(x + y) + z 2


= x2 + 2xy + y 2 + 2xz + 2yz + z 2
= x2 + y 2 + z 2 + 2xy + 2xz + 2yz

3.1 Factorisation

It is impossible to simplify certain algebraic expressions without factorization.


Examples 3.2. 1.
(a) Factorise the expression (4x 1)(x + 2) (12x2 3x) + (7 x)(4x 1).
Soln:

(4x 1)(x + 2) (12x2 3x) + (7 x)(4x 1)


= (4x 1)(x + 2) 3x(4x 1) + (7 x)(4x 1)
= (4x 1)(x + 2 3x + 7 x)
= (4x 1)(9 3x) = 3(4x 1)(x 3)

(b) Factorise the expression x3 a2 x (x a)2(2x + 2a).


Soln:

x3 a2 x (x a)2(2x + 2a) = x(x2 a2 ) 2(x a)2 (x + a)


= x(x a)(x + a) 2(x a)2 (x + a)
= (x a)(x + a)[x 2(x a)]
= (x a)(x + a)(x 2x + 2a)
= (x a)(x + a)(x + 2a).

24
(c) Factorise the expression 25(a 2b)2 49(b a)2 .
Soln:

25(a 2b)2 49(b a)2 = [5(a 2b)]2 [7(b a)]2 (a difference of squares)
= [5(a 2b) 7(b a)][5(a 2b) + 7(b a)]
= (12a 17b)(2a 3b).

(d) Factorise the expression (x + y)3 8x3 .


Soln:

(x + y)3 8x3 = (x + y)3 (2x)3 (a difference of cubes)


= [(x + y) 2x][(x + y)2 + 2x(x + y) + (2x)2 ]
= (x + y)(x2 + 2xy + y 2 + 2x2 + 2xy + 4x2 )
= (x + y)(7x2 + 4xy + y 2).

2. (a) Factorise the expression (x2 9)2 (x 3)2 and find the value corresponding to

x = 3 + 3.
Soln:

(x2 9)2 (x 3)2 = (x 3)2 (x + 3)2 (x 3)2


= (x 3)2 [(x + 3)2 1]
= (x 3)2 (x + 3 1)(x + 3 + 1)
= (x 3)2 (x + 2)(x + 4)

If x = 3 + 3 then

(x 3)2 (x + 2)(x + 4) = (3 + 3 3)2 (3 + 3 + 2)(3 + 3 + 4)

= 3(35 + 12 3 + 3)

= 6(19 + 6 3).

(b) Factorise the expression (2x 3)(5x 1) + (3 2x)(x + 1) and find the value
corresponding to x = 112
.
Soln:

(2x 3)(5x 1) + (3 2x)(x + 1) = (2x 3)(5x 1) (2x 3)(x + 1)


= (2x 3)(5x 1 x 1)
= (2x 3)(4x 2)
= 2(2x 3)(2x 1).
11
If x = then
2
11 11
2(2x 3)(2x 1) = 2(2 3)(2 1)
2 2
= 2(11 3)(11 1) = 160.

25
Exercise 3.3. 1. Simplify the following expressions by using the above mentioned special
products:

(a) (a2 3)3 (a 2)(a2 + 4)(a + 2)


(b) (2a 3)3 4a(2a + 3)(2a 3) + (3 2a)2

(c) (8 a b)(8 a + b) ( a b)2
(d) (3 + x)2 + 5(1 x)2 3(1 x)(1 + x)
(e) (2a + b c)2

2. Factorise the following expressions.

(a) (x y)3 (x y)
(b) x4 10x2 y 2 + 9y 4
(c) (a 2)2 + 5(a 2) + 4
(d) 3(x 1)2 4(x + 3)2
(e) 1 + x x3 x2
(f) (x + y)3 + (2y x)3

3. Factorise the following expressions and then find their values corresponding to the given
x value.

(a) (2x2 + 3x + 2)2 (2x2 3x 6)2 ; x = 2 + 1.

1
(b) (4x2 1)2 (2x + 1)2 ; x= .
31

3.2 Algebraic Fractions

An algebraic fraction is a fraction in which both the numerator and the denominator are
algebraic expressions.

Here are some examples of algebraic fractions

x x2y 2xy + 3 x3 3xy 2 + 3x2 y 5


, , .
y xy 3 + 4xy y + x 1 xy

An algebraic fraction is defined only where the denominator is not zero. For example, the
x5
algebraic fraction is defined everywhere except when x = 1 or when x = 2.
(x + 1)(x 2)

26
1 1 1 1 1 1
NOTE: 6= + i.e., is NOT the same as + .
A+B A B A+B A B

Division by zero is not allowed in Mathematics. An algebraic fraction will only be defined
where the denominator is not zero. When we define an algebraic fraction, we shall always
(explicitly or implicitly) impose the restriction that the denominator is not zero.
1
For example, the algebraic fraction is defined for all real numbers x except for
x(x + 1)
1
x = 0 and x = 1. Therefore, for the algebraic fraction , the restriction is that
x(x + 1)
x 6= 0; x 6= 1.

In the following table, we give some examples of algebraic fractions and the restrictions that
make them well-defined.

Algebraic Fraction Restrictions

2x + 5 1 1
x 6= 0; x 6= ; x 6=
x2(2x 1)(x2 + 1)(3x 1) 2 3

1
a 6= b; a 6= 2b
(a b)2 (a 2b)2

4x 7
x 6= 1; x 6= 3
(x2 1)(x + 3)(4x2 + 1)

3.2.1 Simplifying algebraic fractions by cancelling out common


factors

Just like in numerical fractions, rational fractions can be simplified by cancelling out common
factors in the numerator and denominator. For an example, in order to simplify the numerical
15
fraction we first factorise the numerator and the denominator and then cancel out common
25
factors:
15 35 3 6 5 3
= = = .
25 55 5 6 5 5
There are two steps involved in simplifying an algebraic fraction:

Factorise the numerator and the denominator.

Cancel out common factors in the numerator and denominator.

27
3x2 + xy 2y 2
Examples 3.4. 1. Simplify .
3x2 5xy + 2y 2
Soln:
3x2 + xy 2y 2 (3x 2y)(x + y)
2 2
= (Factorising the numerator and
3x 5xy + 2y (3x 2y)(x y)
the denominator)

(3x
////////
2y)(x + y)
= (Cancel out common factors)
(3x
////////
2y)(x y)

x+y
=
xy
3x3 + 17x2 + 10x
2. Simplify .
x3 + 125
Soln:
3x3 + 17x2 + 10x x(x + 5)(3x + 2)
= (Factorising the numerator and
x3 + 125 (x + 5)(x2 5x + 25)
the denominator)

x(/////
x + 5)(3x + 2)
= (Cross out x + 5 in the numerator
x + 5)(x2 5x + 25)
(/////
and the denominator)

x(3x + 2)
=
x2 5x + 25
x2 + x
3. (a) Simplify the expression and state restrictions for its definition.
x
x2 + x x(x + 1)
Soln. = = x + 1, x 6= 0.
x x

2x2 2xy
(b) Simplify the expression and state restrictions for its definition.
4x2 4y 2
2x2 2xy 2x(x y) x
Soln. = = , x 6= y; x 6= y.
4x2 4y 2 4(x y)(x + y) 2(x + y)

a2 2a 15
(c) Simplify the expression and state restrictions for its definition.
a2 25
a2 2a 15 (a 5)(a + 3) a+3
Soln. = = , a 6= 5.
a2 25 (a 5)(a + 5) a+5

5x2 y 3 10xy 2
(d) Simplify the expression and state restrictions for its definition.
5x3 y 3
5x2y 3 10xy 2 5xy 2(xy 2) xy 2
Soln. 3 3
= 2 3
= , x 6= 0; y 6= 0.
5x y 5x y x2 y
x2 6x + 8
(e) Simplify the expression and state restrictions for its definition.
x4 8x

28
x2 6x + 8 (x 4)(x 2) (x 4)(x 2)
Soln. = =
x4 8x x(x3 8) x(x 2)(x2 + 2x + 4)
x4
= , x 6= 0; x 6= 2.
x(x 2)(x2 + 2x + 4)

3.2.2 Multiplication and Division of algebraic fractions

Multiplication of algebraic fractions is similar to multiplication of numerical fractions - multiply


the numerators and multiply the denominators:
A C AC
=
B D BD
and cancel out all common factors.

Here are some illustrative examples.


2x 4 x+3
Examples 3.5. 1. Simplify .
x2 + x 6 x + 1
Soln:
2x 4 x+3 2(x 2) x+3
= (Factorise the numerators
x2 + x 6 x + 1 (x 2)(x + 3) x + 1
and denominators)

2 (x
/////
2) x/////
+3
= (Cancel out common factors)
(x
/////
2) (x
/////
+ 3) x + 1

2
= .
x+1

3x2 x2 1
2. Simplify the expression .
x2 x x2 + x
Soln:
3x2 x2 1 3xx (x 1)(x + 1)
2
2
= (Factorise the numerators
x x x +x x(x 1) x(x + 1)
and denominators)

3 6 x x (x 1)/////
(x + 1)
= (Cancel out common factors)
6 x(x 1) x (x
/////
+ 1)

3x(x 1)
=
x (x 1)

3 6 x (x
/////
1)
= (Cancel out common factors)
6 x (x
/////
1)

= 3.

29
a 2 b2 ab + b2
3. Simplify 2
a + 2ab + b2 2a2 b 2ab2
Soln:
a 2 b2 ab + b2 (a b)(a + b) b(a + b)
2 2
2 2
=
a + 2ab + b 2a b 2ab (a + b)2 2ab(a b)
1
= .
2a

x2 7x + 12 2x2 + 10x
4. Simplify the expression .
x2 25 x2 9
Soln:
x2 7x + 12 2x2 + 10x (x 4)(x 3) 2x(x + 5)
=
x2 25 x2 9 (x 5)(x + 5) (x 3)(x + 3)
2x(x 4)
=
(x 5)(x + 3)
2x2 8x
= 2 .
x 2x 15

Division of algebraic fractions is performed by inverting the second fraction and multiplying.
5x 7 x+2
Examples 3.6. 1. Simplify
4x + 12 2x + 6
Soln:
5x 7 x+2 5x 7 2x + 6
= (Invert the second fraction
4x + 12 2x + 6 4x + 12 x+2
and multiply)

5x 7 2(x + 3)
= (Factorise the numerator
2 2 (x + 3) x+2
and denominator)

5x 7 6 2 (x
/////
+ 3)
= (Cancel out common fac-
2 6 2 (x
/////
+ 3) x+2
tors)

5x 7
=
2(x + 2)

x2 x 20 x2 x 6
2. Simplify .
x2 + 6x + 8 x2 + 4x + 4
Soln:

30
x2 x 20 x2 x 6 x2 x 20 x2 + 4x + 4
2 = 2 2 (Invert the second fraction
x2 + 6x + 8 x + 4x + 4 x + 6x + 8 x x6
and multiply)

(x + 4)(x 5) (x + 2)(x + 2)
= (Factorise the numerator
(x + 4)(x + 2) (x + 2)(x 3)
and denominator)

(x
/////
+ 4)(x 5) (x
/////
+ 2)(x + 2)
= (Cancel out common fac-
(x
/////
+ 4)(x + 2) (x
/////
+ 2)(x 3)
tors)

x5 x+2
=
x+2 x3

x5 x /////
+2
= (Cancel out common factors)
x/////
+2 x3

x5
= .
x3

2x2 + 2x 1
3. Simplify 2
.
x x x1
2x2 + 2x 1 2x(x + 1)
Soln: 2
= (x 1) = 2(x + 1).
x x x1 x(x 1)

2x3 32x x2 4x
4. Simplify .
x2 + x x
Soln:
2x3 32x x2 4x 2x(x2 16) x
2
=
x +x x x(x + 1) x(x 4)
2(x 4)(x + 4) 1
=
x+1 x4
2(x + 4) 2x + 8
= = .
x+1 x+1

x
5. Simplify x+2 .
x2 + 3x
x2 4

31
Soln:
x x
x+2 = x+2
x2 + 3x x(x + 3)
x2 4 (x 2)(x + 2)
x (x 2)(x + 2)
=
x+2 x(x + 3)
x2
=
x+3
Exercise 3.7. 1. Write down all restrictions for the following fractions:
2x
(a)
(x + 5)3
1
(b) 2
2x 50
1
(c)
x4 (x4
+ 2)(3x 12)
a4
(d)
a(a b2)(a6 + 1)
2

2. Simplify (assume all denominators are non-zero):


x2 x x2 4
(a)
x2 + x 2 x2 3x + 2
x4 81x2 x2 9x
(b)
2x 6x
x x 4 x2 9
2
(c)
x + 2 x2 + 3x x2 2x
3x(a 2b) + y(a 2b) a2 + 2ab + b2
(d)
9x2 (a + b) y 2 (a + b) 3x2 xy
1ab
(e)
1 (a + b)2
2ab + 6ac 3ab3 + 6ab2 b2 9
(f)
6b3 + 12b2 2ab + 6a b + 3c

3.2.3 Addition and Subtraction of algebraic fractions

Addition and subtraction of algebraic fractions follow the same procedure as for numerical frac-
tions. We shall use simple examples to illustrate the step-by-step process of adding algebraic
fractions.

A C A+C
NOTE: + 6= i.e., do not add numerators and denominators
B D B+D
separately.

32
2 1
Examples 3.8. 1. Simplify the expression + .
x+1 x+2
Soln:

The least common multiple of the denominators x + 1 and x + 2 is (x + 1)(x + 2).


Therefore
2 1 2 x+2 1 x+1
+ = +
x+1 x+2 x+1 x+2 x+2 x+1

2(x + 2) x+1
= +
(x + 1)(x + 2) (x + 1)(x + 2)

2(x + 2) + x + 1
=
(x + 1)(x + 2)

3x + 5
= .
(x + 1)(x + 2)

2x 2x + 1
2. Simplify the expression + 2 .
x24 x +x6
Soln:
2x 2x + 1 2x 2x + 1
+ = + (Factorise the numerator and
x2 4 x2 + x 6 (x 2)(x + 2) (x 2)(x + 3)
the denominator)

2x(x + 3) + (2x + 1)(x + 2)


= (Find the least multiple of the
(x 2)(x + 2)(x + 3)
denominator (LCM))

4x2 + 11x + 2
= (Multiply out and simplify the
(x 2)(x + 2)(x + 3)
numerator)

a 1 a + 3 4 + a2
3. Simplify the expression + .
3a2 4a 6a2
Soln:
a 1 a + 3 4 + a2 4(a 1) + 3a(a + 3) 2(4 + a2)
+ =
3a2 4a 6a2 12a2

4a 4 + 3a2 + 9a 8 2a2
=
12a2

a2 + 13a 12
= .
12a2

1 1
4. Simplify + .
xy x+y

33
1 1 x+y+xy 2x 2x
Soln: + = = = 2 .
xy x+y (x y)(x + y) (x y)(x + y) x y2

2x 1 x + 4 3x 1
5. Simplify + + 2 .
x+3 x 6 x 3x 18
Soln:
2x 1 x + 4 3x 1 2x 1 x + 4 3x 1
+ + 2 = + +
x+3 x 6 x 3x 18 x+3 x 6 (x 6)(x + 3)
(2x 1)(x 6) + (x + 3)(x + 4) + 3x 1
=
(x 6)(x + 3)

3x2 3x + 17
=
(x 6)(x + 3)

   
5a 5x 10ax a x 2a
6. Simplify the expression + + 2 + .
a + x a x a x2 a + x a x a 2 x2
Soln:
   
5a 5x 10ax a x 2a
+ + 2 + 2
a + x a x a x2 a + x a x a x2
5a(a x) + 5x(a + x) + 10ax a(a x) + x(a + x) 2ax
=
(a x)(a + x) (a x)(a + x)

5a2 5ax + 5ax + 5x2 + 10ax (a x)(a + x)


= 2
(a x)(a + x) a ax + ax + x2 2ax

5(a2 + 2ax + x2)


=
(a x2 )

(a + x)2
=5
(a x)2
 2
a+x
=5
ax

   
b 2 a b a
7. Simplify 2
+ 2 2+
a + ab a + b b + ab a b

34
Soln:
   
b 2 a b a
+ 2+
a2 + ab a + b b2 + ab a b
 
b 2 a b2 2ab + a2
= +
a(a + b) a + b b(a + b) ab

b2 2ab + a2 ab
= 2
ab(a + b) b 2ab + a2
1
=
a+b

 
a a2  a x3
8. Simplify 1 + + 2 1 3 .
x x x a x3
Soln:
 
a a2 a x3 x2 + ax + a2 x a x3
1+ + 2 1 3 =
x x x a x3 x2 x (a x)(a2 + ax + x2)
xa
=
ax
(a x)
= = 1.
ax

    
2a + 1 2a 1 1 1 2(2a 1)
9. Show that 1 1 + 2 = .
2a 1 2a + 1 a 4a a(2a + 1)
Soln: We simplify the left-hand side and show that it is the same as the right-hand side.
    
2a + 1 2a 1 1 1
1 1 + 2
2a 1 2a + 1 a 4a
 
(2a + 1)2 (2a 1)2 4a2 4a + 1
= 1
(2a 1)(2a + 1) 4a2

(2a + 1)2 (2a 1)2 ) 4a2


=
(2a 1)(2a + 1) (2a 1)2

(2a + 1 + 2a 1)(2a + 1 2a + 1) (2a 1)2


=
(2a 1)(2a + 1) 4a2

4a 2 (2a 1)2
=
(2a 1)(2a + 1) 4a2
2(2a 1)
= .
a(2a + 1)

35
a2 +b2 2 2
b
+ 2a 2b a +b
10. Simplify 1 + 1 1a .
b
+ 1a b
a
Soln:
a2 +b2 2 2 a2 +b2 +2ab 2aba2 b2
b
+ 2a 2b a +b
1 + 1 1a = b
+ a

b
+ 1a b
a a+b
ab
ab
ab

(a + b)2 ab [(a b)2 ] ab


= +
b a+b a ab
= a(a + b) b(a b)
= a2 + ab ab + b2
= a 2 + b2 .

1 1 1 1
x
y x2
y2
11. Simplify 1 1 1 1 .
x
+ y x2
+ y2

Soln:
1 1 1 1 yx y2 x2
x
y x2
y2 xy x2 y 2
1 1 1 1 = x+y x2 +y2
x
+ y x2
+ y2 xy x2 y 2
   
yx xy y 2 x2 x2 y 2
= 2
xy x+y x2 y 2 x + y2

y x y 2 x2
=
x + y x2 + y 2

y x x2 + y 2
=
x + y y 2 x2

yx x2 + y 2
=
x + y (y x)(x + y)

x2 + y 2
= .
(x + y)2

1 1 1 1
x
y+z y
+ x+z
12. Simplify 1 1 1 1 .
x
+ y+z y
x+z

36
Soln:
1 1 1 1 y+zx x+y+z
x
y+z y
+ x+z x(y+z) y(x+z)
1 1 1 1 = x+y+z x+zy
x
+ y+z y
x+z x(y+z) y(x+z)

y + z x x(y + z) x + y + z y(x + z)
=
x(y + z) x + y + z y(x + z) x + z y
y+zx
= .
x+zy

Exercise 3.9. 1. Simplify each of the following expressions and, in each case, state all
the restrictions.
5x2 + 4x 1 x
(a) 2
.
x x (x 1)2
5x + 1 8x x+1
(b) + .
3x 3 5(x 1) (x 1)2
a b a + b a 2 + b2
(c) + .
a + b a b a 2 b2
1 1 4a2
(d) + 4 .
a+1 a1 a 1
8x 3x 2 6x
(e) 3
+ 2
.
x 9x x + 3x (1 3x)2
6 10
(f) 2
2 .
x 3x 54 x + 5x 6
2x + 5 3x 1 5
(g) 2
2 + .
x + 3x 18 x + 4x 12 x 2
x3 2x2 + 19x 46 x + 4
(h) + .
2x + 1 2x2 9x 5 x5
2. Verify that each of the following is true. (Assume that all denominators are non-zero.)
 2  
a ab 2a2 b1 b a+1
(a) 2 3
3 2 2 3
1 2 = .
a b+b b ab + a b a a a ab
   
1 x2
(b) x + 1 x = x.
1x x1
   
4ab a b 2ab
(c) a +b = a b.
a+b a + b b a a 2 b2
     
b 2 + c2 1 1 1 1 a 2 + c2 a 2 + b2 a 2 b2
(d) 2 2 = .
b 2 c2 b 2 c a 2 c a 2 c2 a 2 b2 a 2 b2
    
x+1 1x 4x2 1 1x x+1
(e) 2 2 3 2
2
1 = 2 .
1x x+1 x 1 x +x x x1

37
3. Simplify each of the following algebraic fractions.
1 + x3
(a) .
1 x6

x
(b) 1 + .
1 + x1
2 4
x
x+2
(c) 3 .
x2 +2x
+ x3
2 3
xy
+ x+y
(d) 5 1 .
x+y
x2 y 2

1 1
 b2 +c2 a2

a
+ b+c
1+ 2bc
(e) 1 1 .
a
b+c

2 1 1
(f)  2 +  2 .
3 2x+1 2x1
1+
3
1+
3
 
3a 1 xa x3 27
(g) 2 2 .
9 3x 3a + ax a 9 3a + 9a 3a

3.3 Partial Fraction Decomposition

We have seen in subsection 3.2.3 that we can add/ substract algebraic fractions to form a
single algebraic fraction. Partial fraction decomposition is the process of breaking up
an algebraic fraction into a sum of its constituent parts. In this section we discuss partial
fraction decomposition of simple algebraic fractions in which the denominators are products
of linear factors. We illustrate partial fraction decomposition by first adding rational fractions
and then decomposing their sum into partial fractions:
1 2 1
Example Simplify the expression + .
x x+1 x+2
Soln: The lowest common multiple of the denominators is x(x + 1)(x + 2). Therefore

38
1 2 1 1 (x + 1)(x + 2) 2 x(x + 2) 1 x(x + 1)
+ = +
x x+1 x+2 x (x + 1)(x + 2) x + 1 x(x + 2) x + 2 x(x + 1)
(x + 1)(x + 2) + 2x(x + 2) x(x + 1)
=
x(x + 1)(x + 2)

x2 + 3x + 2 + 2x2 + 4x x2 x
=
x(x + 1)(x + 2)

2x2 + 6x + 2
= .
x3 + 3x2 + 2x

2x2 + 6x + 2
We now want to reverse the above process - we want to write the algebraic fraction
x3 + 3x2 + 2x
as a sum of its constituent algebraic fractions. The first step is to factorise the denominator.
2x2 + 6x + 2
Find the partial fraction decomposition of the algebraic fraction .
x3 + 3x2 + 2x
Soln:
2x2 + 6x + 2 2x2 + 6x + 2
= (Factorise the denominator)
x3 + 3x2 + 2x x(x + 1)(x + 2)

A B C
= + + (where A, B, and C
x x+1 x+2
are constants to be deter-
mined)

A(x + 1)(x + 2) + Bx(x + 2) + Cx(x + 1)


= (Combining the algebraic
x(x + 1)(x + 2)
fractions into a single one)

That is,

2x2 + 6x + 2 A(x + 1)(x + 2) + Bx(x + 2) + Cx(x + 1)


3 2
= .
x + 3x + 2x x(x + 1)(x + 2)

Since the denominators in the above algebraic fractions are the same, their numerators must
also be equal. Therefore

2x2 + 6x + 2 = A(x + 1)(x + 2) + Bx(x + 2) + Cx(x + 1).

We now find the constants A, B, and C.

39
x = 0: 2 02 + 6 0 + 2 = A(0 + 1)(0 + 2) + B 0 (0 + 2) + C 0 (0 + 1)

2 = 2A and therefore A = 1.

x = 1: 2(1)2 + 6(1) + 2 = A(1 + 1)(1 + 2) + B(1)(1 + 2) + C(1) (1 + 1)

2 6 + 2 = B and therefore B = 2.

x = 2: 2(2)2 + 6(2) + 2 = A(2 + 1)(2 + 2) + B(2)(2 + 2) + C(2) (2 + 1)

8 12 + 2 = 2C and therefore C = 1.

2x2 + 6x + 2
It now follows that the partial fraction decomposition of the algebraic fraction
x3 + 3x2 + 2x
is
2x2 + 6x + 2 1 2 1
= + .
x3 + 3x2 + 2x x x+1 x+2
x+2
Examples 3.10. 1. Find the partial fraction decomposition of .
x2 + 4x + 3
Soln:
x+2 x+2
= (Factorise the denominator)
x2 + 4x + 3 (x + 1)(x + 3)

A B
= + (where A and B are constants to
x+1 x+3
be determined)

A(x + 3) + B(x + 1)
= (Combining the algebraic fractions
(x + 1)(x + 3)
into a single one).

That is,
x+2 A(x + 3) + B(x + 1)
= .
x2 + 4x + 3 (x + 1)(x + 3)
Since the denominators in the above algebraic fractions are the same, their numerators
must also be equal. Therefore

x + 2 = A(x + 3) + B(x + 1).

We now find the constants A and B.


1
x = 1 1 + 2 = A(1 + 3) + B(1 + 1) or equivalently 1 = 2A and so A = .
2
1
x = 3 3 + 2 = A(3 + 3) + B(3 + 1) or equivalently 1 = 2B and so B = .
2

40
x+2
It follows that the partial fraction decomposition of the algebraic fraction
x2 + 4x + 3
is
1 1
x+2 2 2
= + .
x2 + 4x + 3 x+1 x+3
2x2 + x 12
2. Find the partial fraction decomposition of .
x(x + 2)(x + 3)
Soln: Note that the denominator is already in factor form.
2x2 + x 12 A B C
= + + , where A, B, and C are
x(x + 2)(x + 3) x x+2 x+3
constants to be deter-
mined.

A(x + 2)(x + 3) + Bx(x + 3) + Cx(x + 2)


= (Combining the algebraic
x(x + 2)(x + 3)
fractions into a single one)

That is,
2x2 + x 12 A(x + 2)(x + 3) + Bx(x + 3) + Cx(x + 2)
= .
x(x + 2)(x + 3) x(x + 2)(x + 3)

Since the denominators in the above algebraic fractions are the same, their numerators
must also be equal. Therefore

2x2 + x 12 = A(x + 2)(x + 3) + Bx(x + 3) + Cx(x + 2).

We now determine the constants A, B, and C.


x = 0: 2 02 + 0 12 = A(0 + 2)(0 + 3) + B 0 (0 + 3) + C 0 (0 + 2)

12 = 6A and therefore A = 2.

x = 2: 2(2)2 2 12 = A(2 + 2)(2 + 3) + B(2)(2 + 3) + C(2)(2 + 2)

6 = 2B and therefore B = 3.

x = 3: 2(3)2 3 12 = A(3 + 2)(3 + 3) + B(3)(3 + 3) + C(3)(3 + 2)

3 = 3C and therefore C = 1.
2x2 + x 12
It now follows that the partial fraction decomposition of the algebraic fraction
x(x + 2)(x + 3)
is
2x2 + x 12 2 3 1
= + + .
x(x + 2)(x + 3) x x+2 x+3

41
Exercise 3.11. 1. Find the partial fraction decompositon of each of the following alge-
braic fractions.
3x
(a)
(x 5)(x + 1)
x4
(b) 2
x +x2
1
(c) 2
x + 5x + 6
2
(d) 2
x 9
4x2 18x 16
(e) 3
x + 2x2 8x
5x + 1
(f)
(x 1)(x 2)(x 3)

42
Chapter 4

Linear and Quadratic Equations

In this chapter we study linear and quadratic equations. An equation is a mathematical


statement that two mathematical expressions are equal. This chapter discusses the ways of
solving linear and quadratic equations. To solve an equation is to find the set of all values of
the unknown that satisfy the equation.

When solving an equation, it is important to remember that whatever is done on one side of
the equation must also be done on the other.

NOTATION: We are going to use the symbol to say that two statements are logically
equivalent. If two statement A and B are logically equivalent, we shall write A B.

4.1 Linear Equations

Definition 4.1. A linear equation in one variable x is an equation that can be written
in the form
ax + b = 0,
for some real numbers a and b with a 6= 0.

Note that the highest power of the variable x in the linear equation ax + b = 0 is 1.

A linear equation has exactly one solution. Indeed,


ax + b = 0 ax + b + (b) = 0 + (b) (adding b on both sides)
ax = b
1 1 1
ax = (b) (multiplying by both sides)
a a a
b
x= .
a
b
Therefore x = is the only solution of the equation ax + b = 0.
a

43
Examples 4.2. 1. Solve for x, if 3x + 2[x + 2(x 3)] 1 = 5.

Soln:

3x + 2[x + 2(x 3)] 1 = 5 3x + 2(3x 6) = 6


9x = 6 + 12 9x = 18
x = 2.

Therefore x = 2 is the solution of the equation 3x + 2[x + 2(x 3)] 1 = 5.

Check if x = 2 satisfies the equation: If x = 2, then

3x + 2[x + 2(x 3)] 1 = 3 2 + 2[2 + 2(2 3)] 1 = 6 + 2 0 1 = 5.

2. Solve for x, if 3x + 5 = 11.

Soln:
3x + 5 = 11 3x + 5 + (5) = 11 + (5) (adding 5 both sides)
3x = 6
1 1 1
3x = 6 (multiplying by both sides)
3 3 3
x=2

Therefore x = 2 is the solution of the equation 3x + 5 = 11.

Check: If x = 2, then 3x + 5 = 3 2 + 5 = 6 + 5 = 11. Therefore x = 2 is indeed the


solution of the equation 3x + 5 = 11.

3. Solve for x, if 5x + 2 = 2x + 17.

Soln: The trick here is to get all terms involving the variable x on one side of the equation
and the constant terms (the terms that are free of x) on the other side.

5x + 2 = 2x + 17 5x + 2 + (2x) = 2x + 17 + (2x) (adding 2x both sides)

3x + 2 = 17

3x + 2 + (2) = 17 + (2) (adding 2 both sides)

3x = 15

1 1 1
3x = 15 (multiplying by both sides)
3 3 3

x = 5.

Therefore x = 5 is the solution of the equation 5x + 2 = 2x + 17.

44
4. Solve for x, if 5(x 4) x + 23 = 5(x 5) x.

Soln:

5(x 4) x + 23 = 5(x 5) x 5x + 20 x + 23 = 5x 25 x
6x + 43 = 4x 25
6x 4x = 25 43
10x = 63
63
x= .
10

5. Solve for x, if 2(3x 2) + 4(x 1) = 5(2x 3) + 7.

Soln:

2(3x 2) + 4(x 1) = 5(2x 3) + 7 6x 4 + 4x 4 = 10x 15 + 7


10x 8 = 10x 8
10x 10x = 8 + 8
0x = 0.

The last equation is true for all values of x.

6. Solve for x, if 5x 7 x = 3(x 3) + x + 5.

Soln:

5x 7 x = 3(x 3) + x + 5 4x 7 = 3x 9 + x + 5
4x 7 = 4x 4
0x = 3.

The last equation says that 0 = 3, which is absurd. Therefore the equation
5x 7 x = 3(x 3) + x + 5 has no solution.
x2 x+1 5
7. Solve for x, if + = .
3 8 6
Soln: The first step is to get rid of the denominators on both sides of the equation. We
do this by multiplying both sides by the least common multiple of the numbers in the
denominators. The least common multiple of 3, 8 and 6 is 24. Therefore
x2 x+1 5 x2 x+1 5
+ = 24 + 24 = 24
3 8 6 3 8 6
8(x 2) + 3(x + 1) = 5 4
11x 13 = 20
11x = 33
x = 3.

45
1 1 1
8. Solve for x, if (x 2) (3x 6) = (x 6) 2.
3 6 2
Soln: The first step is to get rid of the denominators on both sides of the equation. We
do this by multiplying both sides by the least common multiple of the numbers in the
denominators. The least common multiple of 3, 6 and 2 is 6. Therefore
1 1 1 1 1 1
(x 2) (3x 6) = (x 6) 2 6 (x 2) 6 (3x 6) = 6 (x 6) 2 6
3 6 2 3 6 2
2(x 2) (3x 6) = 3(x 6) 12
x + 2 = 3x 30
4x = 32
x = 8.

2(x 6) (x 3)
9. Solve for x, if + = x.
7 2
Soln: The first step is to write the left hand side under the least common denominator. The
least common multiple of 7 and 2 is 14.
2(x 6) (x 3) 2 2(x 6) + 7(x 3)
+ =x = x (putting lhs under common
7 2 14
denominator)

4(x 6) + 7(x 3)
=x
14
4x 24 + 7x 21
=x
14
11x 45
=x (simplifying numerator)
14
11x 45
14 = 14 x (multiply by 14 both sides
14
to get rid of it in the de-
nominator)

11x 45 = 14x

11x 14x = 45

3x = 45

45
x= = 15.
3

2(x 6) (x 3)
Therefore x = 15 is the solution of the equation + = x.
7 2

2 x p + 2x x3
10. Find p, if 3 is the root of + = .
3 4 5

46
Soln: The least common multiple of 3, 4 and 5 is 60. Therefore
2 x p + 2x x3 2x p + 2x x3
+ = 60 + 60 = 60
3 4 5 3 4 5
20(2 x) + 15(p + 2x) = 12(x 3).

Since 3 is the root of the given equation, it must satisfy the last equation. We therefore
have that

20(2 x) + 15(p + 2x) = 12(x 3) 20(2 (3)) + 15(p + 2(3)) = 12((3) 3)


20 5 + 15(p 6) = 12 (6)
100 + 15p 90 = 72
15p = 82
82
p= .
15

11. Solve for x, if a(x 1) b = x a.

Soln:

a(x 1) b = x a ax a b = x a
x(a 1) = b.

b
Case 1: If a 6= 1, then x = is the solution of the given equation.
a1
Case 2: If a = 1 and b = 0, then we get the equation 0x = 0. This is true for any real
number x.

Case 3: If a = 1 and b 6= 0, then the equation 0x = b has no solution.

x+a b xb a
12. Solve for x, if = + .
b a a b
Soln: Note that a 6= 0 and b 6= 0. The least common multiple of the denominators a and b is
ab. Therefore
x+a b xb a x+a b xb a
= + ab ab = ab + ab
b a a b b a a b
2 2
a(x + a) b = b(x b) + a
ax + a2 b2 = bx b2 + a2
ax = bx
(a b)x = 0.

Case 1: If a = b, then the above equation becomes 0x = 0. This is true for any real
number x.

47
0
Case 2: If a 6= b, then x = = 0 is the solution of the given equation.
ab

Exercise 4.3. 1. Solve the following equations.


1 1 3 5x
(a) (5 3x) + (3 5x) =
4 3 2 3
6x + 2 2x + 2 2x + 16
(b) + =
7 3 5
2x + 3 x + 24
(c) =0
6 12
2 1
2. Show that 4 is the root of (x 1) (3x + 2) = 3 2x.
3 2
6x + 2 2x + 2 2x + 16
3. Show that 2 is the root of + = .
7 3 5
1 1 1
4. Check whether 4 is the root of (9 2x) = 1 (7x 18) or not.
2 2 10
5. Solve for x (consider all cases)

(a) ax + bx = p + x
(b) ax bx = a2 b2
a bx cx b
(c) +x=
c c
ax b2 a(b x) b2
(d) + =a
a b a
(e) (x a)(x b) = x2 a2

4.1.1 Simultaneous linear equations

In this section we discuss two methods of solving two linear equations in two unknowns. That
is, we solve equations of the form

a 1 x + b 1 y = c1
(4.1)

a 2 x + b 2 y = c2

A solution of the system (4.1) is a pair of numbers (x, y) that satify both equation in (4.1).
We shall use examples to illustrate the substitution and elimination methods for solving
simultaneous linear equations.

Examples 4.4. 1. Solve the system of linear equations:


x y = 2
3x y = 2.

48
Soln: Using the substitution method: Let us label these equations:

x y = 2 . . . . . . (1)
3x y = 2 . . . . . . (2)

From (1), we have that

x= y2 . . . . . . (3)

Substituting (3) in (2), we get

3x y = 2 3(y 2) y = 2
2y 6 = 2 2y = 8
y=4 . . . . . . (4)

Substitute (4) in (3) to solve for x:

x = y 2 x = 4 2 = 2.

That is, x = 2 and y = 4. So the solution of the system is the ordered pair (2, 4).

Using the elimination method: In this method we multiply one or both equations by
appropriate numbers to make the coefficients of either x or y the same and then
subtract.
Let us label these equations:

x y = 2 . . . . . . (1)
3x y = 2 . . . . . . (2)

Note that the coefficients of the y term are the same. We will therefore eliminate y by
subtracting equation (1) from equation (2) to get

2x = 4 x = 2 . . . . . . (3)

Now substitute (3) in (1) or (2) to solve for y. Substituting (3) in (1), we have that

x y = 2 2 y = 2 y = 4.

Therefore the solution of the given system of linear equations is (2, 4). 

2. Solve the system of linear equations:

3x + 4y = 18
2x + 3y = 13.

49
Soln: Using the substitution method: Consider the system

3x + 4y = 18 . . . . . . (1)
2x + 3y = 13 . . . . . . (2)

Using (2) to solve for x, we get


13 3y
x= . . . . . . (3)
2
Substitute (3) in (1) to get
 
13 3y
3x + 4y = 18 3 + 4y = 18
2
3(13 3y) + 8y = 36
39 9y + 8y = 36
y = 3.

Substitute y = 3 in (3) to solve for x:


13 3y 13 3 3 13 9
x= x = x = = 2.
2 2 2
That is, x = 2. So the solution of the given system of linear equations is (2, 3). 

Using the elimination method: Consider the system

3x + 4y = 18 . . . . . . (1)
2x + 3y = 13 . . . . . . (2)

Let us eliminate y. To do this, we must multiply (1) by 3 and (2) by 4:

9x + 12y = 54 . . . . . . (3)
8x + 12y = 52 . . . . . . (4)

Subtracting equation (4) from equation (3), we have that x = 2.

To get he value of y, substitute x = 2 in equation (1):

3x + 4y = 18 3 2 + 4y = 18 4y = 12 y = 3.

It now follows that the solution of the given system of linear equations is (2, 3). 

3. Solve the system of linear equations:

x + y = 1
xy = 1.

50
Soln: Consider the system

x + y = 1 . . . . . . (1)
xy = 1. . . . . . . (2)

If we multiply equation (1) by 1 we obtain x y = 1, which is equation (2). Therefore


both equations (1) and (2) represent the same equation: x y = 1.
From this equation, we have that y = x 1. There are infinitely many pairs of numbers
(x, y) which satisfy the equation y = x 1. Therefore this system has infinitely
many solutions. 

4. Solve the system of linear equations:

2x + 5y = 8
4x + 10y = 18.

Soln: Consider the system

2x + 5y = 8 . . . . . . (1)
4x + 10y = 18. . . . . . . (2)

Multiply equation (1) by 2:

4x + 10y = 16 . . . . . . (3)
4x + 10y = 18. . . . . . . (4)

Equation (4) minus equation (3):

0x + 0y = 2 0 = 2,

which is absurd. It now follows that the given system of linear equations has no
solution. 

5. Solve the system


x+y y
+ = 2
5 5
2x y 3x 3
= .
3 4 2
Soln: Consider the system
x+y y
+ = 2
5 5
2x y 3x 3
= .
3 4 2

51
We need to get rid of the denominators. The above system is equivalent to the system
x + y + y = 10

4(2x y) 9x = 18,
which leads to the system
x + 2y = 10 . . . . . . (1)

x 4y = 18. . . . . . . (2)
If we add (1) and (2), we can eliminate x:

2y = 8 y = 4.

Substitute y = 4 in (1):

x + 2y = 10 x + 2(4) = 10 x = 10 + 8 = 2;

That is, x = 2. The required solution of the given system of linear equations is
(2, 4). 

6. Solve the system


18x 21y = 2
24x 15y = 7.
Soln: Consider the system
18x 21y = 2 . . . . . . (1)
24x 15y = 7. . . . . . . (2)
We shall solve this system by first eliminating x. We must find the least common
multiple of 18 and 24. Now, since LCM(18, 24) = 72, we must multiply equation (1)
by 4 and equation (2) by 3 to get
72x 84y = 8 . . . . . . (3)
72x 45y = 21. . . . . . . (4)
Equation (4) minus equation (3):
1
39y = 13 3y = 1 y = .
3
1
To find the value of x, substitute y = in equation (1):
3
1 1
18x 21y = 2 18x 21 = 2 18x = 9 x = .
3 2
 
1 1
So the solution of the system is , . 
2 3

52
4.1.2 Applications: Problems leading to systems of linear equa-
tions

In this subsection we give some examples of how systems linear equations arise in solving word
problems.

Examples 4.5. 1. The admission fee in a certain show is R5 for children and R12 for
adults. On a certain day, 120 people attended the show and R1363 was collected. How
many children and how many adults attended the show on this day?

Soln: Let x be the number of children and y the number of adults that attended the show.
From the given information, x + y = 120 and 5x + 12y = 1363.
We now have two linear equations in two unknowns:
x + y = 120 . . . . . . (1)
5x + 12y = 1363. . . . . . . (2)

Eliminate the variable x by multiplying equation (1) by 5 and subtrating the resulting
equation from equation (2). We get

7y = 763 y = 109.

From equation (1) we have that

x = 120 y = 120 109 = 11.

Therefore there are 109 adults and 11 children who attended the show. 

2. How many litres of 20% alcohol solution and 50% alcohol solution must be mixed to
get 90` of 30% alcohol solution?

Soln: Let x be the number litres of the 20% alcohol solution and y the number litres of the
50% alcohol solution. Then
x + y = 90.
We need to set up another equations which relates the amount of alcohol in the different
solutions:

Amount of alcohol + Amount of alcohol = Amount of alcohol


in the 20% solution in the 50% solution in the 30% solution

0.2x + 0.5y = 0.3 90.

We now have two linear equations in two unknowns:


x+y = 90 . . . . . . (1)
0.2x + 0.5y = 0.3 90. . . . . . . (2)

From equation (1), we have that y = 90 x. Substitute this in (2):

0.2x + 0.5y = 27 0.2x + 0.5(90 x) = 27 0.3x + 45 = 27 x = 60.

53
Therefore y = 90 x = 90 60 = 30. That is, we need 60 litres of the 20% alcohol
solution and 30 litres of the 50% alcohol solution to make 90 litres of a 30% alcohol
solution. 

3. One large container and two small containers can together hold 20 litres of water. The
difference in volume between the large container and the small container is 8 litres. How
many litres of water can each container hold?

Soln: Let x be the volume of the large container and y the volume of the small container.
Then, from the given information, x + 2y = 20 and x y = 8. We therefore have two
linear equations in two unknowns:

x + 2y = 20 . . . . . . (1)
x y = 8. . . . . . . (2)
Equation (2) minus equation (1) gives 3y = 12 and so y = 4. From equation (2), we
have that
x = 8 + y = 8 + 4 = 12.

Therefore, the large container can hold 12 litres of water while the small one can hold
4 litres. 
Exercise 4.6. Solve the following systems of equations by using any method:

x+y = 2
1.
x y = 0.


3x + 5y = 8
2.
2x 3y = 1.


x y = 2
3.
3x y = 2.


6x + 7y = 32
4.
3x 2y = 5.



xy 1 xy
3 2 =

4
5.

x+y 9 y1

= +
2 2 3


3x 2y

+ 4 = 3x
3
6.

2.5x 2y

2x = 3
2

54
x+y

2x 3y 1 = +1
2
7.

3 7
x y = 1
4 4


1 x 1 11

2 (y + ) (x + 2) =
2 5 10
8.

1 1

x 2y + 1 = [2x + 3(y )]
4 2

4.2 Quadratic Equations

Definition 4.7. A quadratic equation is an equation of the form

ax2 + bx + c = 0

in which a, b, and c are real numbers and a 6= 0.

For example
2x2 3x + 1 = 0, and 6x2 + 4x = 0
are quadratic equations.

4.2.1 Solving Quadratic Equations

To solve a quadratic equation ax2 + bx + c = 0 is to find all values of x for which the
quadratic equation is true.

The values of x which solve the quadratic equation ax2 +bx+c = 0 are called the roots of the
quadratic equation ax2+bx+c = 0 or the zeroes of the quadratic function f(x) = ax2 +bx+c.

There are three methods that are generally applied to solve quadratic equations: (1) Factoring
Method, (2) Completing the Square Method, and (3) Using the Quadratic Formula.

(1) Factoring Method

The Factoring Method for solving quadratic equations is based on the simple fact that a
product of two real numbers is zero if and only if one of the numbers is zero. That is, if a and
b are real numbers, then

ab = 0 if and only if a = 0 or b = 0.

55
Examples 4.8. 1. Solve the equation x2 2x 15 = 0.

Soln: Factorise the left hand side:

x2 2x 15 = 0 (x 5)(x + 3) = 0
x 5 = 0 or x + 3 = 0
x = 5 or x = 3.

2. Solve the equation 2x2 x 1 = 0.

Soln: Factorise the left hand side:

2x2 x 1 = 0 (2x + 1)(x 1) = 0


2x + 1 = 0 or x 1 = 0
1
x = or x = 1.
2

3. Solve the equation 2x2 7x 4 = 0.

Soln: Factorise the left hand side:

2x2 7x 4 = 0 (2x + 1)(x 4) = 0


2x + 1 = 0 or x 4 = 0
1
x = or x = 4.
2

4. Solve the equation x(x 1) = 6.

Soln:

x(x 1) = 6 x2 x 6 = 0
(x 3)(x + 2) = 0
x = 3 or x = 2.

5. Solve the equation 3x2 + 5x = 0.

Soln: Factorise the left hand side:

3x2 + 5x = 0 x(3x + 5) = 0
x = 0 or 3x + 5 = 0
5
x = 0 or x = .
3

6. Solve the equation 2 5x 12x2 = 0.

56
Soln: First note that 2 5x 12x2 = 0 if and only if 12x2 + 5x 2 = 0. Now, the last
equation looks familiar. Factorise the left hand side.

12x2 + 5x 2 = 0 (4x 1)(3x + 2) = 0


4x + 1 = 0 or 3x + 2 = 0
1 2
x = or x = .
4 3

(2) Completing the Square Method

The roots of the quadratic equation ax2 + bx + c = 0, a 6= 0, can be found by completing


the square.

ax2 + bx + c = 0 ax2 + bx = c
b c
x2 + x =
a a
 2  2
2 b 1 b c 1 b
x + x+ = +
a 2 a a 2 a
 2
b b2 4ac
x+ = .
2a 4a2

Let = b2 4ac. Now we consider three cases:

Case 1: Assume that > 0. Then,


 2
2 b b2 4ac
ax + bx + c = 0 x+ =
2a 4a2

b b2 4ac
x+ =
2a 2a
b b2 4ac
x=
2a 2a
2
b b 4ac
x=
2a
b b2 4ac b + b2 4ac
x= or x = .
2a 2a
In this case, the quadratic equation ax2 +bx+c = 0 has two real roots that are unequal.
If is a perfect square and the coefficients are rational, then the roots of ax2 +bx+c = 0
are rational.
If is not a perfect square, then the roots of ax2 + bx + c = 0 are irrational.
The formula
b b2 4ac
x=
2a
which gives roots of the quadratic equation ax2 + bx + c = 0 is called the Quadratic
Formula.

57
Case 2: Assume that = 0. Then,
 2
2 b
ax + bx + c = 0 x+ =0
2a
b
x+ =0
2a
b
x= .
2a
In this case, the quadratic equation ax2 + bx + c = 0, with rational coefficients, has two
b
rational roots that are equal and each is equal to .
2a
Case 3: Assume that < 0. In this case, the roots of the quadratic equation ax2 + bx + c = 0
are nonreal.

Examples 4.9. 1. Find the roots of the equation 2x2 7x + 3 = 0 by completing the
square.

Soln:
7 3
2x2 7x + 3 = 0 x2 x =
2 2
 2  2
2 7 7 7 3
x 2 x+ =
4 4 4 2
 2  2
7 7 3
x =
4 4 2
 2
7 49 24
x =
4 16
 2
7 25
x =
4 16
7 5 7 5
x = or x =
4 4 4 4
1
x = 3 or x = .
2
2. Find the roots of the equation x2 + 4x + 2 = 0 by completing the square.

Soln:

x2 + 4x + 2 = 0 x2 + 4x = 2
 2  2
2 1 1
x + 4x + 4 = 2 + 4
2 2
 2
4
x+ =2
2
(x + 2)2 = 2

x+2= 2

x = 2 2 or x = 2 + 2.

58
3. Find the roots of the equation 3x2 + 5x + 6 = 0 by completing the square.

Soln:

3x2 + 5x + 6 = 0 3x2 + 5x = 6
5
x2 x = 2
3
  2   2
2 5 1 5 1 5
x x+ =2+
3 2 3 2 3
 2  2
5 5 5
x2 x + =2+
3 6 6
 2
5 25 97
x = 2+ =
6 36 36
r
5 97 97
x = =
6 36 6

5 97
x=
6 6
5 97 5 97
x= + or x = .
6 6 6 6

4. Solve the equation 4x2 + 4x + 1 = 0.

Soln:
1
4x2 + 4x + 1 = 0 x2 + x + = 0
4
 2
1
x+ =0
2
1
x= (double root).
2

5. Solve the equation 2x2 x + 1 = 0.

Soln:
1 1
2x2 x + 1 = 0 x2 x =
2 2
 2  2
2 1 1 1 1
x 2 x+ =
4 4 4 2
 2
1 7
x = no real roots.
4 16

6. Analyse the quadratic equation 2x2 px 3p2 = 0, where p is a real number.

59
Soln:
p 3p2
2x2 px 3p2 = 0 x2 x =
2 2

p  p 2  p 2 3p2
x2 2 x+ = +
4 4 4 2
 p 2 p2 24p2
x = +
4 16 16
 p 2 25p2
x = .
4 16

25p2
Case 1: If p 6= 0, then > 0. Therefore
16
p 5p p 5p 3p
x = or x = x = or x = p.
4 4 4 4 2
p
Case 2: If p = 0, then x = 0 and so x = 0 (double root).
4

7. Solve the equation 2x2 7x 15 = 0 using the quadratic formula.

Soln: Here a = 2, b = 7, and c = 15. Since the discriminant is

= b2 4ac = 49 + 120 = 169 > 0,

the equation has two distinct and real roots. By the quadratic formula,

b b2 4ac
x =
2a p
(7) (7)2 4(2)(15)
=
2(2)

7 49 + 120
=
4
7 169 7 13
= = .
4 4
Therefore
7 13 6 3 7 + 13 20
x= = = or x = = = 5.
4 4 2 4 4
8. Solve the quadratic equation 2x2 + 3x + 1 = 0.

Soln: Here a = 2, b = 3 and c = 1. Since the discriminant

= b2 4ac = 9 8 = 1 > 0,

60
the equation has two distinct and real roots. Using the quadratic formula,

b b2 4ac
x =
p2a
3 (3)2 4(2)
=
2(2)

3 9 8
=
4

3 1 3 1
= = .
4 4
Therefore
3 1 4 3 + 1 2 1
x= = = 1 or x= = = .
4 4 4 4 2

9. Solve the quadratic equation 2x2 + 5x 4 = 0.

Soln: Here a = 2, b = 5 and c = 4. Since the discriminant

= b2 4ac = 25 + 32 = 57 > 0,

the equation has two distinct and real roots. Using the quadratic formula,

b b2 4ac
x =
p2a
5 (5)2 4(2)(4)
=
2(2)

5 25 + 32
=
4
5 57
= .
4
Therefore
5 57 5 + 57
x= or x= .
4 4
10. Solve the equation 3x2 + x + 1 = 0 using the quadratic formula.

Soln: Here a = 3, b = 1, and c = 1. Since the discriminant is

= b2 4ac = 12 4(3)(1) = 1 12 = 11 < 0,

the equaton has no real roots.

11. Solve the equation 3x2 + 2x + 7 = 0 using the quadratic formula.

Soln: Here a = 3, b = 2, and c = 7. Since the discriminant is

= b2 4ac = 22 4(3)(7) = 80 < 0,

the equation has no real roots.

61
12. Find the values of p for which the quadratic equation px2 + 2px 2 = 0 has

(i) distinct real roots (ii) real repeated root, (iii) non-real roots.

Soln: Here a = p, b = 2p, and c = 2. The discriminant is

= b2 4ac = (2p)2 4p(2) = 4p2 + 8p = 4(p2 + 2p).

(i) The equation px2 + 2px 2 = 0 has distinct real roots if and only if the discriminant
is positive. That is, if and only if

>0 4(p2 + 2p) > 0 p2 + 2p > 0 p(p + 2) > 0


p < 2 or p > 0.

In this case, the roots are given by



b b2 4ac
x =
2a
p
2p 4(p2 + 2p)
=
2p
p
2p 2 p2 + 2p
=
2p
p
p p2 + 2p
= .
p
Therefore p p
p p2 + 2p p + p2 + 2p
x= or x= .
p p

(ii) The equation px2 +2px2 = 0 has a real repeated root if and only if the discriminant
is zero. That is,

=0 4(p2 + 2p) = 0 p2 + 2p = 0 p(p + 2) = 0


p=0 or p = 2.

But p 6= 0. (WHY?) Therefore p = 2. In this case, the real repeated root is


2p
x= = 1.
2p

(iii) The equation px2 + 2px 2 = 0 has non-real roots if the discriminant is negative.
That is,

<0 4(p2 + 2p) < 0 p2 + 2p < 0 p(p + 2) < 0


2 < p < 0.

62
Exercise 4.10. 1. Solve the following equations by Completing the Square:

(a) 4x2 7x + 1 = 0
(b) 2x2 x 3 = 0
(c) 10x2 x + 2 = 0
(d) 6x2 px 2p2 = 0 (Consider all cases)
(e) 2x(x + e) = 3e(x + e) (Consider all cases)

2. Solve the following equations by using the Quadratic Formula:

(a) 2x2 7x 4 = 0
(b) 2x2 + 8x + 3 = 0
(c) x2 6x + 2 = 0
(d) 4x2 x + 2 = 0
(e) 2px(x + 2q) = pqx 2 (Consider all cases.)
(f) x2 + (2k 3)x + 2k + 5 = 0 (Consider all cases.)

Nature of the Roots of Quadratic Equation

Consider the quadratic equation


ax2 + bx + c = 0,
with rational coefficients. We have noted above the importance of the expression = b2 4ac
as the key to determining the nature of the roots of a given quadratic equation. is called
the discriminant of the quadratic equation ax2 + bx + c = 0.

(i) If > 0 and the square of a rational number, then the roots of the quadratic equaton
are unequal and rational (real).

(ii) If > 0 and not the square of a rational number, then the roots are unequal,
irrational (real).

(iii) If = 0, then the roots are equal, rational (double root).

(iv) If < 0, then the roots are non-real.

Examples 4.11. 1. Determine of each of the following equations and discuss the na-
ture of their roots.

(a) 6x2 7x 3 = 0.
Soln: = 49 + 72 = 121 > 0.

63
Since is a perfect square and coefficients are rational, the roots of the equation
6x2 7x 3 = 0 are unequal and rational.
(b) 2x2 3x 1 = 0.
Soln: = 9 + 8 = 17 > 0.
Since is not a perfect square, the roots are unequal and irrational.
(c) 4x2 + 4x + 1 = 0.
Soln: = 16 16 = 0.
Roots are equal and rational.
(d) 2x2 3x + 2 = 0.
Soln: = 9 16 = 7 < 0, the equation has nonreal roots.

2. For the quadratic equation x2 + 2k(x + 1) = 1 2x,

(a) show that its roots are rational for all rational values of k.
(b) determine k if roots are real and equal (double root).

Soln: (a)
x2 + 2kx + 2k + 2x + 1 = 0 x2 + x(2k + 2) + 2k + 1 = 0.
The discriminant is

= (2k + 2)2 4(2k + 1) = 4k 2 + 8k + 4 8k 4 = 4k 2 = (2k)2 .

Since is a perfect square and all coefficents are rational, the roots of the quadratic
equation x2 + 2k(x + 1) = 1 2x are rational.
(b) For equal roots, we need = 0. Therefore 4k 2 = 0 and so k = 0. With k = 0,
the given quadratic equation becomes

x2 + 2x + 1 = 0 (x + 1)2 = 0 x = 1.

3. Find the values of the natural number k for which the roots of 2x2 6x + k = 0 are
rational.

Soln: The discriminant is = 36 8k. We need to find the value(s) of k for which is a
perfect square. If k = 4, then = 35 32 = 4 a perfect square.

4. Show that the roots of


k 2 x2 + kx + 2x2 + 2x + 1 = 0
are non-real for all real values of k.

Soln: Note first that

k 2 x2 + kx + 2x2 + 2x + 1 = 0 x2(k 2 + 2) + x(k + 2) + 1 = 0.

64
Now, the discriminant of this quadratic equation is

= (k + 2)2 4(k 2 + 2) = k 2 + 4k + 4 4k 2 8 = 3k 2 + 4k 4.

Now, completing the square, we have


  "  2  2 #
4 4 2 2 2 4
3k 2 + 4k 4 = 3 k 2 k + = 3 k 2 2 k + +
3 3 3 3 3 3
" 2 #
2 4 4
= 3 k +
3 9 3
" 2 #
2 8
= 3 k + .
3 9

Since the expression in the square bracket is always positive, it follows that
" 2 #
2 8
= 3k 2 + 4k 4 = 3 k + <0
3 9

for all values of k R. Therefore the roots of the equation k 2 x2 + kx + 2x2 + 2x + 1 = 0


are non-real.

5. Determine the values of p for which 3x2 px + 3 = 0 has real roots.

Soln: The equation has real roots if and only if its discriminant is nonnegative. That is,

0 p2 36 0 (p 6)(p + 6) 0

p 6 0 and p + 6 0 OR p 6 0 and p + 6 0

p 6 and p 6 OR p 6 and p 6.

Therefore p 6 or p 6. 

6. If the equation x2 + 2kx + 2x = 2x 9k has equal roots, find k.

Soln: Note that

x2 + 2kx + 2x = 2x 9k x2 + x(2k + 4) + 9k = 0.

The discriminant for this equation is

= (2k + 4)2 36k = 4k 2 + 16k + 16 36k = 4k 2 20k + 16.

65
For the given quadratic equation to have equal roots, its discriminant must be zero.
Therefore

=0 4k 2 20k + 16 = 0 k 2 5k + 4 = 0 (k 1)(k 4) = 0
k = 1 or k = 4.

If k = 1, the given quadratic equation becomes

x2 + 6x + 9 = 0 (x + 3)2 = 0 x = 3.

That is, x = 3 is a repeated root.


If k = 4, the given quadratic equation becomes

x2 + 12x + 36 = 0 (x + 6)2 = 0 x = 6.

That is, x = 6 is a repeated root. 

7. Show that the roots of the equation 3x2 k + 2kx 2x = 0, with k R, can never be
equal.

Soln: Note that


3x2 k + 2kx 2x = 0 3x2 + x(2k 2) k = 0.
The discriminant for this equation is

= (2k 2)2 + 12k = 4k 2 8k + 4 + 12k = 4k 2 + 4k + 4 = 4(k 2 + k + 1)


" 2 #
1 3
= 4 x+ + > 0. [obtained by completing the square].
2 4

That is, the discriminant of the given equatoin is always positive. Therefore 6= 0 and
so the given equation can never have equal roots. 

Exercise 4.12. 1. Discuss the nature of the roots of the following equations (do not solve
the equations):

(a) 3x2 x + 4 = 0
(b) x2 + 7x + 1 = 0
(c) 2x2 3x + 1 = 0
(d) 9x2 + 6x + 1 = 0
(e) rx2 = r x ; r 6= 0
(f) 6x2 7px 5p2 = 0 ; p N

66
2. For which value(s) of r will x2 5x = r have real roots?

3. If the roots of the equation rx2 + 2rx + 3 = 0 are real and equal, find r.

4. What is the nature of the roots of the following equations?



(a) x2 2 2x + 2 = 0

(b) 3x2 + 4x + 2 = 0

(c) 2x2 + 8x 18 = 0

5. Ax2 + Bx + C = 0 ; B = 0; C 0; A < 0. Discuss the nature of the roots.

6. Show that the roots of kx2 + kx k = 3x x2 are real for all real values of k.

4.3 Equations leading to Linear or Quadratic Equa-


tions : Rational and Irrational Equations

In this section we solve rational/irrational equations that lead to linear or quadratic equations.
When dealing with such equations it is always important to state the appropriate restrictions
for which the expression is defined.
x3 5 4
Examples 4.13. 1. Solve for x, if 2 = .
x2 + 3x + 2 x 4 x+1
Soln: Note that
x3 5 4 x3 5 4
2 = = .
x2 + 3x + 2 x 4 x+1 (x + 1)(x + 2) (x 2)(x + 2) x+1

It is clear that x 6= 1, x 6= 2 and x 6= 2. Now,


x3 5 4 x3 5 4
2 = =
x2 + 3x + 2 x 4 x+1 (x + 1)(x + 2) (x 2)(x + 2) x+1
(x 3)(x 2)(x 2)(x + 2) 5(x + 1)(x 2)(x + 2) 4(x + 1)(x 2)(x + 2)
=
(x + 1)(x + 2) (x 2)(x + 2) x+1

(x 3)(x 2) 5(x + 1) = 4(x 2)(x + 2)


x2 5x + 6 5x 5 = 4x2 + 16
5x2 10x 15 = 0
5(x2 2x 3) = 0
x2 2x 3) = 0
(x + 1)(x 3) = 0
x = 1 or x = 3.

Since x 6= 1, we must have that x = 3. 

67
16 4 5(x + 2)
2. Solve for x, if + + = 0.
x2 9 x2 5x + 6 x2 + x 6
Soln: Note that, since

16 4 5(x + 2)
+ 2 + 2 =0
x2 9 x 5x + 6 x + x 6
16 4 5(x + 2)
+ + = 0,
(x + 3)(x + 3) (x 2)(x 3) (x + 3)(x 2)

we must have that x 6= 3 x 6= 2.


Now multiply both sides of the equations by (x 3)(x + 3)(x 2).

16 4 5(x + 2)
+ 2 + 2 =0
x2 9 x 5x + 6 x + x 6
16 4 5(x + 2)
+ + =0
(x + 3)(x + 3) (x 2)(x 3) (x + 3)(x 2)
16(x 3)(x + 3)(x 2) 4(x 3)(x + 3)(x 2) 5(x + 2)(x 3)(x + 3)(x 2)
+ + =0
(x + 3)(x + 3) (x 2)(x 3) (x + 3)(x 2)

16(x 2) + 4(x + 3) + 5(x + 2)(x 3) = 0

16x 32 + 4x + 12 + 5x2 5x 30 = 0

5x2 + 15x 50 = 0 5(x2 + 3x 10) = 0

x2 + 3x 10 = 0 (x + 5)(x 2) = 0

x = 5 or x = 2.

Since, as noted above, x 6= 2, it follows that x = 5. 


4 1 2x 1
3. Solve for x, if = 2 .
x+2 x x + 2x
Soln: Note that, since
4 1 2x 1 4 1 2x 1
= 2 = ,
x+2 x x + 2x x+2 x x(x + 2)

68
x 6= 2 and x 6= 0. Now,
4 1 2x 1 4 1 2x 1
= 2 2 =0
x+2 x x + 2x x + 2 x x + 2x
4 1 2x 1
=0
x + 2 x x(x + 2)
4x (x + 2) (2x 1)
=0
x(x + 2)
4x x 2 2x + 1
=0
x(x + 2)
x1
=0
x(x + 2)
x1=0
x = 1.

4 1 2x 1
Therefore x = 1 is the solution of the equation = 2 .
x+2 x x + 2x
x+6 1 2
4. Solve for x, if + = .
x2 4 x + 2 x2
Soln: Since
x+6 1 2 x+6 1 2
+ = + = ,
x2 4 x + 2 x2 (x 2)(x + 2) x + 2 x2
we must have that x 6= 2 and x 6= 2.
Multiply both sides by (x 2)(x + 2):
x+6 1 2
2
+ =
x 4 x+2 x2
x+6 1 2
+ =
(x 2)(x + 2) x + 2 x2
(x + 6)(x 2)(x + 2) 1(x 2)(x + 2) 2(x 2)(x + 2)
+ =
(x 2)(x + 2) x+2 x2

x + 6 + x 2 = 2(x + 2) 2x + 4 = 2x + 4
0x = 0.

The last equation is true for all real numbers x. Because of our original restriction that
x 6= 2, it follows that the solution of the original equation consists of all real numbers
except 2 and 2. 
3 4x 1 x2 + 5
5. Solve for x, if + = 5.
x1 x+1 (x 1)(x + 1)

69
Soln: Note that x 6= 1 and x 6= 1. Multiply both sides of the equation by (x 1)(x + 1):
3 4x 1 x2 + 5
+ = 5
x1 x+1 (x 1)(x + 1)

3(x 1)(x + 1) (4x 1)(x 1)(x + 1) (x2 + 5)(x 1)(x + 1)


+ = 5(x 1)(x + 1)
x1 x+1 (x 1)(x + 1)

3(x + 1) (4x 1)(x 1) = x2 + 5 5x2 + 5

3x + 3 4x2 + 5x 1 = 4x2 + 10
8x = 8 x = 1.
But since x 6= 1, the given equation has no solutoin. 
x 2a 2x2 13a2
6. Solve for x, if =3 2 .
x 3a x 9a2
Soln: Noting that
x 2a 2x2 13a2 x 2a 2x2 13a2
=3 2 x = 3 ,
x 3a x 9a2 x + 3a (x 3a)(x + 3a)
we must have that x 6= 3a. Now, multiplying by (x 3a)(x + 3a):

x 2a 2x2 13a2
=3 2 x
x 3a x 9a2

x 2a 2x2 13a2
=3
x + 3a (x 3a)(x + 3a)

(x 2a)(x 3a)(x + 3a) (2x2 13a2 )(x 3a)(x + 3a)


= 3(x 3a)(x + 3a)
x + 3a (x 3a)(x + 3a)

(x 2a)(x 3a) = 3(x2 9a2 ) (2x2 13a2 )


x2 5ax + 6a2 = 3x2 27a2 2x2 + 13a2
5ax = 20a2 .

If a 6= 0, then x = 4a is the solution of the given equation.

If a = 0, then 0x = 0. This is true of any real number x. In this case, the set of
solutions of the given equation consists of all real numbers except 3a and 3a. 

4.3.1 Irrational Equations

In this subsection we focus on solving irrational equations. An irrational equation is an


equation in which the variable is inside a radical. These are equations like

px + q = r and ax + b px + q = c.

70
For simplicity, we shal confine our discussion to square roots only.

NOTE: If a2 = b2 , then a = b or a = b. Most students make a mistake of saying that


if a2 = b2, then a = b. This is NOT true! That is, a2 = b2 6 a = b. However, if we
know that a 0 and b 0, then we can conclude that a2 = b2 = a = b. It is for this

reason that when solving an irrational equation such as px + q = r, we must imposing the
restrictions that px + q 0 and r 0.

Method of solving irrational equations involving square roots:

1. If there is only one radical in the given equation, rewrite the equation so that the radical
is on one side of the equation and all the other terms on the other side. Then square both
sides to get rid of the radical. Solve the resulting equation. Note that the equation that
resulted from the squaring process is not equivalent to the original equation. Therefore
the solutions of the new equation (arising from the squaring process) are not necessarily
solutions of the original equations. All solutions must be checked in the original equation.

2. If there are two radicals, separate them so that one appears on one side of the equation
and the other on the other side together with the other terms. Square both sides of the
equation to get rid of the radical on the one side of the equation. The other part of
the equation may still have a radical. Proceed as in (1) above to handle the remaining
radical.


Examples 4.14. 1. Solve for x, if 5x + 6 = x.
6
Soln: Note that 5x + 6 0 x (for the square root to be defined). Also, the
5
right hand side must be nonnegative as we are taking the nonnegative square root on
the left hand side. That is, x 0. From these two observations, we have that x 0.

5x + 6 = x = ( 5x + 6)2 = x2
= 5x + 6 = x2 x2 5x 6 = 0
= (x + 1(x 6)) = 0
= x = 1 or x = 6.

From the note in the beginning, we must have that x = 6 is the only solution of the
given equation.
p
Check: If x = 1, then LHS = 5(1) + 6 = 1 = 1 and RHS = 1. Since the left hand
side and the right hand side are not the same, x = 1 is not a solution of the given
equation. p
If x = 6, then LHS = 5(6) + 6 = 36 = 6 and RHS = 6. Therefore x = 6 is the
only solution of the given equation. 

2. Solve for x, if x + 6 x = 4.

71

Soln: Rewrite the equation as x + 6 = x + 4. Note that x + 6 0 x 6 (for the
square root to be defined). Also, the right hand side must be nonnegative as we are tak-
ing the nonnegative square root on the left hand side. That is, x + 4 0 x 4.
From these two observations, we have that x 4.


x + 6 x = 4 = x+6 =x+4

= ( x + 6)2 = (x + 4)2
= x + 6 = x2 + 8x + 16
= x2 + 7x + 10 = 0 (x + 5)(x + 2) = 0
= x = 5 and x = 2.

Since x 4, it follows that x = 2 is the only solution of the given equation.



Check: If x = 5, then LHS = 5 + 6 (5) = 1 + 5 = 6 6= 4 = RHS. Since the left
hand side and the right hand side are not the same, x = 5 is not a solution of the
given equation.

If x = 2, then LHS = 2 + 6 (2) = 4 + 2 = 4 = RHS. Therefore x = 2 is
the only solution of the given equation. 

3. Solve for x, if 2 x 3 = x 3.

Soln: Note that x 3 0 (for the square root to be defined); i.e., x 3. Also, since the left
hand side is a nonnegative square root, the right hand side should also be nonnegative;
i.e. x 3.
Now,
2
2 x 3 = x 3 = 2 x 3 = (x 3)2
= 4(x 3) = (x 3)2
= x2 10x + 21 = 0 (x 3)(x 7) = 0
= x = 3 or x = 7.

Since both these numbers are greater or equal to 3, they must both be solutions of the
given equation.

Check: If x = 3, then LHS = 2 2 3 = 0 = 3 3 = RHS. Therefore x = 3 is a solution of
the given equation.

If x = 7, then LHS = 2 7 3 = 4 = 7 3 = RHS. Therefore x = 7 is a solution of
the given equation. 

4. Solve for x, if 6 + x 7 x = 1.

72
Soln:

6+x 7x=1 6+x = 7x+1
 2 2
= 6+x = 7x+1

= 6+x= 7x+2 7x+1

= 7x = x1
2
= 7 x = (x 1)2
= 7 x = x2 2x + 1
= x2 x 6 = 0 (x + 2)(x 3) = 0
= x = 2 or x = 3.
p
Check: If x = 2, then LHS = 6 2 7 (2) = 4 9 = 2 3 = 1 6= 1 = RHS.
Therefore x = 2 is not a solution of the given equation.

If x = 3, then LHS = 6 + 3 7 3 = 9 4 = 3 2 = 1 = RHS. Therefore
x = 3 is the only solution of the given equation. 

5. Solve for x, if x + 1 x + 6 = 1.

Soln:

x+1 x+6 =1 x+1 = x+6+1
 2  2
= x+1 = x+6+1

= x+1 = x+6+2 x+6+1

= x + 6 = 3.

The left hand side of the last equation is nonnegative whereas the right hand side is
negative. Therefore the equation has no solution. 

Exercise 4.15. 1. Solve the following rational equations:


x+2 3 1
(a) =
x+1 x2 x+1

6 x+2 x + 3 2x 2
(b) 1 + + = +
x+1 x+1 x 1 1 x2

4 3x + 6 4
(c) = 2 + 2
x2 x x 2 x + 4x + 3

x+1 1x 2
(d) 2
+ =
x 4 x+2 5(x 2)

2 2 1 3
(e) = + 2 + 2
x2 + 3x + 2 1x 2 x + 3x + 2 x + x 2

73
x+3 1 1 7
(f) 2
+ 2 =
x 4 x +x2 2 x 4 x2
2. Solve for x, if
a ab a+b b
+ = .
ac + bc 2bx 2bc ax + bx

3. Solve the following irrational equations:



(a) 9 + 4x = 2x 3

(b) 3x 11 x = 3

(c) 3 x2 1 = 2 (Hint: a3 = b3 a = b.)

(d) x + 2 = 7 x + 9

(e) 3 2x + 3 = 2

(f) x + 3 x 2 = 7 x

(g) 2 x + x 2 = 1

(h) 2x + 8x 3 = 0

(i) 2x + 9 + 3x + 16 = 17
1 2
(j) = 0.
x x + 27

4.4 Applications: Problems leading to linear/quadratic


equations

In this section we look at some examples of word problems that lead to linear/quadratic
equations.
Examples 4.16. 1. Find two integers whose sum is 4 and whose product is 45.

Soln: Let x and y be the two numbers. Then

x+y =4 (1.1)

and
xy = 45 (1.2)
From (1.1) we have that y = 4 x. Substituting this in (1.2), we have that

x(4x) = 45 4xx2 = 45 x2 4x45 = 0 (x9)(x+5) = 0.

Therefore x = 5 or x = 9. If x = 5, then y = 4 (5) = 9 and, if x = 9, then


y = 4 9 = 5. the two numbers are 5 and 9. 

74

2. The sum of a number and its reciprocal is 20. Find the number. (Leave you answer
in surd form).
1
Soln: Let the required number be x . Then x + = 20.
x
Now,
1 x2 + 1
x+ = 20 = 20
x x
x2 + 1 = 20 x

x2 20 x + 1 = 0
q 
2
2 5 20 4
x=
2
2 5 16
x=
2
2 54
x= .
2

2 54 2 5+4
That is, x = = 5 2 or x = = 5 + 2. 
2 2
3. The printed area of a rectangular page is to be 60% of the total area. If the dimensions
of the page are 50cm by 40cm, find the margin, of constant width, to leave around the
printed area.

Soln: Let x be the width of the margin.


6
x
?

Printed
Area
 - Printed
x x-
Area 50cm
Printed
Area
6
x
?
40cm
The printed area is

(40 2x)(50 2x) = 60% of 2000 (40 2x)(50 2x) = 1200


2000 180x + 4x2 = 1200
x2 45x + 200 = 0
(x 40)(x 5) = 0.

Therefore x = 40 or x = 5. But 40 2x > 0 x < 20 and so 0 < x < 20. Therefore


x = 5cm is the only solution. 

75
4. A man paid R40 for paint at x rand per litre. Two weeks later the price of paint was
increased by R1 per litre and he left the shop with 2 litres less paint than the previous
time having paid R40. Determine x.
40
Soln: The number of litres of paint that the man bought with his R40 is . Two weeks later,
x
40 40
the number of litres of paint that the man bought with his R40 is . Since is
x+1 x+1
2 litres less than the amount of paint that the man had got two weeks earlier, we have
that
40 40
= 2.
x x+1
Now,
40 40 40x(x + 1) 40x(x + 1)
=2 = 2x(x + 1)
x x+1 x x+1
40(x + 1) 40x = 2x(x + 1)
40x + 40 40x = 2x2 + 2x
2x2 + 2x 40 = 0
2(x2 + x 20) = 0
x2 + x 20 = 0
(x + 5)(x 4) = 0
x = 5 or x = 4.

Since x > 0, we must have that x = 4. 

5. A man travels 180km from his farm to Pretoria in a loaded truck. On the return journey,
he is able to travel 30km/h faster having unloaded his produce in Pretoria. He saves 1
hours on the return journey by travelling faster. How fast did he travel to Pretoria?

Soln: Distance =speed time.

Journey Distance Speed Time

180
From farm to Pretoria 180km x
x

180
From Pretoria to farm 180km x + 30
x + 30

Now we can form the time equation in hours:


180 180
= + 1, x > 0.
x x + 30

76
Now,
180 180 180x(x + 30) 180x(x + 30)
= +1 = + x(x + 30)
x x + 30 x x + 30

180(x + 30) = 180x + x(x + 30)


180x + 5400 = 180x + x2 + 30x
x2 + 30x 5400 = 0
(x 60)(x + 90) = 0
x = 60 or x = 90.

We discard the value x = 90 as the speed cannot be negative. Hence x = 60km/h.

6. A tank has two pipes entering it. When operating together, the tank is filled in 40
mintures. Operating independently, one of the pipes fills the tank 60 minutes faster
than the other is able to on its own. Find the time taken by each pipe to fill the tank
on its own.

Soln: Let x be the time (in minutes) taken by slower pipe to fill the tank on its own. Then
1
x 60 is the time taken by the faster pipe to fill the tank on its own. is the fraction
x
1
of the tank filled by the slower pipe in 1 minute and is the fraction of the tank
x 60
1
filled by the faster pipe in 1 minute. is the fraction of the tank filled by both pipes
40
in 1 minute. Now we form an equation.
In 1 minute, we have
1 1 1
+ = .
x x 60 40
Now,
1 1 1 40x(x 60) 40x(x 60) 40x(x 60)
+ = + =
x x 60 40 x x 60 40

40(x 60) + 40x = x(x 60)


40x 2400 + 40x = x2 60x
x2 140x + 2400 = 0
(x 120)(x 20) = 0
x = 120 or x = 20.

We discard the value of x = 20 as this will make x 60 negative. Therefore x = 120


minutes. That is, the slower pipe takes x = 120 minutes to fill the tank on its own while
the faster pipe takes x = 120 60 = 60 minutes to fill the tank on its own. 

Exercise 4.17. 1. A man calculated that if his speed had been increased by 4km/h he
would have completed his 48km journey 1 hour sooner. Determine his speed.

77
2. One water pipe fills a swimming bath in x hours. A second pipe takes 3 hours longer
to fill the bath. The two pipes together fill the bath in two hours. How long will it take
each pipe to fill the swimming bath separately?

3. The lengths of three sides of a right-angled triangle are consecutive integers. Find the
lengths.

4. The length of a rectangle is three times its width. If the length is increased by 2cm
and the width by 1cm, the new rectangle has area 70cm2 . Find the dimensions of the
rectangle.

5. Find two consecutive integers whose product is 56.

6. A man jogs for 10km and then walks for 10km. He jogs 2 12 km/hr faster than he walks.
The entire distance of 20km takes 6 hours. Find his jogging and walking speeds.

7. It takes a woman twice as long to do a job as a man. Working together, they take 40
minutes to do the job. How long would it take them individually to complete the job?

8. A boat can travel 30km/h in still water. The boat takes one hour to travel 12km
upstream and back again. Find the speed at which the stream is flowing. (Leave your
answer in surd form.)

78
Chapter 5

Linear and Quadratic Functions

5.1 Concept of Function

The concept of a function is central in Mathematics. A function is used to represent a


relationship between object.

Definition 5.1. A function from a set A into a set B is a rule that assigns to each
element x in A exactly one element, denoted by f(x), in B. The number f(x) is called
the value of f at x. The set A is called the domain of the function f. We shall denote
by Dom(f) the domain of the function f. The range of f is the set of all possible values
of f(x) as x varies throughout the domain.
If f is a function with domain A, then its graph is the set of ordered pairs

{(x, f(x))|x A}.

'$'$ '$'$
A B A B

XXX -
XX
XX
z

XXX -
XX
XX
z
:

 
 -
&%&% &%&%

In this course we consider functions for which the sets A and B are sets of real numbers.

79
Let f be a function from a set A into a set B. When deciding on the domain of a function
f, always bear in mind that

1. Division by zero is not defined. Any value of the variable x that makes the denominator
cannot be in the domain of f.

2. Square roots of negative numbers are not real numbers. Any value of the variable x
that makes the expression under the square root negative cannot be in the domain of
the function f.

How does one decide whether a given curve in the plane is the graph of a function or not?

Vertical line test: A curve in the plane is the graph of a function if and only if no vertical
line cuts that curve in more than one point.
Put differently: if a vertical line cuts the curve at two or more points, then that curve is not
a graph of a function.

Examples 5.2. 1. The graph of a function f is shown below.

(a) Find the values of f(1) and f(2).


(b) What are the domain and range of f?
y
6

3 . . . . ...
..
..
..
.. .... . . ..
.. 1 ..
.. ..
. . -
3... 2 0 1 4.... x
.. ..
. .
.. . .. . . . . 2

Soln: (a) f(1) = 3 and f(2) = 1.


(b) The domain of f is the interval [3; 4] and the range of f is the interval [2; 3].

2. Find the domain of each of the following functions.


1
(a) f(x) = (d) k(x) = x2 4
(x 2)(x 3)

1 1
(b) g(x) = 2x 1 (e) p(x) = + 2
x x 9
r
x 3x
(c) h(x) = (f) q(x) =
x2 x+1
Soln:

80
(a) The domain of f is the set of all real numbers except 2 and 3. That is,

Dom(f) = {x R | x 6= 2, x 6= 3} = (, 2) (2, 3) (3, +).

(b) The square root function is defined only for nonnegative numbers. Therefore, for
the function g to be defined, we require that 2x 1 0, and so x 21 . Therefore
1
the domain of g is the set of all real numbers that are greater than or equal to ,
2
i.e.,    
1 1
Dom(g) = x R | x = , .
2 2
(c) The square root function is defined only for nonnegative numbers. Therefore, for
the function h to be defined, we require that x 0. But since h is a rational
fraction, we must also require that the denominator is not zero, i.e., x 6= 2.
Therefore the domain of h is

Dom(h) = {x R | x 0 and x 6= 2} = [0, 2) (2, ).

(d) The square root function is defined only for nonnegative numbers. Therefore, for
the function k to be defined, we require that x2 4 0. Now,

x2 4 0 (x 2)(x + 2) 0
x 2 0 and x + 2 0 OR x 2 0 and x + 2 0
x 2 and x 2 OR x 2 and x 2
x 2 OR x 2.

It now follows that the domain of the function k is the set of all real numbers that
are greater than or equal to 2 or less than or equal to 2. That is,

Dom(k) = {x R | x 2 or x 2} = (, 2] [2, ).

(e) The function p is a sum of two rational fractions. For the function p to be defined,
we require that the denominators should not be zero; i.e., x 6= 0 and x2 9 6= 0.
Equivalently, x 6= 0 and x 6= 3. Therefore, the domain of the function p is the
set of all real numbers x except 0, 3 and 3. That is,

Dom(p) = {x R | x 6= 0, x 6= 3, x 6= 3} = (, 3)(3, 0)(0, 3)(3, ).

(f) Note that x 6= 1. (Why?) For the square root function to be defined, we require
3x
that 0. Now,
x+1
3x
0 3 x 0 and x + 1 > 0 OR 3 x 0 and x + 1 < 0
x+1
3 x and x > 1 OR 3 x and x < 1
1 < x 3,

81
since there is no real number that is simultaneously greater than 3 and less than
1. We noted in the beginning of the solution that x 6= 1. Therefore, the
domain of q consists of real numbers that are strictly bigger than 1 and less than
or equal to 3. That is,

Dom(q) = {x R | 1 < x 3} = (1, 3].

3. Determine whether the given curve is the graph of a function of x or not.


y y
6 6

- -
0 x 0 x

(a) (b)

y 6 y 6

,
,
,- -
0 , x 0 x
,
,,
,
,
,

(c) (d)

y 6 y 6






 - -
0 x 0 x

(e) (f)
82
Soln: (a) Yes (b) No (c) Yes (d) No (e) Yes (f) No.

4. Plot the graphs of the following functions f : X Y where X = {2, 1, 0, 1, 2} and


Y = Z.

(a) f(x) = 2x (b) f(x) = x + 1 (c) f(x) = 1 (d) f(x) = x2.

Soln:
y y
6 6
4 4

3 3

2 2

1 1

| | | |- | | | |-
2 1 0 1 2 x 2 1 0 1 2 x
1 1

2 2

3 3

4 4

(a) (b)

y6 y6
2 4

1 3

| | | |- 2
2 1 0 1 2 x
1
1

2 | | | |-
2 1 0 1 2 x
1
(c) (b)

83
5. Let X = {1, 2, 3, 4, 5, 6} and Y = Z. Plot the graph of the following functions
f :X Y

1 if x is even
(a) f(x) =
0 if x is odd.

1 if x is divisible by 3
(b) f(x) =
1 otherwise.
Soln:
y y
6 6
2 2

1 1

| | | | | | |- | | | | | | |-
1 0 1 2 3 4 5 6 x 1 0 1 2 3 4 5 6 x
1 1

2 2

(a) (b)

5.2 Properties of Functions

5.2.1 Increasing and Decreasing Function

Definition 5.3.

1. A function f is said to be increasing on an interval [a, b] if f(x1 ) < f(x2 ) whenever


x1 < x2 in [a, b].

2. A function f is said to be decreasing on an interval [a, b] if f(x1 ) > f(x2 ) whenever


x1 < x2 in [a, b].

3. A function f is said to be constant on an interval [a, b] if f(x1 ) = f(x2 ) whenever


x1 < x2 in [a, b].

Intuitively, a function f is increasing on an interval [a, b] if the graph of f goes up as one


moves from left to right in the interval [a, b]; and decreasing if its graph goes down as one
moves from left to right in the interval [a, b]. A function f is constant on the interval [a, b] if
it remains horizontal as one moves from left to right in the interval [a, b].

84
y y y
6 6 6

- - -
0 x 0 x 0 x

Increasing function Decreasing function Constant function

5.2.2 Even and Odd Functions

Definition 5.4. 1. A function f is said to be even if f(x) = f(x) for every real
number x in its domain.

2. A function f is said to be odd if f(x) = f(x) for every real number x in its
domain.

The graph of an even function is symmetric with respect to the y-axis; the graph of an odd
function is symmetric with respect to the origin.
y y
6 6

- -
0 x 0 x

Graph of an even function is Graph of an odd function is


symmetric about the y-axis symmetric about the origin

Examples 5.5. Determine whether each of the functions is even, odd or neither.

1. f(x) = x2 + x4

2. f(x) = x3 x

85

3. f(x) = 4 x2

4. f(x) = 3 x

5. f(x) = x4 + x.

Soln: 1. Since f(x) = (x)2 + (x)4 = x2 + x4 = f(x), the function f is even.

2. Since f(x) = (x)3 (x) = x3 + x = (x3 x) = f(x), the function f is


odd.
p
3. Since f(x) = 4 (x)2 = 4 x2 = f(x), the function f is even.

4. Since f(x) = 3 x = 3 x = f(x), the function f is odd.

5. Since f(x) = (x)4 + (x) = x4 x 6= (x4 + x) = f(x), f is not an odd


function.
Similarly, since f(x) = (x)4 + (x) = x4 x 6= x4 + x = f(x), f is not an even
function.
It follows that the function f is neither odd nor even.

Note: The functions f(x) = cos x and f(x) = sec x are even, whereas the functions
f(x) = tan x, f(x) = cot x, f(x) = sin x and f(x) = cosec x are odd.

5.2.3 Zeroes of functions

Definition 5.6. Let f be a function from a set A into a set B. A zero of the function f
is a number x0 in A such that f(x0 ) = 0.

Examples 5.7. Find zeroes of the following functions.

x+2 1
1. f(x) = 3x + 1 (b) f(x) = 9 x2 (c) f(x) = (d) f(x) = .
x1 x+1

Soln: 1. Let f(x) = 3x + 1. We want to solve the equation f(x) = 0. Now


1
f(x) = 0 3x + 1 = 0 x = .
3
1
It follows that x = is the zero of the function f(x) = 3x + 1. Indeed,
3
   
1 1
f =3 + 1 = 1 + 1 = 0.
3 3

86

2. Let f(x) = 9 x2 . We want to solve the equation f(x) = 0. Now,

f(x) = 0 9 x2 = 0 9 x2 = 0 (3 x)(3 + x) x = 3.

Therefore x = 3 and x = 3 are zeroes of the function f(x) = 9 x2 . Indeed,
p
f(3) = 9 (3)2 = 9 9 = 0 and
p
f(3) = 9 (3)2 = 9 9 = 0.

x+2
3. Let f(x) = . We want to solve the equation f(x) = 0. Now,
x1
x+2
f(x) = 0 x + 2 = 0 x = 2.
x1
x+2
Therefore x = 2 is the zero of the function f(x) = . Indeed,
x1
(2) + 2 0
f(2) = = = 0.
(2) 1 3

1
4. Let f(x) = . If we try to solve the equation f(x) = 0, we get that 1 = 0, which
x+1
1
is absurd. Therefore the function f(x) = = 0 has no zeroes.
x+1


Exercise 5.8. 1. If f(x) = 4x + 1, find f(0), f(2), f( 2).

2. If g(x) = x2 x + 1, find g(2), g(k), g(x).

3. The domain of a function f is A = {1, 2, 3, 4, 5} and f(1) = 2, f(2) = 1, f(3) = 0,


f(4) = 1, f(5) = 2. What is the range of f? Draw an arrow diagram and a graph of f.

4. Find the domain of the following functions.


r
x x
(a) f(x) = (d) k(x) =
2x 2x

1
(b) g(x) = (e) `(x) = 3
x+2
x2 + x 6

x1
(c) h(x) = 4 3x + 4 (f) p(x) = .
2x 5
5. Determine whether each of the following functions is even, odd or neither.
sin x
(a) f(x) = x7 + x (d) F (x) =
x

87
(b) g(x) = 2 3x4 (e) G(x) = x cos x
(c) h(x) = 5x + 3x2

6. Find zeroes of each of the following functions.



x2 1 x+1
(a) f(x) = (d) f(x) =
x x

1
(b) f(x) = (e) f(x) = x2 5x + 6
x2 4

x
(c) f(x) = 2
(f) f(x) = 2x+ x2
x +1
7. Let f : X Y be a function, where X = {1, 2, 3, 4, 5, 6, 7, 8, 9} and Y = Z.
Draw graphs of the following functions.
x
2 if x is even
(a) f(x) =

1 if x is odd

x + 1 if x is divisible by 3
(b) f(x) =

x 1 otherwise

1 if x is even
(c) f(x) =

0 if x is odd

5.3 Linear Function

In this section we discuss linear functions and their graphs.

Definition 5.9. A linear function in the variable x is any function f that can be written
in the form
f(x) = ax + b, where a and b are real numbers.
If a 6= 0 then the linear function f is called a polynomial of first degree.

Here are some examples of linear functions:


1 2
f(x) = 2x + 1, g(x) = 6x, h(x) = 3, k(x) = x .
2 3

The graph of a linear function is a straight line. Graphing a linear function is quite easy.

88
The y-intercept of a straight line is the point where the graph of the line cuts the y-axis.
The y-intercept of the straight line y = ax + b is (0, b).
The x-intercept of a straight line is the point where
 the graph of the line cuts the x-axis.
b
The x-intercept of the straight line y = ax + b is , 0 .
a
Consider the straight line
y = ax + b, where a 6= 0.
The number a is called the slope (or gradient) of the straight line y = ax + b.
If a > 0, i.e., the slope is positive, then the graph of the function f(x) = ax + b rises as one
moves from the left to the right.
If a < 0, i.e., the slope is negative, then the graph of the function f(x) = ax + b falls as one
moves from the left to the right.

5.3.1 Sketching a graph of a linear function

In this subsection we discuss two easy methods of sketching a graph of a linear function. We
shall call these the intercepts method and the slope-intercept method.

Suppose that we want to sketch the graph of

y = ax + b, where a 6= 0.

 
b
Method of Intercepts: Find the x-intercept by setting y = 0. The xintercept is ,0 .
a
Find the y-intercept by setting x = 0. The y-intercept is (0, b).
Plot these two points on your graph and draw a straight line between them.
y 
6
 b)
(0,






-
 b , 0) 0 x
(
 a


Method of Intercepts

Slope-intercept Method The slope a of the straight line y = ax + b, where a 6= 0, is given


by
vertical change change in y
a= = .
horizontal change change in x

89
Find the y-intercept of the straight line. It is (0, b).
y
6 



 a

(0, b)
 1
 -

 0 x



Slope-intercept Method

Examples 5.10. 1. Use the two methods discussed above to sketch the graphs of the
following linear functions.
1
(a) y = 2x + 3 (b) y = x 1.
3
Soln: (a) Using the Method of Intercepts: For x intercept, set y = 0. The x-intercept is
3

2
,0 .
For the y-intercept, set x = 0. The y-intercept is 0, 3).
Plot these two points and join them by a straight line.

y6
A
A
A
(0, 3) A
A
A
A
A
A
A 3 -
0 A ( 2 , 0) x
A
A
A

Using the Slope-intercept method: Note that the slope of the straight line y =
2
2x + 3 is a = 2 = and its y-intercept is (0, 3).
1
y6
A
A
A 1
(0, 3) A
A 2
A
A
A
A
A -
0 A x
A
A
A

90
(b) Using the Method of Intercepts: For the x-intercept, set y = 0. Then
1
x 1 = 0 x = 3. Therefore, the x-intercept is (3, 0).
3
For the y-intercept, set x = 0. Then y = 1. That is, the y-intercept is (0, 1).
Plot these two points and join them by a straight line.
y6

 
 
 -
0  (3, 0) x


  (0, 1)



1
Using the Slope-intercept method: The slope of the line y = 31 x 1 is a = and
3
its y-intercept is (0, 1).
y6

 
 

 -
0  x
 1
(0, 1) 
 
 3


2. On the same system of axes, sketch the graph of

(a) y = 2x + 3 (b) y = 2x (c) y = 2x 3.


2
Soln: Slope = 2 = .
1
y
AA 6
A AA AA
A A A
y=

y=

y=

A A A
A A A
2

A A A
x

x+

A A A
A A A
3

A A A
A A A
A A A -
A 0A A x
A A A
A A A
A A A
A A A
A A A
A A A
A A A
A A A
A A A
91
Since these three lines have the same slope of 2, they are parallel.

3. Let f(x) = 3x 2; x R.

(a) Find f(0), f(1), f(1) and indicate them on the graph of f.
(b) Find the values of x for which
(i) f(x) = 0 (ii) f(x) > 0 (iii) f(x) < 0.
(c) Is f increasing or decreasing in R?

Soln: (a) f(0) = 2; f(1) = 3 1 2 = 1; f(1) = 3(1) 2 = 5.


y

6 

1 - f(1) 

 -
-

1 0  1 - x

1 - 

2 - f(0)

3 -

4 -

5 - f(1)



2
(b) (i) f(x) = 0 3x 2 = 0 x = .
3
2
(ii) f(x) > 0 = 3x 2 > 0 = x > .
3
2
(iii) f(x) < 0 = 3x 2 < 0 = x < .
3
(c) f(x) = 3x 2 is increasing in R since its slope, a = 3, is positive.

4. Find the linear function f(x) = ax + b such that f(3) = 1 and f(2) = 4.

Soln:

1 = a(3) + b 3a + b = 1
=

4 = a 2 + b 2a + b = 4

From these two equations, we have that a = 1 and b = 2. Therefore f(x) = x2.

5. The graph of the function f(x) = ax + b passes through the points (1, 4) and (1, 2).
Find the function f.

Soln:

92

4 = a 1 + b a + b = 4
=

2 = a(1) + b a + b = 2.

From these two equations we have that a = 3 and b = 1. Therefore f(x) = 3x1.

6. Draw the graphs of the following functions.



x+1 if x 0
(a) f(x) =

x + 1 if x < 0


2x 2 if x < 2



(b) f(x) = 2 if 2 x < 1





x + 3 if x 1
Soln:
y=
y A y
6 A 6
2

@ A
y

@
x
=

A
@
x

A
x

@
2

A
+

@ 2-
1

y
@ @

=
@ @


1 1-

x
@

+
@

3
- -
@
-

-
-
0 x 2 1 0 1 2 3@ x
@
@

(a) (b)

7. The relationship between the temperature measured in Celsuis (C) and Fahrenheit (F)
is given by
C F 32
= .
5 9
(a) Obtain C as a function of F .
(b) Obtain F as a function of C.
(c) What is
(i) 100 degrees F in terms of degrees C?
(ii) 32 degrees F in terms of degrees C?
(iii) 100 degrees C in terms of degrees F ?
(iv) 30 degrees C in terms of degrees F ?

93
5 160
Soln: (a) C = F (Linear function of F ).
9 9

9
(b) 9C = 5F 160 5F = 9C + 160 F = C + 32.
5
(Linear function of C).

5 160
(c) (i) C = 59 F 160
9
; C= 100 37.8.
9 9
5 160
(ii) C = 32 = 0.
9 90
9 9
(iii) F = C + 32; F = 100 + 32 = 212.
5 5
9
(iv) F = 30 + 32 = 86.
5

We can solve a sytem of two linear equations in two unknowns


(i) algebraically and (ii) graphically.
Two linear functions either (i) intersect at one point, (ii) are parallel (have the same slope) or
(iii) are coincident (they represent the same straight line).

Example 5.11. Solve the following systems of linear equations.



x + y = 2 . . . (1)
(a)

x y = 0 . . . (2)


x + y = 1 . . . (1)
(b)

xy = 1 . . . (2)


2x y = 0 . . . (1)
(c)

2x + y = 1 . . . (2)

Soln: (a) Consider the system:



x + y = 2 . . . (1)

x y = 0 . . . (2)

94
Algebraic solution: Adding equations (1) and (2), we have that 2x = 2 and so
x = 1. Substituting the value x = 1 in equaton (2), we have that y = 1. Therefore
the solution of the given system if (1, 1).

Graphical solution:
y
@ 6
@

x
=
@

y
@
@
@
1 @
. (1, 1)
.. @
..
.. @

y
.. @ -

=
@ x


0 1

x
@

+
@

2
(b) Consider the system:
x + y = 1 . . . (1)

xy = 1 . . . (2)
Algebraic solution: Equations (1) and (2) stand for the same line x y = 1.
Therefore this system has infinitely many solutions.

Graphical solution:
y
1

6
=
y

x

(x, x 1)
-
-

0 1 x

(c) Consider the system:


2x y = 0 . . . (1)

2x + y = 1 . . . (2)
Algebraic solution: Adding equations (1) and (2), we have that 0 x + 0 y =
1 0 = 1, which is absurd. Therefore the system has no solution.

95
Graphical solution:
y
6  
 
 

2x
 
 

y=
 

+ 1
 

2x
 
 

y=
 
  -
 0 x
 
 
 
 
 

Exercise 5.12. 1. Draw the graphs of the following functions.

(a) y = x 2 (b) y = 21 x + 1

(c) y = 2x + 4 (d) y = 23 x + 2


2x 1 if x 0
(e) f(x) =

1 if x < 0

3x + 3 if x 1
(f) f(x) = 0 if 1 x < 1

3x + 3 if x < 1


x+4 if x 1



(g) f(x) = 2x + 6 if 1 < x < 5





x6 if x 5

2. Points A(1, 2) and B(1, 4) lie on the graph of y = ax + b. Determine whether point
C(2, 6) lies on this graph.

3. The graph of the linear function y = ax + b is parallel to the graph of y = 2x + 1 and


is passing through A(1, 5). Find the function.

4. Show that if A(p, q) lies on the graph of y = x + 5 then p + q = 5.

5. For what values(s) of m is y = (2m 3)x + 1.

(a) increasing?
(b) decreasing?
(c) constant?

96
6. Let f(x) = 2x 5. Solve the following inequalities graphically.

(a) f(x) > 0.


(b) f(x) < 0.
(c) f(x) 1.
(d) f(x) < 3.

7. Let f(x) = x + 5 and g(x) = 2x + 10.

(a) Draw the graphs of f and g on the same system of axes.


(b) Use the graphs of f and g to find the values of x for which
(i) f(x) = g(x).
(ii) f(x) g(x).

8. Solve the following systems graphically.



x + 2y = 1 3x + y = 2
(a) (b)

x y = 2 xy = 4
9. There is a linear function which describes population growth in town. Assume that the
population changes in time t linearly. This means that the population at any time t is
given by P (t) = at + b, (t in years).
Given that the population size in 1990 was 30 000 people and in 2000 it was 32 000
people.

(a) Find the linear function P .


(b) What is the population size in 2004?

10. The monthly cost of driving a car depends on the number of miles driven. Jane found
that in May it cost her $380 to drive 480 miles and in June it cost her $460 to drive
800 miles.

(a) Express the monthly cost C as a function of the distance driven d, assuming that
a linear function gives a suitable model.
(b) Use part a) to predict the cost of driving 1500 miles per month.
(c) Draw the graph of the function.
(d) What does the y-intercept of the graph represent?
(e) Why is a linear function a suitable model in this situation?

97
5.4 Quadratic Functions

Definition 5.13. A quadratic function in a variable x is a function of the form

f(x) = ax2 + bx + c,

where a, b, and c are real numbers with a 6= 0.


The graph of the quadratic function is a parabola.

Consider the quadratic function

f(x) = ax2 + bx + c,

where a, b, and c are real numbers with a 6= 0.

If a > 0, then the parabola opens upward and has a lowest point. On the other hand,
if a < 0, then the parabola opens downward and has a highest point.

The greater |a| the narrower the parabola.

The lowest or highest point of a parabola is called a vertex.

The line that passes through the vertex and splits the parabola into two equal halves is
called the axis of symmetry (or simply axis) of the parabola.

y6 y6

- -
0 x 0 x

a>0 a<0

b
The x-coordinate of the vertex of the parabola y = ax2 + bx + c, a 6= 0 is x = . Its
2a
ycoordinate is
   2  
b b b
f = a +b +c
2a 2a 2a
b2 b2
= +c
4a 2a
b2 + 4ac b2 4ac
= = .
4a 4a

98
That is, the coordinates of the vertex (or turning point) of the parabola y = ax2 +bx+c, a 6= 0
b b2 + 4ac
are x = and y = . That is, the turning point (vertex) of the parabola is the
2a 4a
point  
b b2 + 4ac
, .
2a 4a

The function y = ax2 + bx + c, a 6= 0 can be written in the form y = a(x p)2 + q.


(Canonical Form)

y = ax2 + bx + c
 
2 b c
= a x + x+
a a
"  2   #
2 b b b c
= a x +2 x+ +
2a 2a 2a a
" 2 #
b b2 c
= a x+ 2+
2a 4a a
" 2 #
2
b b 4ac
= a x+
2a 4a2
" 2 #
b
= a x+ 2
2a 4a
 2
b
= a x+
2a 4a

b
Let p = and q = . We obtain f(x) = (x p)2 + q.
2a 4a
There is a relationship between the graph of the function f(x) = ax2 + bx + c, a 6= 0 and
the discriminant = b2 4ac.

(i) If = b2 4ac > 0, then the graph of f(x) = ax2 + bx + c crosses the xaxis in two
places, i.e., there are two x-intercepts;

(ii) If = b2 4ac = 0, then the graph of f(x) = ax2 + bx + c will touch the x-axis at
its vertex;

(iii) If = b2 4ac < 0, then the graph of f(x) = ax2 + bx + c will not cross the x-axis,
i.e., the function f(x) = ax2 + bx + c has no zeroes.

99
y6 y6

- -
0 x 0 x

> 0: there are two x-intercepts. The function has two distinct zeroes

y6 y6

-
0 x
-
0 x

= 0: there is one x-intercept. The function has one repeated zero

y6 y6

-
0 x
-
0 x

< 0: there is no x-intercept. The function has no zeroes

The graph y = ax2 + c is a parabola with axis of symmetry x = 0 (the y-axis).

The function f(x) = ax2 + bx + c, a 6= 0 has the following properties:

(i) Dom(f) = (, )
 

(ii) Range of f is ; if a > 0
 4a 

Range of f is , if a < 0
4a

(iii) ymin = if a > 0; ymax = if a < 0
4a 4a
! !
b + b
(iv) xintercepts are , 0 and , 0 if > 0.
2a 2a
 
b
,0 if = 0
2a

100
b
(v) Axis of symmetry if x = .
2a
 
b
(vi) Turning point (vertex) = ; .
2a 4a
(vii)
 
b

The function increases in ;
2a  if a > 0.
b

and decreases in ;
2a
 
b

The function increases in ;
 2a
 if a < 0.
b

and decreases in ;
2a

Now recall Canonical Form of Quadratic Function.


" 2 #
b
y = a x+ 2
2a 4a
!2
 2
b
= a x + if > 0
2a 2a
! !
b b
= a x+ x+ +
2a 2a 2a 2a
" !# " !#
b + b
= a x x
2a 2a
= a (x x1) (x x2) ,

b + b
where x1 = and x2 = are the zeroes of the function.
2a 2a
Finding the equation of a parabola

Consider three cases.

Case 1: Given the vertex (turning point) and one other point on the parabola. We use the form:

y = a(x p)2 + q.

The coordinates of the turning point of the parabola are (p, q).

Example 5.14. Find the equation of the parabola with vertex (2, 6) which
passes through (1, 5).

101
Soln: The required equation of the parabola has the form

y = a(x + 2)2 + 6,

where a is to be determined. Since the point (1, 5) lies on the parabola, we must
have that
5 = a(1 + 2)2 + 6 = a + 6 a = 1.
The required equation is

y = (x + 2)2 + 6 = x2 4x + 2.

Case 2: Given the x-intercepts and one other point on the parabola. We use the form:

y = a(x x1)(x x2),

where (x1 , 0) and (x2, 0) are the given x-intercepts and a is to be determined.
Example 5.15. Find the equation of the parabola with x-intercepts (2, 0) and
(4, 0) and passing through (3, 14).
Soln: The required equation has the form

y = a(x 2)(x + 4).

Since the point (3, 14) lies on the parabola, we must have that

14 = a(3 2)(3 + 4) 14 = 7a a = 2.

The required equation is

y = 2(x 2)(x + 4) = 2x2 4x + 16.

Case 3: Given any 3 points on the parabola. We use the form:

y = ax2 + bx + c

and solve a system of three equations in three unknowns.


Example 5.16. Find the equation of the parabola passing through (1, 2); (1, 2)
and (2, 7).
Soln: We use the form: y = ax2 +bx+c. Since the given three points lie on the parabola,
we must have that
2 = a + b + c ......(1)
2 = a b + c ......(2)
7 = 4a + 2b + c ......(3)
Equation (1) minus equation (2) gives 2b = 4, and so b = 2. Adding equations (1)
and (2), we have that 2a + 2c = 0, and so a = c. Substituting these in equation
(3), we get
7 = 4a + 4 a 3a = 3 a = 1.
It now follows that c = 1. That is, a = 1, b = 2 and c = 1. Therefore the
required equation is y = x2 + 2x 1.

102
Exercise 5.17. 1. Write the following quadratic functions in the form y = a(x p)2 + q.

(a) y = 2x2 + 4x 6
1
(b) y = x2 3x 2
2
(c) y = 3x2 2x + 1.

2. Sketch the graphs of the functions on seperate sets of axes.



(a) y = x2 2x 3





(b) y = 3x + 6x 1
2
For each of the functions, find
intercepts on the axes, axis of

(c) y = x2 + 3x 4
symmetry, turning point and range.





(d) y = x2 + 6x + 9

3. Make rough sketches of y = ax2 + bx + c if

(a) a > 0, b > 0, c > 0 and > 0


(b) a > 0, b < 0, c > 0 and > 0
(c) a < 0, b > 0, c = 0
(d) a < 0, b < 0, c > 0
(e) a < 0, b > 0, c < 0 and < 0.

4. Find r and s in y = x2 + rx + s if ymin = 5 for x = 2.

5. Find k and ` in f(x) = x2 + kx + ` if f(2) = 0 and f(3) = 0.

6. Find the equations of the parabolae in the sketch if A and B are their turning points.
y
6
B(3, 9)

-
0 x


A(2, 4)

103
7. In each of the following cases, find the equation of the parabola.

(a) Given vertex (1, 5), passing through (1, 13).


(b) Given axis of symmetry x = 4, passing through (3, 2) and with minimum y-
value 1.
(c) Given the x-intercepts (1, 0) and (3, 0) and passing through (1, 4).
(d) Passing through the points (1, 4), (2, 5) and (0, 1).

8. f : y = ax2 + bx + c. Find a, b and c if ymax = 12 when x = 1 and point A(3, 0) lies on


the graph of f.

5.5 Quadratic Inequalities

Definition 5.18. A quadratic inequality is an inequality of the form

ax2 + bx + c > 0, ax2 + bx + c < 0 with a 6= 0 (Strong inequalities)

or
ax2 + bx + c 0, ax2 + bx + c 0 with a 6= 0 (Weak inequalities).

These inequalities can be solved algebraically and graphically. The algebraic solution is based
on the following facts:

1. A product of two real numbers is nonnegative if and only if the two real numbers have
the same sign.

2. A product of two real numbers is nonpositive if and only if the two real numbers have
opposite signs.

Examples 5.19. 1. Solve the inequality x2 2x 3 > 0.

Soln:

Algebraic solution:

x2 2x 3 > 0 (x + 1)(x 3) > 0


x + 1 > 0 and x 3 > 0 OR x + 1 < 0 and x 3 < 0
x > 1 and x > 3 OR x < 1 and x < 3
= x > 3 OR x < 1.

104
Graphical solution: Let f(x) = x2 2x 3.

y6

f(x) > 0 for x < 1 or x > 3


+ +
-
1 0 3 x

2. Solve the inequality 2x2 x 1 0.

Algebraic solution:

2x2 x 1 0 (2x + 1)(x 1) 0


2x + 1 0 and x 1 0 OR 2x + 1 0 and x 1 0
1 1
x and x 1 OR x and x 1
2 2
1
= x 1.
2

Graphical solution: Let g(x) = 2x2 x 1.

y6

1
g(x) 0 for x 1
2
-
21 0 1 x

3. Solve the inequality x2 25 0.

Soln: Algebraic solution:

x2 25 0 (x 5)(x + 5) 0
x + 5 0 and x 5 0 OR x + 5 0 and x 5 0
x 5 and x 5 OR x 5 and x 5
= x 5 OR x 5.

105
Graphical solution: Let h(x) = x2 25.

y6 h(x) 0 for x 5 or x 5
+ +
-
5 0 5 x

4. Solve the inequality x2 2x + 1 0.

Soln: Algebraic solution:


x2 2x + 1 0 (x 1)2 0.
This is true for all real numbers x.

Graphical solution: Let f(x) = x2 2x + 1.


y6

-
0 1 x

5. Solve the inequality x2 + 6x + 9 > 0.

Soln: Algebraic Solution:


x2 + 6x + 9 > 0 (x + 3)2 > 0.
This is true for all real numbers except 3.

Graphical solution: Let g(x) = x2 + 6x + 9.

y6

-
3 0 x

106
6. Solve the inequality x2 + 6x + 7 0.

Soln: Algebraic solution:

x2 + 6x + 7 0 x2 6x 7 0
(x 7)(x + 1) 0
x 7 0 and x + 1 0 OR x 7 0 and x + 1 0
x 7 and x 1 OR x 7 and x 1
= 1 x 7.

Graphical solution: Let g(x) = x2 + 6x + 7.

y6

+ g(x) 0 for 1 x 7
-x
1 0 7

7. Solve the inequality 3x2 x + 1 < 0.

Soln: Algebraic solution: Since the discriminant = 1 12 = 11 < 0, this inequality has
no solution.

Graphical solution: Let h(x) = 3x2 x + 1.

y6

g(x) > 0 for all real numbers x

-
0 x

107
8. Solve the inequalities 4x2 9 < 0 and x 1 0.

Soln: Let f(x) = 4x2 9 < 0.

y6
3 -
23 0 2
x

f(x) < 0 for 32 < x < 3


2

4x2 9 < 0 and x 1 0


(2x 3)(2x + 3) < 0 and x 1
3 3
<x< and x 1.
2 2

JJ J J J J J

JJ J J J J J J J J J

JJ JJJ J J J J J J J J J J
J
J
J

3

-
32 0 1 62 x
3
The common area is 1 x < 2

9. Solve the inequalities x2 4 < 0 and 3x2 + 2x 5 0.

Soln:
y6 y6
3x2 + 2x 5 0
- -
2 0 2 x 53 0 1 x
2
x 4<0

2 < x < 2 x 53 or x 1

JJJJJ

J JJJJ J J J J JJJ J J

J





J
J J J J J J J JJ
5
J J J J
J

J



-
x
2 6 3 0 1 62
Common area is 1 x < 2
Common area is 2 < x 53 .

Therefore the solution is 2 < x 35 or 1 x < 2.

108
Rational Inequalities can also be handled in the same way as above.
Note: If p and q are real numbers such that q 6= 0, then the following statements are true:

p
> 0 p q > 0.
q
p
0 p q 0.
q
p
< 0 p q < 0.
q
p
0 p q 0.
q
x+2
Examples 5.20. 1. Solve the inequality >0
x5
Soln: Algebraic solution:
x+2
>0 (x + 2)(x 5) > 0
x5
x + 2 > 0 and x 5 > 0 OR x + 2 < 0 and x 5 < 0
x > 2 and x > 5 OR x < 2 and x < 5
x > 5 OR x < 2.

Graphical Solution:

y
6

+ +
-
2 0 5 x

x+2
> 0 on x < 2 or x > 5.
x5

2x 1
2. Solve the inequality 0.
x+3
Soln: Algebraic solution: Note that x + 3 6= 0; i.e., x 6= 3.
2x 1
0 (2x 1)(x + 3) 0
x+3
2x 1 0 and x + 3 < 0 OR 2x 1 0 and x + 3 > 0
1 1
x and x < 3 OR x and x > 3
2 2
1
= 3 < x .
2
Therefore the solution set is 3 < x 21 .

109
Graphical solution:

y6

-
3 0 1 x
2
2x 1
0 on 3 < x 12 .
x+3

2
3. Solve the inequality 1.
x+3
Soln: Algebraic solution: Note that x 6= 3.
2 2
1 10
x+3 x+3
2x3
0
x+3
x 1
0
x+3
x+1
0
x+3
(x + 1)(x + 3) 0
x + 1 0 and x + 3 > 0 OR x + 1 0 and x + 3 < 0
x 1 and x > 3 OR x 1 and x < 3
x 1 OR x < 3.

Therefore the solution set consists of all real numbers x such that x 1 or x < 3.

Graphical solution:

y6

-
3 1 0 x
2
1 on x 1 or x < 3
x+3

x+1
4. Solve the inequality 2.
x4

110
Soln: Algebraic solution: In this example demonstrate another method of solving inequalities.
Multiply both sides of the inequality by the square of the denominator.

x+1 (x + 1)(x 4)2


2 2(x 4)2
x4 x4
(x + 1)(x 4) 2(x 4)2
x2 3x 4 2(x2 8x + 16)
x2 3x 4 2x2 + 16x 32 0
x2 + 13x 36 0
x2 13x + 36 0 (note the reversal of inequality)
(x 4)(x 9) 0
x 4 > 0 and x 9 0 OR x 4 < 0 and x 9 0
x > 4 and x 9 OR x < 4 and x 9
= 4 < x 9.

Graphical solution:
y
6

-
0 4 9 x

(x 4)(x 9) 0 on 4 < x 9

Exercise 5.21. 1. Solve the following inequalities algebraically and graphically.


x+3
(a) x2 5x + 6 0 (h) <0
x1

3x + 5
(b) x2 3x + 18 < 0 (i) 0
x2

2x 5
(c) 10 + x 2x2 < 0 (j) >0
3x

111
9
(d) x2 x 1 0 (k) 3
x+1

2x 1 3 2
(e) <0 (l)
x+2 x9 x+2

2x 3 3x + 2
(f) >0 (m) 0
2x + 3 2x 7

x2 + 3x + 2 x2
(g) <
x2 x+2

2. Solve the following inequalties,

(a) x 1 1 3x and 3x + 2 7

1 2x 1 + 3x
(b) < and 1 7x 6x
3 4
3. Solve the following inequalities algebraically and graphically.

x2 + x 2 > 0 and 2 x2 0

x2 x 6 0 and x2 1 > 0
3x 2
4. (a) Solve the inequalities 1 and x2 < 30 graphically.
x4
(b) Find all integers which satisfy (a).

5. Find the value(s) of p for which the following inequalities are satisfied for any x R.

(a) x2 + 3x p > 0
(b) px2 + 12x 5 < 0
(c) (p 4)x2 2px + 2p > 0

6. Solve the following inequalities algebraically and graphically.

(a) (x 1)2 3
(b) 3 < x2 2x 3 4
(c) (x + 2)2 < 2x

112
Chapter 6

Polynomials

Definition 6.1. A polynomial in x is a function that can be written as

p(x) = an xn + an1 xn1 + + a1 x + a0 ,

where a0, a1 , a2, . . . , an are real numbers.


The letter x is called a variable. The real numbers a0, a1, a2, . . . , an are called the
coefficients of the polynomial p(x).
If an 6= 0, then we say that p(x) is a polynomial of degree n. In this case, we write
deg p(x) = n. The number an is called the leading coefficient of the polynomial p(x)
and a0 is called the constant term.

Note that the powers of x in the polynomial p(x) = an xn + an1 xn1 + + a1x + a0 are
nonnegative integers.

A polynomial of degree zero is called a constant polynomial;

A polynomial of degree one is called a linear polynomial;

A polynomial of degree two is called a quadratic polynomial;

A polynomial of degree three is called a cubic polynomial;

A polynomial of degree four is called a quartic polynomial;

A polynomial of degree five is called a quintic polynomial.

A polynomial with only one term is called a monomial.


A polynomial with a leading term of 1 is called a monic polynomial.

Definition 6.2. A zero of a polynomial p(x) is a number r such that p(r) = 0.

113
Clearly, a zero of a polynomial p(x) is the same as the root of the equation p(x) = 0.

Equating two polynomials: Two polynomials are equal if and only if the corresponding coeffi-
cients are equal.
Examples 6.3. 1. Consider the polynomial p(x) = 6x5 + 2x4 x3 + x 3. We observe
the following features of this polynomial:

p(x) is a polynomial of degree 5, i.e., p(x) is a quintic polynomial;


the leading term of the polynomial p(x) is 6x5 ;
the leading coefficient of the polynomial p(x) is 6;
the constant term of the polynomial p(x) is 3;
the coefficient of x3 is 1.

2. Consider the polynomial p(x) = x4 2x3 + 8x2 3x + 9. We observe the following


features of this polynomial:

p(x) is a polynomial of degree 4, i.e., p(x) is a quartic polynomial;


the leading term of the polynomial p(x) is x4 ;
the leading coefficient of the polynomial p(x) is 1, i.e., p(x) is a monic polynomial;
the constant term of the polynomial p(x) is 9;
the coefficient of x is 3.

3
3. The numbers x = 1, x = 0, and x = are zeroes of the polynomial
2
p(x) = 2x4 x3 + 3x2 since
3
p(1) = 0, p(0) = 0, and p( ) = 0.
2

6.1 Addition and Subtraction of Polynomials

Addition (Subtraction) of polynomials (with the same variable, say x) is carried out by adding
(subtracting) the coefficients of corresponding powers of x. In other words, we collect like
terms and then add (subtract) in the usual way.
Examples 6.4. 1. Add the polynomials a(x) = x4 3x2 + 2x 5 and
b(x) = 2x5 + 4x3 + 3x2 + 2.

Soln:
x4 3x2 + 2x 5
5 3
+ 2x + 4x + 3x2 + 2
2x5 + x 4
+ 4x3 + 2x + 3

The answer is 2x5 + x4 + 4x3 + 2x 3.

114
2. If p(x) = x3 + 2x2 1 and q(x) = 2x4 x3 + 3x2 x 4, find 3p(x) 2q(x).

Soln: Since

3p(x) = 3 x3 + 2x2 1 = 3x3 + 6x2 3 and

2q(x) = 2 2x4 x3 + 3x2 x 4 = 4x4 + 2x3 6x2 + 2x + 8,

it follows that 3p(x) 2q(x) is

3x3 + 6x2 3
4
4x + 2x3 6x2 + 2x + 8
4x4 + 5x3 + 2x + 5

That is, 3p(x) 2q(x) = 4x4 + 5x3 + 2x + 5.

Exercise 6.5. 1. Find the following sums and differences.



(a) (x6 + 3x4 5x2 + 2x + 1) + (3x5 + 2x4 3x 5)

(b) (10x8 2x5 + x) + (3x4 x2 + 7x 1)

(c) (x3 x + 3) ( 2x + 1)

(d) (2x8 10x4 + x 3) ( 2x8 + 3x4 x3 + 3x)

2. If a(x) = 2x2 x + 5, b(x) = 3x and c(x) = x3 + x 2, find

(a) 3a(x) + b(x)


(b) 2b(x) c(x)
(c) xa(x) + c(x)
(d) a(x) 2xb(x)

3. Find polynomials a(x) and b(x) such that deg(a(x) + b(x)) 6= max{deg a(x); deg b(x)}.

6.2 Multiplication of Polynomials

Multiplication of polynomials is based on the distribution property of multiplication: If a, b, c


and d are real numbers, then

(a + b)(c + d) = ac + ad + bc + bd.

When two non-constant polynomials are multiplied together, every term in each polynomial
has to be multiplied by every term in the other polynomial. Then like terms must be collected
and added together.

Examples 6.6. 1. Write out the product (x3 2x2 4)(2x3 + 5x2 3x + 2) in full.

115
Soln: (x3 2x2 4)(2x3 + 5x2 3x + 2) is equal to
2x6 + 5x5 3x4 + 2x3
4x5 10x4 + 6x3 4x2
8x3 20x2 + 12x 8
6 5 4
2x + x 13x 24x2 + 12x 8
The required product is
(x3 2x2 4)(2x3 + 5x2 3x + 2) = 2x6 + x5 13x4 24x2 + 12x 8.

2. Find the product (x2 2x 5)(2x2 9x + 5).

Soln: The product (x2 2x 5)(2x2 9x + 5) is

2x4 9x3 + 5x2


4x3 + 18x2 10x
10x2 + 45x 25
4 3
2x 13x + 13x2 + 35x 25

The required product is (x2 2x 5)(2x2 9x + 5) = 2x4 13x3 + 13x2 + 35x 25.

Note:

1. If a(x) and b(x) are non-zero polynomials, then deg(a(x)b(x)) = deg a(x) + deg b(x)
i.e. the degree of a product is equal to the sum of the degrees of the factors.

2. The leading coefficient of a non-zero product of polynomials is the product of the leading
coefficients of the factors.

3. The constant term of a product of polynomials is the product of the constant terms of
the factors.

6.3 Polynomial Division

For integers a, b with b a we can write


a r
=q+ or a = b q + r
b b

23 2
For example, = 3 + or 23 = 3 7 + 2 where r = 2 is the remainder after division of 23
7 7
by 7 and q = 3 is the quotient.

We do the same thing for polynomials.


If a(x) and b(x) are two polynomials with deg b(x) deg a(x) then we can find a quotient
q(x) and a remainder r(x) with deg r(x) < deg b(x), such that
a(x) r(x)
= q(x) + or a(x) = b(x) q(x) + r(x).
b(x) b(x)

116
The polynomial a(x) is called the dividend; the polynomial b(x) is called the divisor and
the polynomial r(x) is called the remainder.

If the remainder is zero, i.e., if a(x) = b(x)q(x), then we say b(x) is a factor of a(x) or a(x)
is divisible by b(x).

Note: If deg b(x) deg a(x), then deg q(x) = deg a(x) deg b(x).

Division algorithm

1. Divide the leading term of the dividend by the leading term of the divisor, and write the
quotient at the top.

2. Multiply the whole divisor by this quotient.

3. Subtract this product from the dividend.

These three operations replace the dividend by a polynomial of lower degree. The process can
then be repeated until the degree of the resulting polynomial (the remainder) is less than that
of the divisor.

Examples 6.7. 1. Find the quotient and the remainder when 2x3 x2 + 5 is divided by
x2 2x + 4.

Soln:
2x + 3
2
x 2x + 4 | 2x3 x2 + 5
2x3 4x2 + 8x
3x2 8x + 5
3x2 6x + 12
2x 7

Thus the quotient is 2x + 3 and the remainder is 2x 7.


You should check that indeed 2x3 x2 + 5 = (2x + 3)(x2 2x + 4) + (2x 7).

2. Find the quotient and the remainder when 8x4 + 4x3 x 1 is divided by 2x 1.

Soln:

117
1
4x3 + 4x2 + 2x + 2
2x 1 | 8x4 + 4x3 x2 x 1
8x4 4x3
8x3
8x3 4x2
4x2 x
4x2 2x
x 1
x 21

1
2

The quotient is 4x3 + 4x2 + 2x + 21 and the remainder is 21 . You should check that
8x4 + 4x3 x 1 = (4x3 + 4x2 + 2x + 12 )(2x 1) + ( 12 ).

3. Find the quotient and the remainder when x5 + 4x3 x + 4 is divided by x3 x2 + 2x 1.

Soln:
x2 + x + 3
3 2
x x + 2x 1 | x5 + 4x3 x + 4
x5 x 4
+ 2x3 x2

x4 + 2x3 + x2 x
x4 x3 + 2x2 x
3x3 x2 + 4
3x3 3x2 + 6x 3
2x2 6x + 7

The quotient is x2 + x + 3 and the remainder is 2x2 6x + 7. You should check that
x5 + 4x3 x + 4 = (x2 + x + 3)(x3 x2 + 2x 1) + (2x2 6x + 7).

4. Find the quotient and the remainder when 27x3 1 is divided by 3x 1.

Soln:
9x2 + 3x + 1
3x 1 | 27x3 1
27x3 9x 2

9x2
9x2 3x
3x 1
3x 1
0

The quotient is 9x2 + 3x + 1 and the remainder is 0. You should check that
27x3 1 = (3x 1)(9x2 + 3x + 1).

118
Exercise 6.8. 1. Find the following products.

(a) (2x2 + 4x + 5)(3x3 + x2 x 2)


(b) (x3 3x2 + 2x 1)(2x4 x3 + x2 + 2x 1)
(c) (3x8 10x5 + 2x)(4x4 3x2 + x 5)

2. In the product (2x3 + 3x2 + x 2)(5x2 3x + 7), find

(a) the coefficient of x4.


(b) the coefficient of x3.
(c) the leading term.
(d) the constant term.

3. Use the Division Algorithm to find


x3 + x 1
(a)
x1

3x3 2x2 3x + 2
(b)
3x 2

2x3 + 9x2 17x + 6


(c)
2x 1

x3 + 125
(d)
x+5

x5 + 3x4 + 7x3 + 10x2 + 8x + 5


(e)
x2 + x + 4

6x4 + 4x3 9x2 x 1


(f)
3x2 x + 1
4. Let f(x) = x3 + x 1. Find f(1).

5. Let g(x) = 3x3 2x2 3x + 2. Find g( 23 ).

6. Let h(x) = 2x3 + 9x2 17x + 6. Find h( 12 ).

7. Let k(x) = 2x3 7x2 3x + 2 and `(x) = 2x + 3.

(a) Divided k(x) by `(x).


(b) Find k( 32 ).

119
6.4 The Remainder Theorem and Factor Theorem

In this section we discuss the Remainder Theorem and the Factor Theorem.

Theorem 6.9.(Remainder
 Theorem) If a polynomial p(x) is divided by ax + b, then the
b
remainder is p .
a

Proof: Dividing ax + b into p(x), we have

p(x) r
= q(x) + ,
ax + b ax + b
where q(x) is the quotient and r the remainder. Therefore

p(x) = (ax + b)q(x) + r

and so
       
b b b
p = a +b q +r
a a a
 
b
= (b + b)q +r
a
 
b
= 0q +r
a
= r.

Examples 6.10. 1. Find the remainder when p(x) = 2x3 4x2 + x 3 is divided by
3x + 1.

Soln:
   3  2  
1 1 1 1
p = 2 4 + 3
3 3 3 3
 
1 1 1
= 2 4 3
27 9 3

2 12 9 81
=
27 27 27 27
104
= .
27
104
Therefore the remainder is r = .
27

120
2. Let f(x) = 16x3 ax + 12x 3. When f(x) is divided by 2x + 1, then the remainder
is 7. Find the value of a.
 
1
Soln: We are given that f = 7. Therefore,
2
   3    
1 1 1 1
7 = f = 16 a + 12 3
2 2 2 2
 
1 1
7 = 16 + a63
8 2
1
7 = 2 + a 9
2
1
4 = a
2
8 = a.

That is, a = 8.

3. Let f(x) = ax3 + bx2 4x + 6. When f(x) is divided by x 1, the remainder is 1 and
if it is divided by x 2, the remainder is 2.

(a) Find a and b.


(b) Find the remainder when f(x) is divided by x + 1.

Soln: (a) We are given that f(1) = 1 and f(2) = 2. Therefore

1 = f(1) = a 13 + b 12 4 1 + 6 = a + b 4 + 6 = a + b + 2
2 = f(2) = a 23 + b 22 4 2 + 6 = 8a + 4b 8 + 6 = 8a + 4b 2

We obtain the following system of linear equations:


 
a + b = 1 a + b = 1 . . . (1)
8a + 4b = 4 2a + b = 1 . . . (2)

Solving this system we get that a = 2 and b = 3. Therefore the given polynomial is
f(x) = 2x3 3x2 4x + 6.

(b) Since f(x) = 2x3 3x2 4x + 6, the remainder when this polynomial is divided by
x + 1 is

f(1) = 2(1)3 3(1)2 4(1) + 6 = 2 3 + 4 + 6 = 5,

that is, when f(x) is divided by x + 1, the remainder is 5.

Theorem 6.11. (Factor Theorem).


 Let p(x) be a polynomial. Then ax + b is a factor of
b
p(x) if and only if p = 0.
a

121
 
b
Proof: We show that if ax + b is a factor of p(x), then p = 0. To that end, assume
a
that ax + b is a factor of p(x). Then

p(x) = (ax + b)q(x)

for some polynomial q(x). Now


       
b b b
p = a +b q
a a a
 
b
= (b + b) q = 0.
a
 
b
That is, p = 0.
a

 
b
Conversely, we show that if p = 0, then ax + b is a factor of p(x). Indeed, by the
  a
b
Remainder Theorem, p is the remainder when p(x) is divided by ax + b. That is,
a
 
b
p(x) = (ax + b)q(x) + p .
a
 
b
Since we are assuming that p = 0, it follows that
a

p(x) = (ax + b)q(x),

and so ax + b is a factor of p(x).

Examples 6.12. 1. If x 1 and x + 1 are factors of f(x) = ax3 + bx2 + 5x 5, determine


the values of a and b.

Soln: If x 1 and x + 1 are factors of f(x) = ax3 + bx2 + 5x 5, then f(1) = 0 and
f(1) = 0. Therefore

0 = f(1) = a 13 + b 12 + 5 1 5 = a + b + 5 5 = a + b
0 = f(1) = a (1)3 + b (1)2 + 5 (1) 5 = a + b 5 5 = a + b 10.

We get the following system of two equations in two unknowns:



a+b = 0
a + b = 10.

Solving these simultaneously, we have that a = 5 and b = 5.

122
2. Find k if x2 x 6 is a factor of f(x) = x3 + k 2 x2 11x 15k.

Soln: Note that x2 x 6 = (x 3)(x + 2). Since we are given that x2 x 6 is a factor of
f(x), we can conclude that x 3 and x + 2 are also factors of f(x). Therefore f(3) = 0
and f(2) = 0. Therefore

0 = f(3) = 33 + 9k 2 33 15k 9k 2 15k 6 = 0 3k 2 5k 2 = 0


(3k + 1)(k 2) = 0
1
k = 2 or k =
3
and

0 = f(2) = (2)3 + 4k 2 + 22 15k 4k 2 15k + 14 = 0


(k 2)(4k 7) = 0
7
k = 2 or k = .
4
The common value is k = 2.

3. If x + 2 is a common factor of of the polynomials f(x) = x2 + ax + a2 and


g(x) = x2 + bx + b2 , and a 6= b, show that a + b = 2.

Soln: Since x + 2 is a factor of f(x) and g(x), it follows that f(2) = 0 and g(2) = 0.
Therefore

0 = f(2) = (2)2 + a(2) + a2 = a2 2a + 4 a2 2a + 4 = 0 . . . . . . (1)


0 = g(2) = (2)2 + b(2) + b2 = b2 2b + 4 b2 2b + 4 = 0 . . . . . . ...(2)

Equation (1) minus equation (2):

a2 b2 2a + 2b = 0 (a b)(a + b) 2(a b) = 0
(a b)(a + b 2) = 0.

Since a 6= b, it follows that a + b 2 = 0 and so a + b = 2.

Finding roots of polynomial equations

How does one find roots of a polynomial equation of the form

an xn + an1 xn1 + + a1x + a0 = 0?

Let p(x) = an xn + an1 xn1 + + a1x + a0.

1. Find all the divisors of the constant term a0.

123
2. If the number x0 is a divisor of a0 and p(x0 ) = 0, then x x0 is a factor of p(x).

Examples 6.13. 1. Consider the polynomial p(x) = x4 4x3 7x2 + 22x + 24. The
divisors of the constant term 24 are:

1, 2, 3, 4, 6, 8, 12, 24.

Now,

p(1) = 14 4 13 7 12 + 22 1 + 24 = 1 4 7 + 22 + 24 = 36 6= 0,
and so x 1 is not a factor of p(x),

p(1) = (1)4 4 (1)3 7 (1)2 + 22 (1) + 24 = 1 + 4 7 22 + 24 = 0,


and so x + 1 is a factor of p(x).

Divide x + 1 into p(x) = x4 4x3 7x2 + 22x + 24 to get

x4 4x3 7x2 + 22x + 24 = (x + 1)(x3 5x2 2x + 24).

Let q(x) = x3 5x2 2x + 24. Now

q(2) = 23 5 22 2 2 + 24 = 10 6= 0,
and so x 2 is not a factor of q(x),

q(2) = (2)3 5 (2)2 2 (2) + 24 = 0,


and so x + 2 is a factor of q(x).

Divide x + 2 into q(x) = x3 5x2 2x + 24 to get

x3 5x2 2x + 24 = (x + 2)(x2 7x + 12).

Let f(x) = x2 7x + 12. Then

f(3) = 32 73+12 = 921+12 = 0, and so x3 is a factor of f(x) = x2 7x+12.

Divide x 3 into f(x) = x2 7x + 12 to get x2 7x + 12 = (x 3)(x 4). It now


follows that

p(x) = x4 4x3 7x2 + 22x + 24 = (x + 1)(x + 2)(x 3)(x 4)

and the roots of the equation p(x) = 0 are 1, 2, 3, 4.

2. Factorise f(x) = x4 2x3 12x2 + 40x 32 completely.

124
Soln: The divisors of the constant tern 32 are 1, 2, 4, 8, 16, 32. Now,

f(1) = 14 2 13 12 12 + 40 1 32 = 1 2 12 + 40 32 = 5 6= 0
and so x 1 is not a factor of f(x),

f(1) = (1)4 2 (1)3 12 (1)2 + 40 (1) 32 = 1 + 2 12 40 32 6= 0


and so x + 1 is not a factor of f(x),

f(2) = 24 2 23 12 22 + 40 2 32 = 16 16 48 + 80 32 = 0
and so x 2 is a factor of f(x).

Divide x 2 into f(x) = x4 2x3 12x2 + 40x 32 to get

x4 2x3 12x2 + 40x 32 = (x 2)(x3 12x + 16).

Let g(x) = x3 12x + 16. Then

g(2) = (2)3 12(2) + 16 = 8 + 24 + 16 6= 0 and so x + 2 is not a factor of g(x),


g(4) = 43 12(4) + 16 6= 0 and so x 4 is not a factor of g(x),

g(4) = (4)3 12(4) + 16 = 64 + 48 + 16 = 0 and so x + 4 is a factor of g(x).


Divide x + 4 into x3 12x + 16 we get

x3 12x + 16 = (x + 4)(x2 4x + 4).

The polynomial x2 4x + 4 factorises into (x 2)2 . It now follows that

f(x) = x4 2x3 12x2 + 40x 32 = (x + 4)(x 2)3 .

Exercise 6.14. 1. Find the real zeroes for the following polynomials.

(a) a(x) = 2x4 + 17x3 + 28x2 17x 30.


1 1 3
(b) b(x) = x3 + x2 2x + .
3 6 2
(c) c(x) = 2x3 + x2 13x + 6.
(d) d(x) = 4x4 + 4x3 25x2 x + 6.

2. Factorise completely the following polynomials.

(a) x3 6x2 + 11x 6.


(b) 2x3 + 13x2 + 22x + 8.
(c) x4 x3 5x2 + 3x + 6.
(d) 3x5 14x4 + 36x2 + 43x + 10.

3. Factorize p3 3p + 2 and hence find the factors of (x + y)3 3(x + y + 2) + 8.

4. Find p if k + 2 is a factor of k 50 p25 .

125
6.5 Inequalities

In this section we use the sign tables to solve inequalities.

Examples 6.15. 1. Solve the inequality x3 + 2x2 15x < 0.

Soln: x3 + 2x2 15x < 0 x(x2 + 2x 15) < 0 x(x + 5)(x 3) < 0.
We use the sign table to find the values of x for which this inequality holds:

5 0 3
x+5 0 + + +
x 0 + +
x3 0 +
x(x + 5)(x 3) 0 + 0 0 +

This inequality holds for all values of x such that x < 5 or 0 < x < 3.
2 1
2. Solve the inequality .
x3 x+2
2 1 2 1 x+7
Soln: 0 0.
x3 x+2 x3 x+2 (x 3)(x + 2)

7 2 3
x+7 0 + + +
x+2 0 + +
x3 0 +
x+7
+ UN UN +
(x 3)(x + 2)

So this inequality holds for all values of x such that 7 x < 2 or x > 3.

x 2x x
3. Solve the inequality + .
x2 +x2 x+2 x1
Soln:
x 2x x
+
x2 +x2 x+2 x1
x 2x x
+ 0
(x + 2)(x 1) x + 2 x 1
x + 2x(x 1) x(x + 2)
0
(x + 2)(x 1)
x2 3x x(x 3)
0 0.
(x + 2)(x 1) (x + 2)(x 1)

126
2 0 1 3
x+2 0 + + + +
x 0 + + +
x1 0 + +
x3 0 +
x(x 3)
+ UN 0 + UN 0 +
(x + 2)(x 1)

So this inequality holds for all values of x such that 2 x 0 or 1 < x 3.

4. Solve the inequality 2x3 5x2 23x 10 > 0.

Soln: Let a(x) = 2x3 5x2 23x 10. The divisors of the constant term 10 are
1, 2, 5, 10. Since
a(2) = 2(2)3 5(2)2 23(2)10 = 2(8)5423(2)10 = 1620+4610 = 0,
it follows that x + 2 is a factor of a(x) = 2x3 5x2 23x 10. Divide x + 2 into
2x3 5x2 23x 10 to get
2x3 5x2 23x 10 = (x + 2)(2x2 9x 5).
The factor 2x2 9x 5 factorises into (2x + 1)(x 5). Therefore
a(x) = 2x3 5x2 23x 10 = (x + 2)(2x + 1)(x 5).
It now follows that
2x3 5x2 23x 10 > 0 (x + 2)(2x + 1)(x 5) > 0.
We now use the sign table to find the values of x for which this inequality holds.

2 12 5
x+2 0 + + +
2x + 1 0 + +
x5 0 +
(x + 2)(2x + 1)(x 5) 0 + 0 0 +
1
So this inequality holds for all values of x such that 2 < x < or x > 5.
2

Exercise 6.16. Solve the following inequalities:

1. x3 2x2 5x + 6 0

2. x4 + 4x3 9x2 36x > 0


1 3 2
3. + >
4x2 1 2x + 1 2x 1
4. (x2 + 1)(x2 1)(x + 2) 0

127
Chapter 7

The Absolute Value (Modulus)


Function

Definition 7.1. The absolute value of a real number x, denoted by |x|, is defined by

x if x 0
|x| =

x if x < 0.

Note that |x| 0 for all x R. The absolute value |x| measures the distance between the
real number x and zero.

Examples 7.2. 1. |2| = 2 since 2 0.

| 3| = (3) = 3 since 3 < 0.

|0| = 0 since 0 0.


x1 if x 1 0 x 1 if x 1
2. |x 1| = =

(x 1) if x 1 < 0 x + 1 if x < 1.


2x + 1 if 2x + 1 0 2x + 1 if x 12
3. |2x + 1| = =

(2x + 1) if 2x + 1 < 0 2x 1 if x < 12 .

4. Write each of the following without the absolute value sign(s).

(a) |4| + | 7| (b) |4 7| (c) |x 3|, if x < 3 (d) |3x + 6|, if x < 4.

Soln: (a) |4| + | 7| = 4 + 7 = 11

128
(b) |4 7| = | 3| = 3
(c) If x < 3, then |x 3| = (x 3) = x + 3.
(d) If x < 4, then |3x + 6| = (3x + 6) = 3x 6.

5. a < b < c, write |a b| + |b c| without using the absolute value signs.

Soln: If a < b then a b < 0, and if b < c then b c < 0. Therefore

|a b| + |b c| = (a b) + [(b c)] = b a b + c = c a.

7.1 Absolute-value Equations and Inequalities

Let a be a positive real number and let x be any real number. Then

1. |x| = a if and only if x = a or x = a.

2. |x| < a if and only if x < a (if x 0) or x < a (if x < 0), if and only if x < a or
x > a. Therefore

|x| < a if and only if a < x < a.

Similarly,

|x| a if and only if a x a.

3. |x| > a if and only of x > a (if x 0) or x > a (if x < 0), if and only if x > a or
x < a. That is,

|x| > a if and only if x > a or x < a.

Similarly,

|x| a if and only if x a or x a.

Examples 7.3. 1. |x| = 4 x = 4 or x = 4

2. |x| 3 3 x 3

3. |x| > 2 x > 2 or x < 2

129
4. |2x 1| = 5 2x 1 = 5 or 2x 1 = 5 x = 3 or x = 2.

5. Solve the equation |x 1| = 12 x + 1.

Soln: Recall that



x 1 if x 1
|x 1| =

x + 1 if x < 1.
Now, if x 1, then
1 1
|x 1| = x + 1 x1 = x+1
2 2
2x 2 = x + 2
x = 4.

That is, x 1 and x = 4 and so x = 4.

If x < 1, then
1 1
|x 1| = x + 1 (x 1) = x + 1
2 2
2x + 2 = x + 2
3x = 0 x = 0.

That is x < 1 and x = 0, and so x = 0.

It now follows that x = 0 and x = 4 are solutions of the equation |x 1| = 21 x + 1.

6. Solve the equation |x 1| = 2x + 1.

Soln: Recall that



x 1 if x 1
|x 1| =

x + 1 if x < 1.
Now, if x 1, then

|x 1| = 2x + 1 x 1 = 2x + 1
x = 2
x = 2.

That is, x 1 and x = 2. There is no solution.

If x < 1, then

|x 1| = 2x + 1 (x 1) = 2x + 1
x + 1 = 2x + 1
3x = 0 x = 0.

That is x < 1 and x = 0, and so x = 0.

130
It now follows that x = 0 is the only solution of the equation |x 1| = 2x + 1.
x5
7. Solve the equation |x 1| = .
3
Soln: Recall that

x 1 if x 1
|x 1| =

x + 1 if x < 1.
Now, if x 1, then
x5 x5
|x 1| = x1 =
3 3
3x 3 = x 5
2x = 2 x = 1.

That is, x 1 and x = 1. There is no solution.

If x < 1, then
x5 x5
|x 1| = (x 1) =
3 3
3x + 3 = x 5
4x = 8 x = 2.

That is, x < 1 and x = 2. There is no solution.


x5
It now follows that the equation |x 1| = has no solution.
3
8. Solve the equation |x 1| = x.

Soln: Recall that



x 1 if x 1
|x 1| =

x + 1 if x < 1.
Now, if x 1, then

|x 1| = x x1 =x
1 = 0,

which is impossible.

If x < 1, then

|x 1| = x (x 1) = x
x + 1 = x
1
2x = 1 x = .
2
That is x < 1 and x = 12 , and so x = 12 .

131
1
It now follows that x = 2
is the only solution of the equation |x 1| = x.

9. Solve the equation |3x 2| = |x + 3|.

Soln:
|3x 2| = |x + 4| 3x 2 = x + 4 OR 3x 2 = (x + 4)
3x 2 = x + 4 OR 3x 2 = x 4
2x = 6 OR 4x = 2
x=3 OR x = 21 .

10. Solve the inequality |x 1| > 3x.

Soln: By definition of the absolute value



x 1 if x 1
|x 1| =

x + 1 if x < 1.

Now, if x 1, then

|x 1| > 3x x 1 > 3x
1
2x < 1 x < .
2
That is, x 1 and x < 1 2
. There is no solution; i.e., there is no real number x that
is simultaneously greater or equal to one and less than 1
2
.

If x < 1, then

|x 1| > 3x (x 1) > 3x
x + 1 > 3x
1
4x < 1 x < .
4
That is, x < 1 and x < 41 , and so x < 14 .

It follows that the solution to the inequality |x 1| > 3x is the interval x < 14 .

11. Solve the inequality |2x + 3| < 6.

Soln:
9 3
|2x + 3| < 6 6 < 2x + 3 < 6 9 < 2x < 3 < x < .
2 2
The solution to the inequality |2x + 3| < 6 is the interval 29 < x < 32 .

132
7.2 The functions y = a|x p| + q

An absolute value graph can be presented as y = a|x p| + q, where the coordinates of the
turning point are (p, q).

Examples 7.4. Draw the graphs of the following functions.

1. y = |x|

2. y = |x| + 1

3. y = |x 1|

4. y = |x 1| 2

5. y = 2|x + 2| 1

6. y = 3|x 1| + 2

7. y = 12 |x + 1| + 3

8. y = |2x 1| 1

9. y = |x + 1| + |x + 1|

Solution

1. Let y = |x|. If x 0, then y = x. If x < 0, then y = x.

y
6

@
@
|
|x

@
=

@
y

@
@
@ -
0 x

2. Let y = |x| + 1. Start with the graph of y = |x| and move it up by 1 unit.

133
y
6

1
@

|+
@

|x
@

=
@ @

y
@ @

|
|x
@ @

=
@ @

y
@ 1
@
@ -
0 x

3. Let y = |x 1|. Start with the graph of y = |x| and move it to the right by 1 unit.

y
6
1|

@
@ @
@

|
|x

|x

@ @
=

@ @
y

@ 1@
@ @
@ @ -
0 1 x

4. Let y = |x 1| 2. Start with the graph of y = |x|. Move it 1 unit to th right and
then move the resulting graph downward by 2 units.

y
6
2

@
1|

@
|x

@
=

@.
y

. . .-
1@ 0 1 2 3 x
@
@
1@
@
2 @(1, 2)

134
5. Let y = 2|x + 2| 1. Start with the graph of y = |x|. Move it 2 unit to left. Make the
slope of the grapher steeper. Move the resulting graph downward by 1 unit.

y
A 6 
A 
A 
A 
A 3
A 

1
2|
A 
A 
A 

+
A 
A 2|x 
y=
A 
. A .  . . -
3 A2  1 0 x
A 
A 1
(2, 1)

T P (2, 1)
x int : y = 0
2|x + 2| 1 = 0
|x + 2| = 12 x + 2 = 1
2
or x + 2 = 12
x = 23 or x = 52
y int : x = 0
y = 2|2| 1 = 4 1 = 3

6. Let y = 3|x 1| + 2.

y
6

(1, 2)
2
B
 B
 B
 B
 B
.  . B . .-
1 0  1 B2 3 x
 B
y=

1 B
 B
3|x

2  B
 B
 B
1|

 B
 B
+2

 B
 B

T P (1, 2)
x int : y = 0
3|x 1| + 2 = 0

135
|x 1| = 23 x 1 = 23 or x 1 = 23
x = 1 32 or x = 13
y int : x = 0
y = 3| 1| + 2 = 3 1 + 2 = 1

7. Let y = 21 |x + 1| + 3.

y
6

(1, 3)
H 3
:
:
 : H y= :
 : H
 : HH 2 1|
 H
 :
: H x+1
 HH |
 :
:: H +3
 H -
 H

 7 1 0 5 HH
H
x

T P (1, 3)
x int : y = 0
12 |x + 1| + 3 = 0
|x + 1| = 6 x + 1 = 6 or x + 1 = 6
x = 5 or x = 7
y int : x = 0
y = 12 |1| + 3 = 12 1 + 3 = 2 21

8. Let y = |2x 1| 1.

y 6

. . . .-
1 0 1 2 3 x
1
1 (
A2
, 1)
 A
2  A
y=

 A
 A
|2

 A
 A
x

 A
1|

 A
 A
 AA
1

T P ( 12 , 1)

136
xint : y = 0
|2x 1| 1 < 0 always
no x int
y int : x = 0
y = | 1| 1 = 2

9. Let y = |x + 1| + |x 1|.
Case 1 : x < 1 Case 2 : 1 x < 1 Case 3 : x 1
y = (x + 1) (x 1) y = (x + 1) (x 1) y = x+1+x1
y = 2x y=2 y = 2x

y6
A 
A 
A 
2x
y=

A 
A 
y=
2

A 
A 
x

A 
A 2 

. . . .-
1 0 1 2 3 x
1

7.3 Algebraic and graphical solution of equations and


inequalities

In this section we use algebraic and graphical methods to solve equations and inequalities that
involve absolute values.

Examples 7.5. 1. Solve the equation |x 3| = 2x + 1.

Algebraic Solution: By definition of the absolute value



x 3 if x 3
|x 3| =

x + 3 if x < 3.

Now, if x 3, then

|x 3| = 2x + 1 x 3 = 2x + 1
x = 4.

Hence x 3 and x = 4. There is no real number x that can satisfy these conditions.

137
If x < 3, then

|x 3| = 2x + 1 (x 3) = 2x + 1
x + 3 = 2x + 1
2
3x = 2 x = .
3
That is, x < 3 and x = 32 , and so x = 23 .
2
It follows that x = 3
is the only solution of the equation |x 3| = 2x + 1.

Geometric solution:

y6 
1


+


2x

@
@ 
y=

@ 

3|
3@ 


@ 
|x
:
@ =
2  :: @
y

 : @
 :: @
1 : @
 : @
.  :: . . @. -
1  0 2
1 2 3 x
3

 1


2. Solve the inequality |2x 4| 2.

Algebraic solution:

|2x 4| 2 2 2x 4 2 2 2x 6 1 x 3.

Geometric solution:

AAy 6 
|

A 
4

4A 
|2x

A 
A 
y=

3 A 
A 
A 
2 A: :
:A :
: A  ::
1 : A  :
:
: A  :
. : A. : -
.: :. x
1 0 1 2 3
1 138
3. Solve the inequality |x 2| < 1 2x.

Algebraic Solution: By the definition of the absolute value



x 2 if x 2
|x 2| =

x + 2 if x < 2.

If x 2, then

|x 2| < 1 2x x 2 < 1 2x
3x < 3 x < 1.

Hence x 2 and x < 1. There is no real number x that can satisfy these conditions.

If x < 2, then

|x 2| < 1 2x (x 2) < 1 2x
x + 2 < 1 2x
x < 1.

That is, x < 2 and x < 1, and so x < 1.

It follows that x < 1 is the solution for the inequality |x 2| < 1 2x.

Geometric Solution:

AA
y6
A
@ A 4
@ A
@A
2|

@: 3

::
::
:

:A@
|x

:A
: A@
=

:
y

: A 2@
:
: A @
@
:
: 1A @
: A
A @
: @. -
: A . .
x
1 0 A 1 2 3
A
1 A
A y = 1 2x
A
A

7.4 Geometric representation of Absolute Value

When we measure distance we require the answer to be non-negative irrespective of the


direction involved. Suppose c is a fixed point on a number line and x is a variable point.

139
(a) If x is to the right of c, then the distance from c to x is x c.
. . -
c x (c < x)

(b) If x coincides with c, then the distance from c to x is x c = c c = 0.


. -
x=c (x = c)

(c) If x is to the left of c, then the distance from c to x is c x = (x c).


. . -
x c (x < c)

We can summarise this:



x c if x c 0
the distance from c to x =

(x c) if x c < 0
which is saying the same as saying the distance from c to x is |x c|.

7.5 Miscellaneous Examples

Examples 7.6. 1. Solve the following equations:

(a) x3 + |x| = 0.
(b) x3 + x2 + |x + 1| = 0.

Soln: (a) Case 1: x 0. In this case,

140
x3 + |x| = 0 x3 + x = 0
x(x2 + 1) = 0
x = 0 since x2 + 1 > 0 for all x R.

Case 2: x < 0. In this case,


x3 + |x| = 0 x3 x = 0
x(x2 1) = 0
x(x + 1)(x 1) = 0
x = 0, x = 1 OR x = 1.
Since x < 0, we discard the solutions x = 0 and x = 1.
Therefore x = 0 (from the first case) and x = 1 (from the second case) are the
only solutions of the equation x3 + |x| = 0.
(b) Case 1: x + 1 0 x 1. In this case,
x3 + x2 + |x + 1| = 0 x3 + x2 + x + 1 = 0
x2(x + 1) + (x + 1) = 0
(x + 1)(x2 + 1) = 0
x = 1 since x2 + 1 > 0 for all x R.

Case 2: x + 1 < 0 x < 1. In this case,


x3 + x2 + |x + 1| = 0 x3 + x2 (x + 1) = 0
x3 + x2 x 1 = 0
x2(x + 1) (x + 1) = 0
(x + 1)(x2 1) = 0
(x + 1)2 (x 1) = 0
x = 1 OR x = 1.

Since x < 1, neither x = 1 nor x = 1 meets this condition.


It follows that x = 1 obtained in the first case is the only solution for the equation
x3 + x2 + |x + 1| = 0.

2. Solve the following system of equations:



|x| + |y| = 4

xy = 3.

Soln: Since xy = 3 < 0, we have that either x > 0 and y < 0 or x < 0 and y > 0.

Case 1: x > 0 and y < 0. In this case

|x| + |y| = 4 x + (y) = 4 x y = 4.

We get the system of equations:



xy = 4 . . . . . . . . . (1)

xy = 3 . . . . . . . . . (2)

141
From equation (1), we have that x = 4 + y. Substituting this in equation (2), we get
that

xy = 3 (4 + y)y = 3 y 2 + 4y + 3 = 0
(y + 3)(y + 1) = 0
y = 3 OR y = 1.

The corresponding x values are: x = 4 3 = 1 and x = 4 1 = 3. We get solutions


(1, 3) and (3, 1).

Case 2: x < 0 and y > 0. In this case

|x| + |y| = 4 x + y = 4.

We get the system of equations:



x + y = 4 . . . . . . . . . (3)

xy = 3 . . . . . . . . . (4)

From equation (3), we have that x = y 4. Substituting this in equation (4), we get
that

xy = 3 (y 4)y = 3 y 2 4y + 3 = 0
(y 3)(y 1) = 0
y = 3 OR y = 1.

The corresponding x values are: x = 3 4 = 1 and x = 1 4 = 3. We get solutions


(1, 3) and (3, 1).
Therefore the system of equations

|x| + |y| = 4

xy = 3

has four solutions:

(1, 3); (3, 1); (1, 3); (3, 1).

3. Draw the graphs of the following functions:

(a) y = x|x| + 1.
(b) y = |x2 4|.
(c) y = |x2 x 6|.

142
Soln: (a) If x 0, then y = x|x| + 1 y = x2 + 1 and,
if x < 0, then y = x|x| + 1 y = x(x) + 1 = x2 + 1.

6
y

-
1 0 x

(b)

x2 4 if x2 4 0 x2 4 if x 2 or x 2
y = |x2 4| = =
2 2
x + 4 if x 4 < 0 x2 + 4 if 2 < x < 2.

y6

-
2 0 2 x

(c)

x2 x 6 if x2 x 6 0
2
y = |x x 6| =

x2 + x + 6 if x2 x 6 < 0


x2 x 6 if x 2 or x 3
=

x2 + x + 6 if 2 < x < 3.

143
y6

-
2 0 3 x

Exercise 7.7. 1. Simplify the following expressions.


1
(a) 2|x 1| 3|2x 1| + |x + 3| if 3 < x < .
2
|x|
(b) |x + 1| + 3|x 1| 2|x 2| if 0 < x < 1.
x
(c) x + |1 x| + 2|x 2| if 1 < x < 2.

2. Solve the following equations:

(a) |4x 2| = 4.
(b) | 2x + 3| = 1.
(c) |x + 3| = 2x.
(d) 3|2x 4| = x 2.
(e) |x + 2| + |x 2| = 4.

3. Solve the following inequalities:

(a) | 3x + 2| 5
(b) | x + 1| > 7
(c) |2x 1| x
|x 1|
(d) <1
|x + 1|
(e) |2x 4| < 1 x

4. Sketch the graphs of the following functions:

(a) y = |2x + 4|
(b) y = |x| 2
|x|
(c) y =
x
(d) y = 2|x + 1| + 3
(e) y = 2|x| |x + 1|

144
5. Solve the following equations and inequalities algebraically and graphically

(a) |x 1| = x + 2
(b) |1 2x| < 3
(c) 2 < |x 1| 3
1
(d) |x + 2| x + 3
2
6. Solve the following equations:

(a) x|x 1| x 1 = 0
(b) x2 3|x| + 2 = 0

7. Draw the graphs of the following functions:

(a) y = x|x| 2x + 3
(b) y = |x2 1|
(c) y = 2|x2 + 1| |x| 1
1
(d) y = | x2 + 2|
2

145
Chapter 8

Trigonometry

8.1 The radian measure

Angles can be measured in degrees or in radians (abbreviated as rad). We will no longer refer
to angles in terms of degrees; instead we use the radian measure. Consider a circle centred
at O and with radius r.


 s
r


O r

We see a sector of a circle with central angle and radius r subtending an arc with length s.
The radian measure of the angle is
s
= (8.1)
r

The radian measure of an angle is the ratio of the arc-length to the radius of a circle.

In particular, when s = r, we see that an angle of 1rad is the angle subtended at the centre
of a circle by an arc equal in length to the radius of the circle.
The length of the circumference of a circle of radius r is 2r. It follows that the radian
measure corresponding to 360 is

2r
= = 2.
r
That is, 360 = 2 rad.

146
We can get some conversions: 360 = 2rad = 180 = rad
 
180
1 rad = 57, 3 (8.2)


1 = rad 0, 017rad. (8.3)
180

From equations (8.2) and (8.3) we see that to convert from degrees to radians we multiply by
180
and to convert from radians to degrees we multiply by .
180


To convert from degrees to radians, multiply the angle by
180

180
To convert from radians to degrees, multiply the angle by .

Examples 8.1. 1. (a) Find the radian measure of 30 .


3
(b) Express rad in degrees.
2
 
Soln: (a) 30 = 30

= rad
180 6
 
3 3 180
(b) rad = = 270
2 2
2. (a) If the radius of a circle is 10cm, what angle is subtended by an arc of length 25cm?
(b) If a circle has radius 5cm, what is the length of an arc subtended by a central angle
3
of rad?
10
Soln: (a) Let s = 25cm and r = 10cm. Using equation (8.1)

s 25
= = = 2, 5rad.
r 10
(b) From equation (8.1), we have that
 
s 3 3
= s = r = 5 = cm
r 10 2

Remarks 8.2. 1. The radian measure of an angle is the ratio of two lengths (measured
in the same units) so it is a pure number, with no units. It is for this reason that we
shall henceforth write the radian measure of an angle without any units.
arc length
2. The definition depends on the fact that if the angle is fixed, then the ratio
radius
remains the same, irrespective of the length of the radius.

147
It would be very useful to memorise some conversions:


0 = 0 30 =
6

45 = 60 =
4 3
2
90 = 120 =
2 3
3
180 = 270 =
2

360 = 2

8.2 Trigonometric ratios in a right-angled triangle



Consider a right-angled triangle with the angle A = .
2

B


a c

b

C A

We can define six trigonometric ratios: sine, cosine, tangent, cotangent, cosecant and secant
(abbreviated as sin, cos, tan, cot, cosec and sec, respectively.)

c b
sin = cot =
a c
b a
cos = cosec =
a c
c a
tan = sec =
b b

Where c is the opposite side to , b is the adjacent side to , a is the hypotenuse.



Note that + = since the sum of angles in a triangle is . From the above definitions
2

148

and the equation + = we can get the following identities:
2


sin( ) = cos cot( ) = tan
2 2

cos( ) = sin cosec ( ) = sec
2 2

tan( ) = cot sec ( ) = cosec
2 2
Remarks 8.3. 1. It is important to remember that the trigonometric ratios depend only
on the angle .

2. Sine and cosine are called each other co-functions as are tangent and cotangent, secant
and cosecant.

The exact trigonometric ratios for certain angles can be read from the given triangles:



4
2 1

4

1

1 1
sin = cos = tan =1
4 2 4 2 4

cosec = 2 sec = 2 cot =1
4 4 4




 3

2  1




 6

3

149

1 3 1
sin = cos = tan =
6 2 6 2 6 3
2
cosec =2 sec = cot = 3
6 6 3 6


3 1
sin = cos = tan = 3
3 2 3 2 3
2 1
cosec = sec =2 cot =
3 3 3 3 3
1
Examples 8.4. 1. If cos = 0<< , find the other five trigonometric functions
5 2
of .
1
Soln: Since cos = , we can label hypotenuse as having length 5 units and the adjacent side
5
as having length 1 unit. If the opposite side has length y, then the Pythagoras Theorem

gives y 2 + 1 = 25 and so y = 24.





5  y= 24






1

From our diagram we get



24 1
sin = cos = tan = 24
5 5
5 1
cosec = sec = 5 cot =
24 24
2. Let ABC be an equilateral triangle with sides a. Find the height and the area of the
triangle in terms of a.

Soln:

150
A
SS
 S
 S
 S
a h S a
 S
 S
 S
 S
 S
 3 3 S
B C

h
Since sin = , it follows that
3 a

a 3
h = a sin = .
3 2

1 1 a 3 a2 3
Area(4ABC) = base x height = a = .
2 2 2 4
3. From two observation points on opposite sides of a tower, y metres high and in the
same horizontal plane as the foot of the tower, the angles of elevation of the top of the
tower are and .

(a) Determine, in terms of y, and , the distance between the two observation points.

(b) Calculate this distance if y = 30m, = and = .
3 6
Soln:

C
c
 cc
 c
 c
Observation Tower

 c
 c
y c
 c
 c
 c
 c
c
 c
 c
A D B

AD
(a) In 4ADC, = cot , and so AD = y cot .
y
DB
In 4CDB, = cot , and so DB = y cot .
y
Therefore distance between the two observion points A and B is

AD + DB = y cot + y cot = y(cot + cot ).

151
(b)
 
AD + DB = AB = 30 cot + cot
 3 6
1
= 30 + 3
3
4 3
= 30
3
= 40 3 m.
4. Let P QRS be a rhombus with sides 20cm. Calculate the length of each diagonal if
b = .
P QR
3
Soln:

P
H
 HH
 H
 H
 HH
 H
 H
 HH
 H
H 
Q HH 6 O   S
H 
H 
HH
H 
H 
HH 
H
R

QO
In 4OQR, = cos . Therefore
QR 6

3
QO = QR cos = 20 = 10 3 cm.
6 2
OR
Also, = sin and so
QR 6
1
OR = QR sin = 20 = 10 cm.
6 2

Therefore the diagonals QS and P R are 20 3 cm and 20 cm respectively.

5. From a window in a building on one side of a street, 30 metres wide, the angle of

depression of the bottom of the building across the street is and the angle of elevation
4

of the top of the building across the street is . Find
6
(a) the height of the observer above street level and
(b) the height of the building being observed.

Soln:

152
A
H
H
H
H
HH
H
H
H
H
H
6 H
B
E
4
H


4
C 30 m D

h
(a) In 4CDE, = tan . Therefore h = 30 tan = 30 1 = 30 m.
30 4 4
AB
(b) In 4ABE, = tan . Therefore
30 6

1 3
AB = 30 tan = 30 = 30 = 10 3 m.
6 3 3

Thus, H = h + AB = 30 m + 10 3 m = 10(3 + 3) m.

8.3 Trigonometric function of any angle

The above definitions do not apply to obtuse or negative angles. For a general angle in
standard position, we let P (x, y) be any point on the terminal side of the angle and we let
r be the distance OP .
y
6

P (x, y)

@
@
@ r
@
@
@
@
@ -
0 x

153
We define the trigonometric functions as follows:

y x
sin = cot = ; y 6= 0
r y
x r
cos = cosec = ; y 6= 0
r y
y r
tan = , x 6= 0 sec = ; x 6= 0.
x x

The signs of the trigonometric functions for angles in each of the four quadrants depend on
the signs of the co-ordinates x and y.

y
6
SECOND QUADRANT FIRST QUADRANT

Only sin and cosec All trig functions are


are positive positive
-
x
Only tan and cot Only cos and sec are
are positive positive

THIRD QUADRANT FOURTH QUADRANT

Examples 8.5. 1. In which sub-intervals of the interval (0, 2) do the angles mentioned
below lie, if:

(a) sin > 0 and cos < 0


(b) cosec > 0 and cos < 0
(c) sec < 0 and tan > 0
(d) cot < 0 and sin < 0

Soln: (a) Recall the definitions of sin and cos  


y x y x
sin = , cos = so > 0 and < 0 = ,
r r r r 2
r x r x
(b) Since cosec = and cos = , it follows that > 0 and < 0. Therefore
  y r y r
, .
2

154
r y
(c) Since sec < 0 and tan > 0, and sec = and tan = , it follows that
  x x
r y 3
< 0 and > 0 and so , .
x x 2
(d) cot < 0 and sin < 0  
x y x y 3
cot = and sin = so < 0 and < 0 = , 2
y r y r 2
2. The sign of sec is the same as that of cosec but opposite to that of tan . In which
sub-interval of the interval (0, 2) does lie?
 
3
Soln: Since sec < 0 and cosec < 0 but tan > 0, we must have that , .
2
3. Calculate the values of sin and cot if the terminal side of in standard position

passes through (3, 2).

Soln:

y
6

Q -
:: 
::  O x
:: 

::  r
: 
P (3, 2)

y x
Recall that sin = and cot = .
r y
b =
We construct 4P OQ with OQP . Using the Pythagoras Theorem,
2


P Q2 + OQ2 = OP 2 | 2| + | 3|2 = r2
= r2 = 11

= r = 11.
r
2 2 3 3
Thus sin = = and cot = = .
11 11 2 2

155
 
3
4. Determine cos if sin = and , .
2 2
Soln:
y
6

P (x, 3)
:
::AA
: A
:: A
:: A r
:: A
:: A
:: A
: A -
Q O x


3
If sin = , then y = 3 and r = 2. In 4P QO,
2
 2
P Q2 + QO2 = P O2 3 + |x|2 = 22 3 + x2 = 4.
 
Therefore x2 = 1. Since , , we must have that x = 1. Thus,
2
x 1
cos = = .
r 2

8.4 Trigonometric Identities

From the definitions mentioned in section 8.3, we can easily deduce the following fundamental
identities:

sin
1. tan = if cos 6= 0 or 6= + k , where k is an integer.
cos 2
cos
2. cot = if sin 6= 0 or 6= k, where k is an integer.
sin
1
3. sec = if 6= + k , where k is an integer.
cos 2
1
4. cosec = if 6= k , where k is an integer.
sin
5. sin2 + cos2 = 1.

6. 1 + tan2 = sec2 .

156
7. 1 + cot2 = cosec2 .

Identity (5) is a different form of Pythagoras Theorem. From these identities we can derive
by means of the algebraic operations, new identities.

In Algebra there is distinction between an identity and a conditional equation. The latter holds
only for a limited number of values of the variable, while the former is true for all values. For
example, cos2 = sin2 is an equation, for it is true only for angles of a certain group of

which the angle is one.
4
When proving Trigonometric Identities, it is advisable to write the trigonometric functions
involved in terms of sine and cosine functions.

Examples 8.6. 1. Prove that sec2 + cosec2 = sec2 cosec2

Proof:
1 1
sec2 + cosec2 = +
cos2 sin2

sin2 + cos2
=
cos2 sin2
1
=
cos2 sin2
1 1
=
cos sin2
2

= sec2 cosec2 .

tan (cot + 1) cos + sin


2. Prove that = .
1 tan cos sin
Soln:
tan(cot + 1) 1 + tan
=
1 tan 1 tan
sin
1+
cos
=
sin
1
cos
cos + sin
cos
=
cos sin
cos
cos + sin
= .
cos sin

157
3. Prove that sin4 + cos4 = 1 2 sin2 cos2 .

Soln:

sin4 + cos4 = sin4 + cos4 + 2 sin2 cos2 2 sin2 cos2

= (sin2 + cos2 )2 2 sin2 cos2

= 1 2 sin2 cos2 .
cos
4. Prove that sec tan = .
1 + sin
Soln:
1 sin
sec tan =
cos cos
1 sin 1 sin 1 + sin
= =
cos cos 1 + sin

1 sin2 cos2
=
cos (1 + sin ) cos (1 + sin )
cos
= .
1 + sin

Conclusions

1. It is easy to transform a complicated expression into a simple one.

2. A good knowledge of algebraic operations and identities is advantageous.

8.5 Periodic Relations

We say that a number T 6= 0 is a period of a function f(x) if f(x + T ) = f(x) is true for
all x. The function f(x) is called a periodic function. The smallest positive T is called
the fundamental period of the function f. The functions sin x, cos x, sec x and cosec x are
periodic with the fundamental period 2.


sin(x + 2k) = sin x







cos(x + 2k) = cos x
where k = 0 1, 2, .


sec(x + 2k) = sec x






cosec(x + 2k) = cosec x

158
The function tan x, cot x are periodic functions with the fundamental period . That is,

tan(x + k) = tan x
where k = 0, 1, 2, .

cot(x + k) = cot x

Examples 8.7. 1. Prove that f(A) = tan A has period .

Soln:

y6


(x, y)



+ 
AA -

 O x
 





(x, y)

y y
f(A + ) = tan(A + ) = = = tan A = f(A).
x x
2. Prove that f(A) = cos 2A has the period .

Soln: f(A + ) = cos[2(A + )] = cos(2A + 2) = cos 2A = f(A)


2
3. Show that if k is a nonzero integer, then f(A) = sin(kA) has the period .
k
Soln:
    
2 2
f A+ = sin k A + = sin(kA + 2) = sin(kA) = f(A).
k k
4. Show that if g(A) is a functions defined on [1, 1], then f(A) = g(sin A) has a period
2.

Soln: f(A + 2) = g[sin(A + 2)] = g(sin A) = f(A).

8.6 Parity relations

We recall the definitions of an even and odd functions:


A function f(x) is even if f(x) = f(x) for all x. The functions f(x) = cos x and g(x) =

159
sec x are even.
A function f(x) is odd if f(x) = f(x) for all x. The functions f(x) = sin x, g(x) =
tan x, h(x) = cot x and `(x) = cosec x are odd.

sin(x) = sin x
tan(x) = tan x
cot(x) = cot x
cosec(x) = cosec x.
Examples 8.8. 1. Prove that sin(A) = sin x and cos(A) = cos A.

Soln:

y6



(x, y)
 ::
r 

:
 :
 :
<

 A :
: -
H
O H A : x
HH :
:
<

H
H
r HH ::
H:
H
(x, y)

y y x
sin(A) = = = sin A and cos(A) = = cos A.
r r r
2. Prove that tan(A) = tan A.
y y 
Soln: tan(A) = = = tan A.
x x
3. Show that the function f(A) = sin2 A is even.

Soln: f(A) = sin2 (A) = [ sin(A)]2 = ( sin A)2 = sin2 A = f(A).

4. Prove that if g(A) is any function defined on [1, 1], then f(A) = g(cos A) is even.

Soln: f(A) = g(cos(A)) = g(cos A) = f(A).

8.7 Reduction Formulae

The purpose of the reduction formulae is to express the value of a trigonometric function of
any angle in terms of the value of the trigonometric function of the corresponding angle in
the first quadrant.

160
   
sin A = cos A sin + A = cos A
2 2
   
cos A = sin A cos + A = sin A
2 2
   
tan A = cot A tan + A = cot A
2 2
-
   
3 3
sin A = cos A sin +A = cos A
2 2
   
3 3
cos A = sin A cos +A = sin A
2 2
   
3 3
tan A = cot A tan +A = cot A
2 2

sin ( A) = sin A sin ( + A) = sin A

cos ( A) = cos A cos ( + A) = cos A

tan ( A) = tan A tan ( + A) = tan A

sin (2 A) = sin A

cos (2 A) = cos A

tan (2 A) = tan A

The formulae for cotangent are the same as for tangent. Similarly, secant as for cosine and
cosecant as for sine. It is important to remember that the angle 00A00 in the reduction formulae
can take any value.

161
 
Examples 8.9. 1. Simplify E if E = tan + A cot(+A)sec(2A) cos(A).
2
Soln:
 
E = tan + A cot( + A) sec(2 A) cos( A)
2
= ( cot A) cot A sec A( cos A)
= cot2 A + 1
= cosec2 A

2. Prove that


cos A if k = 4n






  sin A if k = 4n + 1
cos k A =
2


cos A if k = 4n + 2





sin A if k = 4n + 3
where n is an integer.
Note that these are all the cases since k divided by 4 gives the remainders 0, 1, 2, 3.
 
Soln: If k = 4n then cos 4n A = cos (2n A) = cos(A) = cos A.
2
If k = 4n + 1 then
h ih  i  
cos (4n + 1) A = cos 2n + A = cos A = sin A.
2 2 2

If k = 4n + 2 then
h i
cos (4n + 2) A = cos [2n + ( A)] = cos ( A) = cos A.
2

If k = 4n + 3 then
h i    
3 3
cos (4n + 3) A = cos 2n + A = cos A = sin A.
2 2 2

162
8.8 More Trigonometric Identities

It is very useful to memorise the following identities:

Addition Formulae:
sin(A + B) = sin A cos B + cos A sin B

cos(A + B) = cos A cos B sin A sin B

tan A + tan B
tan(A + B) =
1 tan A tan B

Subtraction Formulae:
sin(A B) = sin A cos B cos A sin B

cos(A B) = cos A cos B + sin A sin B

tan A tan B
tan(A B) =
1 + tan A tan B
Double Angle: You must know how these are derived from the addition formulae given above.

sin 2A = 2 sin A cos B

cos 2A = cos2 A sin2 A = 2 cos2 A 1 = 1 2 sin2 A

2 tan A
tan 2A =
1 tan2 A
Half Angle:

A 1 cos A
sin2 =
2 2
A 1 + cos A
cos2 =
2 2
A 1 cos A
tan2 =
2 1 + cos A

163
The sum and the difference of the two sines or cosines:
A+B AB
sin A + sin B = 2 sin cos
2 2
A+B AB
sin A sin B = 2 cos sin
2 2
A+B AB
cos A + cos B = 2 cos cos
2 2
A+B AB
cos A cos B = 2 sin sin
2 2
sin(A + B)
Examples 8.10. 1. Prove that tan A + tan B = .
cos A cos B
Soln:
sin A sin B
tan A + tan B = +
cos A cos B
sin A cos B + cos A sin B
= .
cos A cos B
sin(A + B)
=
cos A cos B
2. Prove that sin 3A = 3 sin A 4 sin3 A.

Soln:
sin 3A = sin(2A + A)
= sin 2A cos A + cos 2A sin A
= 2 sin A cos2 A + (1 2 sin2 A) sin A
= 2 sin A(1 sin2 A) + sin A 2 sin3 A
= 3 sin A 4 sin3 A
sin 7A + sin 5A
3. Prove that = cot A.
cos 5A cos 7A
Soln:
7A + 5A 7A 5A
sin 7A + sin 5A 2 sin
cos
= 2 2
cos 5A cos 7A 5A + 7A 5A 7A
2 sin sin
2 2
sin 6A cos A
=
sin 6A sin(A)

cos A
=
sin(A)

= cot A

164
A 1 cos A
4. Prove that tan = .
2 sin A
  A
A 2 tan
Soln: tan A = tan 2 = 2
2 A
1 tan2
2
A
Let tan = x.
2
sin A 2x 2 cos A + 2 1 cos A
We get = 2
x= =
cos A 1x 2 sin A sin A
2 cos A 2
(we cannot accept ).
2 sin A

Conditional identities

If A, B and C are angles of a triangle then A+B+C = . The relations between trigonometric
functions of three angles in a triangle are important.

1. If A + B + C = , prove that tan A + tan B + tan C = tan A tan B tan C.

Soln: tan(A + B) = tan( C) = tan C but


tan A + tan B
tan(A + B) = = tan C
1 tan A tan B
we get tan A + tan B = tan C(1 tan A tan B).Thus tan A + tan B = tan C +
tan A tan B tan C
tan A + tan B + tan C = tan A tan B tan C.

165
A B C
2. If A + B + C = , prove that sin A + sin B + sin C = 4 cos cos cos .
2 2 2
Soln:
A+B AB
sin A + sin B + sin C = 2 sin cos + sin C
2 2
C AB C
= 2 sin cos + sin 2
2 2 2
 
C AB C C
= 2 sin cos + 2 sin cos
2 2 2 2 2

C AB C C
= 2 cos cos + 2 sin cos
2 2 2 2
 
C AB C
= 2 cos cos + sin
2 2 2
 
C AB A+B
= 2 cos cos + cos
2 2 2

C A B
= 2 cos 2 cos cos
2 2 2
A B C
= 4 cos cos cos
2 2 2

8.9 Graphs of Trigonometric Functions

Below are the graphs of the functions y = sin x, y = cos x and y = tan x. The graphs of the
functions , and

1. y = sin x

y y = sin x
1

3
2
2 0

2 3
2 2 5
2 3 x
1

166
2. y = cos x

y y = cos x
1

3
2
2 0

2 3
2 2 5
2 3 x
1

3. y = tan x
y
y = tan x

3 2 0
3
2 1 2 2 x

Note that for both the sine and cosine functions the domain is (, ) and the range is the
closed interval [1, 1]. Thus, for all values of x, we have

1 sin x 1 and 1 cos x 1.

The important property of the sine and cosine functions is that they are periodic functions
with the fundamental period 2. This means that for all values of x

sin(x + 2) = sin x and cos(x + 2) = cos x.

The periodic nature of these functions makes them suitable for modeling periodic phenomena
such as tides, vibrating strings and sound waves. The tangent function is related to the sine
and cosine functions by the equation.

sin x
tan x = .
cos x

167

It is undefined when x = + k, k = 0, 1, 2, . . .. Its range is (, ). Notice that
2
the tangent functions has the fundamental period :

tan(x + ) = tan x for all x.

8.9.1 Some important transformations which are applied to the


trigonometric functions

1. y = af(x), a R.
y
(a) y = a sin x y = 3 sin x
3
y = 2 sin x
2

2 0

2 3
2 2 x
1
y = sin x
2

(b) y = a tan x

y = tan x

1 y = 2 tan x

x
2 0
2
1 y = 21 tan x

168
2. y = f(kx), k > 0.

(a) y = sin(kx)

y y = sin x
1

2 3
2
2 0

2 3
2 2 5
2 3 x
1

y y = sin 12 x
1

2 3
2
2 0

2 3
2 2 5
2 3 x
1

y y = sin 2x
1

2 3
2 2 0

2 3
2 2 5
2 3 x
1

In general, for y = sin kx and k 6= 0,


2
0 kx 2 0 x .
k
2
The period is .
k
The same applies to y = cos kx.

169
(b) y = tan kx

y = tan 12 x

1 y = tan x

x
2 0
2
1


In general y = tan kx Period= .
k

3. y = f(x `), ` > 0.



(a) y = cos(x ) - shift the graph of y = cos x to the right by .
3 3

y y = cos(x 3 )
1

3
2
2
3
2
6 0

3
5
6
4
3
11
6 2 3 x
1

y = f(x + `)- shift the graph of y = f(x) to the left by `.


 
(b) y = sin 2x + = sin 2(x + )-shift the graph of y = sin 2x to the left by .
3 6 6

4. y = f(x) + b, b > 0 shift the graph of y = f(x) upward by b units.


y = f(x) b shift the graph of y = f(x) downward by b units.

170
8.10 Trigonometric Equations

Solving trigonometric equations involves your knowledge of identities, ability to solve algebraic
equations, values of trigonometric functions for special angles and graphs of trigonometric
functions.

Let us start with elemenatary equations to get general solutions.

1. sin x = a, |a| 1 (see graphs 1)


sin x0 = a, where x0 is an initial solution.
The general solution is

x = x0 + 2k or x = x0 + 2k where k = 0, 1, 2, . . . .

1
Example: Solve the equation sin x =
2
1
Soln: Since sin = , it follows that the required general solution is
6 2
5
x= + 2k or x = + 2k where k = 0, 1, 2, . . . .
6 6

2. cos x = a, |a| 1 (see graph 2)


cos x0 = a.
The general solution is x = x0 + 2k, where k = 0, 1, 2, . . . .
1
Example: Solve the equation cos x = .
2
 
2 1
Soln: Since cos = , it follows that the general solution is
3 2
2
x= + 2k where k = 0, 1, 2, . . . .
3

2   1
(Note that cos = cos = cos = ).
3 3 3 2
3. tan x = a where a is any number.

Soln: The general solution of the equation tan x = a is

x = x0 + k where k = 0, 1, 2, . . . .

Example: Solve the equation tan x = 3.

171

Since tan = 3, it follows that the general solution of the equation tan x = 3 is
3

x= + k where k = 0, 1, 2, . . . .
3

Now we should consider special equations (see the graphs).


sin x = 1 x = + 2k

2







sin x = 1 x= + 2k

2







sin x = 0 x = k



cos x = 1 x = 2k where k = 0, 1, 2,






cos x = 1 x = + 2k









cos x = 0 x= + k

2





tan x = 0 x = k

We use general solutions to solve equations in given intervals.


Example:
1
sin x = where x [2, 2]
2  

x = + 2 k or x = + k k = 0, 1, 2,
4 4
3
If k = 0 then x = , .
4 4
If k = 1 out of [2, 2].
7 5
If k = 1 then x = ,
4 4
3 5 7
The solution is: , , , .
4 4 4 4

172
8.10.1 Solving Equations:

1. Solve the equation sin x = cos x.

Soln: Note that cos x 6= 0. (Why?)

Method 1  
sin x = sin x
2

x = x + 2k x = + k, where k = 0, 1, 2, . . .
2 4
h  i
or x = x + 2k x = + x + 2k (invalid)
2 2
Method 2

Divide both sides by cos x. We can do this because cos x 6= 0.


sin x
sin x = cos x = 1 tan x = 1
cos x

x = + k, where k = 0, 1, 2, . . .
4

2. Solve the equation sin 2x = sin x.

Soln: Method 1

2x = x + 2k = x = 2k
2
or 2x = ( x) + 2k = x = + k, where k = 0, 1, 2, . . .
3 3
Method 2
2 sin x cos x sin x = 0
1
sin x(2 cos x 1) = 0 sin x = 0 or cos x =
2
x = k or x = + 2k, k Z
3
We can check that those two solutions are equivalent.

3. Solve the equation sin x + 3 cos x = 0.

Soln:

sin x + 3 cos x = 0
sin x = 3 cos x
tan x = 3

we get x = + k, kZ
3
4. Solve the equation cos 3x + sin x = 0.

173
Soln:

cos 3x + sin x = 0 cos 3x = sin


 x 
cos 3x = cos +x
 2
3x = + x + 2k, where k = 0, 1, 2, . . .
2

Therefore

3x = + x + 2k 2x = + 2k


2 2
where k = 0, 1, 2, . . .


OR 3x = x + 2k 4x = + 2k
2 2


x= + k

4
where k = 0, 1, 2, . . .
k


OR x= +
8 2

5. Solve the equation 2 sin x + cot x = cosec x.

Soln: Note that sin x 6= 0. (Why?)


cos x 1
2 sin x + cot x = cosec x 2 sin x + =
sin x sin x
2
2 sin x + cos x = 1
2(1 cos2 x) + cos x = 1
2 cos2 x cos x 1 = 0
(2 cos x + 1)(cos x 1) = 0
2 cos x + 1 = 0 OR cos x 1 = 0
2 cos x = 1 OR cos x = 1

1
cos x = OR cos x = 1.
2
Now,
1 2
cos x = x = + 2k where k = 0, 1, 2, . . . .
2 3
As for the equation cos x = 1,

cos x = 1 x = 0 + 2k = 2k, where k = 0, 1, 2, . . . .

But for these values of x, sin x = 0, which cannot be! Therefore the solutions of the
equation 2 sin x + cot x = cosec x are
2
x= + 2k where k = 0, 1, 2, . . . .
3

174
6. Solve the equation 1 + tan x = cos 2x.

Soln: Note that cos x 6= 0. (Why?)

1 + tan x = cos 2x 1 + tan x = cos2 x sin2 x


1 + tan x = (1 sin2 x) sin2 x
1 + tan x = 1 2 sin2 x
tan x = 2 sin2 x
sin x
= 2 sin2 x
cos x
sin x + 2 sin2 x cos x = 0
sin x(1 + 2 sin x cos x) = 0
sin x(1 + sin 2x) = 0
sin x = 0 OR 1 + sin 2x = 0.

Now,
sin x = 0 x = k, where k = 0, 1, 2, . . .
and

1 + sin 2x = 0 sin 2x = 1

2x = + 2k, where k = 0, 1, 2, . . .
2

x = + k, where k = 0, 1, 2, . . .
4
Therefore the solutions of the equation 1 + tan x = cos 2x are

x = k

where k = 0, 1, 2, . . .


x = + k
4

175
7. Solve the equation sin 2x 2 sin2 x = 4(cos x sin x), x [2, 2].

Soln:

sin 2x 2 sin2 x = 4(cos x sin x) 2 sin x cos x 2 sin2 x 4(cos x sin x) = 0


2 sin x(cos x sin x) 4(cos x sin x) = 0
2(sin x 2)(cos x sin x) = 0
(sin x 2)(cos x sin x) = 0
sin x 2 = 0 OR cos x sin x = 0
sin x = 2 OR cos x = sin x.

Since sin x = 2 is invalid, we must have that cos x = sin x. Now,



cos x = sin x x = + k, where k = 0, 1, 2, . . .
4
The restriction that x [2, 2] implies that we must choose those values of k for
which x lies in the given interval. Therefore
5 3 7
x= , , , .
4 4 4 4

8. Solve the equation 3 sin x + cos x = 3.

Soln:

3 1 3
3 sin x + cos x = 3 sin x + cos x =
2 2 2
3
cos sin x + sin cos x =
6 6 2
  3
sin x + = .
6 2

Either x + = + 2k, k = 0, 1, 2, . . . x= + 2k, k = 0, 1, 2, . . .
6 3 6
2
OR x + = + 2k, k = 0, 1, 2, . . . x= + 2k, k = 0, 1, 2, . . .
6 3 2
Exercise 8.11.
7
1. Calculate the value of cos , leaving it in surd form.
12
 
5 11
cos sin + sin
4 4 6
2. Calculate, without using a calculator, the value of .
5
tan
4

176
3. Simplify:

sec() tan( + )cosec( )
  2 .
cot cosec( + ) cot( )
2
1 p
4. If sec = + and 0 < < , determine the value of tan + sec in terms of p.
2p 2 2
2
cosecA + 1 (1 + sin A)
5. (a) Prove that = .
cosecA 1 cos2 A

(b) What values may A not assume in the interval [0, ]?
2
tan tan 2
6. (a) Prove that + = .
sec 1 sec + 1 sin

(b) For what [0, ] is the identity not defined?
2
sin 2x
7. (a) Prove the identity: = tan x.
1 + cos 2x
(b) For what value(s) of x [0, 2] is the identity not defined?
cos 3A + sin 3A
8. (a) = 1 + 4 sin A cos A
cos A sin A
 
1 tan2 A
(b) 4  = sin 2A
2
1 + tan +A
4
A A
(c) cot + tan = 2cosecA
2 2
9. Solve the following equations:

(a) 4 cos sec = 0


(b) sin 2x + sin x = 6 cos x + 3 if x [, ]

(c) cos + cos + 1 = 0
2
h i
2 2
(d) (sin 2x sin x) + (2 sin 2x + 5 sin x 4) = 0 where x ,
2

(e) cos 3 sin 3 = 2 if [, ]
(f) cos 2 + 3 sin2 4 sin + 2 = 0

10. Given that f : y = tan 2x and g : y = sin x,



(a) sketch the graphs of f and g on the same set of axis for the domain [ , ].
2 2
(b) determine the value of x in the given domain for which g(x) f(x).

11. Draw the graphs of the following functions.

177
(a) y = | sin 2x| (d) y = tan |x|
1  
(b) y = | cos x| (e) y = 2 sin 2x
2 3
1  
(c) y = | tan x| (f) y = cos x + +1
2 4

178
Chapter 9

Exponential and Logarithmic


Functions

9.1 Exponential Laws and Definitions

9.1.1 Exponential Laws

1. apaq = ap+q (p, q R)


ap
2. = apq
aq
3. (ap)q = apq

4. (ab)p = apbp
 a p a p
5. = p
b b

9.1.2 Definitions

1. a0 = 1

00 is meaningless
ap
a0 = app = p = 1 (p N)
a
1
2. ap =
ap
a0 1
ap = a0p = p
= p
a a
1
3. a p = p
a

179
 1
p 1
a p = a p p = a1 = a
p
4. a q = q
ap
p p
a q = ( q a)

9.1.3 Surds Laws



1. a b = ab
p p p

r
a p
a
p
=
b p
b
p
2. p q a = pq a
q
3. ( p a) = p aq

Note: If a root of a number is irrational, it is called a surd.

9.2 Operations on Exponents and Surds

Examples 9.1.
  43
81
1. Simplify .
625
Soln:
  34 "  # 34    3
4 3
81 3 3 5 125
= = = =
625 5 5 3 27
3 1 2
2. Calculate the value of 4 2 + 8 3 32 5 + (0.5)2 .

Soln:
3 1 2 3 1 2
4 2 +8 3 32 5 +(0.5)2 = (22 ) 2 +(23 ) 3 (25 ) 5 +( 12 )2 = 23 +222 +22 = 8+2 = 10.

3. Prove that 5 > 3 11.

Soln:
1 3 6

5 = 5 2 = 5 6 = 53 = 6 125
1 2
6

3
11 = 11 3 = 11 6 = 112 = 6 121

5 = 6 125 > 6 121 = 3 11.

4. Write 3 32 as a power of 16.

180
Soln:
1 5
3
32 = (25 ) 3 = 2 3
5 5 5 5
2 3 = 16x 2 3 = 24x 4x = 3
x= 12
.
5
So, 3 32 = 16 12 .
92p+1 62p3
5. Simplify .
35p2 2 3p 4p2
Soln:
92p+1 62p3 (32 )2p+1 (3 2)2p3 34p+2 32p3 22p3
= =
35p2 2 3p 4p2 35p2 2 3p (22 )p2 35p2 2 3p 22p4
= 34p+2+2p35p+2p 22p312p+4 = 31 20 = 3
ax+3 + 2ax+2
6. Simplify .
ax + 2ax1
Soln:
ax+3 + 2ax+2 ax1 (a4 + 2a3 ) a4 + 2a3 a3(a + 2)
x x1
= x1
= = = a3 .
a + 2a a (a + 2) a+2 a+2
p 1
(a b)3 (a2 b2) 2
7. Simplify 1 .
(a2 + 2ab + b2) 4
Soln:
p 1 3 1 1 3 1 1
(a b)3 (a2 b2) 2 (a b) 2 (a b) 2 (a + b) 2 (a b) 2 2 (a + b) 2
1 = 1 = 1
(a2 + 2ab + b2 ) 4 [(a + b)2] 4 (a + b) 2
1 1 ab
= (a b)(a + b) 2 2 = (a b)(a + b)1 = .
a+b
Exercise 9.2.
  1
25 2
1. Simplify .
16

5
2 3 2 2
2. Calculate the value of 27 + 16 3
2 (no calculator).
4
23
+
4 5 8

3. Show by calculations (no calculator) which of 5 6 and 3 has the greater value.

4. Find the value of (0.04)0.5(no calculator).

5. Simplify
2x+1 4x+1
(a)
(2x )x1 (2x1 )x+1
32x 4 3x
(b) .
2 3x1 + 3x

181
2 1 2 1
6. Multiply (x 3 + x 3 + 1)(x 3 x 3 + 1) and check your result by finding the value of
your expression when x = 8.
1 1
7. If a 2 + a 2 = 1, find the value of a + a1.

9.3 Exponential Equations

An exponential equation is one with the unknown in the exponent.


If ax = ay , a > 0 and a 6= 1 then x = y because the exponential function is one-to-one.

Examples 9.3.

1.
27x 9x2 = 1
3x 2 x2
3 (3 ) = 3
so 3 5x4
= 3 5x 4 = 0 x = 54 .

2.
2x+2 + 2x2 + 2x = 42
x 2 x x
2 2 + 2 2 + 2 = 42
2
1
2x (22 + 22 + 1) = 42 2x (4 + 4
+ 1) = 42
21 4
We get 2x 4
= 42 2x = 42 21
2x = 8 2x = 23 . Thus x = 3.

3. 22x+2 5 2x = 1
22x 22 5 2x + 1 = 0 Let 2x = p.
We get 4p2 5p + 1 = 0 (4p 1)(p 1) = 0
p = 14 or p = 1
i.e 2x = 22 or 2x = 2
We get x = 2 or x = 0.

4. 3x 31x 2 = 0
3x 3 3x 2 = 0
1
3x 3 x 2 = 0 Let 3x = k
3
k 3 k1 2 = 0 k 2 2k 3 = 0, (k 3)(k + 1) = 0
We get k = 3 or k = 1
i.e. 3x = 31 or 3x = 1 but 3x 6= 1 since 3x > 0.
Thus x = 1.

9.4 Equations with Rational Exponents

These are equations with rational exponents where the unknown is in the base.

182
Examples 9.4.
1 1
 1 2
1. 2x 2 = 4 x 2 = 2 = x 2 = 22 x = 4.
1 1
2. x x 2 6 = 0. Let x 2 = k.
Then k 2 k 6 = 0 (k 3)(k + 2) = 0 k = 3 or k = 2
1 1 1
Thus, x 2 = 3 or x 2 = 2 (no solution since x 2 0)
1
Thus (x 2 )2 = 32
We get x = 9.
2
3. x 3 = 64
1 1 1
(x 3 )2 = 64 x 3 = 8 or x 3 = 8
1 1
(x 3 )3 = 83 or (x 3 )3 = (8)3
Thus x = 83 or x = (8)3 = 83 .
1 1
4. x1 7x 2 18 = 0. Let x 2 = k.
Then k 2 7k 18 = 0 (k 9)(k + 2) = 0 = k = 9 or k = 2.
1 1 1
Thus x 2 = 9 or x 2 = 2 (no solution since x 2 > 0)
1
Thus (x 2 )2 = 92
1
We get x = 81 .

9.5 Number Systems

The Hindu-Arabic system of numbers which we use is a base 10 system. It uses the digits 0 to 9.

2547, 326 means 2 103 + 5 102 + 4 101 + 7 100 + 3 101 + 2 10 2 + 6 103 .


This is an expression in descending powers of 10.

In a base 8 system we use the digits 0 to 7 only. Lets see what 765.42 (base 8) means in our
base 10 system.

765.42 (base 8) = 7 82 + 6 81 + 5 8 + 4 81 + 2 82
= 7 64 + 48 + 5 + 12 + 64 2

= 448 + 53 + 0.5 + 0.03125


= 501.53125 base 10)

The machine language of a computer uses a binary (base 2) system using the digits 0 and 1
only.
1.101 (base 2) = 1 2 + 1 21 + 0 22 + 1 23
= 1 + 12 + 0 + 81
= 1.625 (base 10)
Exercise 9.5.

183
1. Convert each of these numbers to a base 10 number:

(a) 234.21 (base 5)


(b) 110101 (base 2)
(c) 101.11 (base 2)
(d) 122.2 (base 4)

2. Solve the following exponential equations:

(a) 52x+3 = 0.008



(b) 3x+1 3x1 = 24 3
(c) 45x1 = 5x1 3x
(d) 9x+1 10 3x + 1 = 0
1
(e) x + 5 2x + 21x = 8
2
(f) 2x+1 + 23 = 4 2x + 20

3. Solve the following equations:


3 4 4
(a) x3 =0
4 27
1
(b) x1 + x 2 = 34
2 1
(c) 4x 3 + 5x 3 6 = 0

9.6 Scientific Notation

Scientists frequently work with very large or very small numbers e.g. a light year is approxi-
mately 9460000000000km. So a light year 9.46 1012 km. One light year is the distance
that light travels in one year. Now the speed of light is approximately 300000km/s. How far
does it go?

One year is 365.25 days, a day has 24 hours, an hour has 3600 seconds.
So

1 light year (365.25 24 3600)s 300000km/s


9.46 1012 km

9.46 is a number in standard form, i.e. it has one non-zero digit in front of the decimal comma.
To write a number in Scientific Notation, we write it as a number in standard form a power
of 10:
N 10k where 1 N < 10

184
Examples 9.6.

1. Write each of these numbers in Scientific Notation:

(a) 123000000 = 1.23 108


(b) 365000 = 3.65 105
(c) 0.457 = 4.57 101
(d) 0.00000036 = 3.6 107

2. Write each of these numbers in full:

(a) 3.456 104 = 0.0003456


(b) 1.234 107 = 0.0000001234
(c) 2.36 106 = 2360000

3. Calculate, using Scientific Notation:

(a) (1.2 104 ) (9.1 108 ) = 10.92 104 = 1.092 101 104 = 1.092 103
(b) 3.02 103 + 9.1 104 = 3.02 103 + 9.1 101 103 = 3.02 103 +
0.91 103 = 3.93 103
(c) (2.34 1012 ) (5 107 ) = 11.7 105 = 1.17 101 105 = 1.17 104
(d)

4.32 106 + 2.42 104 = 4.32 102 104 + 2.42 104


= 0.0432 104 + 2.42 104
= 2.4632 104

4
4. The volume of a sphere with radius r is given by V = r3. Let us find the volume of
3
the earth. Circumference of the earth = 40 000km = 2r
so
2r = 40 000 000m = 4 107 m
4 107 2 107
r= m= m
2
 3
4 2 107 4 8 32 1 21 3
VE = m = 3 1021 m3 = 10 m 1.081021 m3.
3 3 3 2
Exercise 9.7. Simplify, using Scientific Notation:

1. (4.62 1011 ) (3.2 107 )

2. (8.7 1016 (2.4 1010 )

3. 1.6 108 + 1.2 106

185
4. 6.3 104 + 3.45 202
(2.4 105 ) (7.24 108 )
5.
8.3 104
5.5 103 + 4.2 102
6.
2.5 102
7.8 103 8.6 104
7.
4.6 104 3.2 103
8. Write in Scientific Notation:

(a) 928700000 (b)0.00000783 (c)0.000000000282

9.7 Exponential Functions

Definition 9.8.
An exponential function is a function defined by y = ax where a > 0, a 6= 1, x R.

Note: The name exponential is used since y changes as x, the exponent, changes.

Lets consider graphing some exponential functions.

y

1 x

1 x y= y = 3x
y= 2
3
9 y = 2x
8

4
3
2

x
3 2 1 0 1 2 3

 x  x
1 1
The pair of graphs of y = 2x and y = ; y = 3x and y = is symmetrical with
2 3
respect to the y-axis.

186
In general we can say that for a function y = ax , a > 1

1. The y-intercept is always the point (0, 1).

2. As x increases, y increases (Increasing Functions).

3. The x-axis (y = 0) is a horizontal asymptote.

4. The domain is R and the range is (0, ).

5. The greater the value of a, the steeper the curve of y = ax, a > 1 becomes.

For a functions of a form y = ax, 0 < a < 1.

1. The y-intercept is always the point (0, 1).

2. As x increases, y decreases (Decreasing Function).

3. The x-axis is a horizontal asymptote.

4. The domain is R and the range is (0, ).

5. The smaller the value of a i.e. as a becomes a smaller positive fraction, the steeper the
curve of y = ax , 0 < a < 1 becomes.

 x
x 1
We can notice that the curves y = a and y = a = x
are reflections of each other in
a
the y-axis, and y = ax and y = ax are reflections of each other in the x-axis. This illustrated
in the diagram below.
y


1 x
y= 2
= 2x y = 2x

0 x


1 x
y= 2
= 2x y = 2x

187
Examples:

1. f(x) = 4x Determine the equation of

(a) g, the reflection of f about the y-axis.


(b) p, the reflection of f about the x-axis.
(c) s, the reflection of g about the x-axis.

Solution:

 x
1
(a) g(x) = 4 x
or g(x) =
4
(b) p(x) = 4x
 x
x 1
(c) s(x) = 4 or s(x) =
4
2. y = ax and point B(4, 16 ) lies on the graph of y = ax . Find a.

Solution
1
16
= a4
4 1
a4 = (21 ) = a = 2

3. Solve the following inequalities:



1 x
(a) 1 2x 2 (b)3 < 3
<9

Solution

(a) 2 2x 21 = 0 x 1 since y = 2x is an increasing function.


 x

1 1

1 x

1 2 1
(b) 3 < 3 < 3 = x > x > 2 since y = is a decreasing
3
function.

Exercise:

1. Graph each of the exponential functions:


 x
3
(a) y =
2
 x
2
(b) y =
3
(c) y = 2x + 1
(d) y = 2x1

188
(e) y = 2x+1
(f) y = 2x+2 1
(g) y = |3x 2|

2. Solve the following inequalities:


 x  x
x 1 8 2 4
(a) 3 81 (b) > 64 (c) <
2 125 5 25

9.8 Logarithms (Definitions)

Let ab = c. Assume that a and c are given. We would like to find b.

b = logac

So, ab = c b = loga c a > 0, a 6= 1, c > 0, b R

Note: log10 x was commonly used to do calculations before the advent of the electronic cal-
culator. It thus became practice to write 00logx00 for 00 log10x00.

Examples (Short problems on Logarithms)

1. Express the following in log-form.


1 2 1
(a) 27 3 = 3 (b) 32 5 = 4 (c) 3 2 = x

Solution:
1
1
27 3 = 3 3
= log273
2
2
32 5 = 4 5
= log324
1
1
32 = x 2
= log3x
2. Express the following in exponential form.
1
(a) log16 4 = 2
(b) log5x = 2 (c) log28x = 3x

Solution:
1
log164 = 12 16 2 = 4
log5x = 2 52 = x
log28x = 3x 23x = 8x
3. Simplify:

(a) log2 32 (b) log 1 4 (c) log0.001
4

189
Solution:
Let 1
log2 32 = x 2x = 32 2
5
2x = 2 2 x = 52
x
log 1 4 = x 14 = 4 x = 1
4
log0.001 = x 10x = 103 x = 3
4. Solve the equation:

(a) log 1 x = 1 ( 14 )1 = x; x = 4
4

(b)
logx 18 3 x3 = 18
x3 = 23 x = 2
(c)
1
2
logx
=1
2
so logx = 2 10 = x

9.9 Logarithmic Laws

In the following laws: a > 0, a 6= 1, x, y > 0.

1. loga xy = logax + loga y

2. loga xy logax loga y

3. loga xn = nlogax nR

Proof:

1. Let loga x = p and loga y = q


loga x = p ap = x and loga y = q aq = y.
xy = ap aq
xy = ap+q (Exp. Law) but xy = ap+q p + q = logaxy so loga xy = loga x + loga y

2. From (1) ap = x and aq = y


x ap
= q
y a

Examples (Applications of Logarithmic Laws)

1. Express as the log of a single number

(a) 2log5 + 3log2 log2

190
(b) log18 2log3 log2
(c) 5logp 3logq 2logr

Solution:
52 23
(a) 2log5 + 3log2 log2 = log52 + log23 log2 = log = log100 = 2
2
18
(b) log18 2log3 = log2 = log = log1 = 0
322
p5
(c) 5logp 3logq 2log4 = logp5 logq 3 logr2 = log
q 3 r2
2. Expand the following:
pq 2 7x3 y 2
(a) log7a2 bc (b) log2 (c) log3
r3 pq 4
Solution:

(a) log7a2 bc = log7 + loga2 + logb + logc = log7 + 2loga + logb + logc
pq 2
(b) log2 3 = log2 p + log2 q 2 log2 r3 = log2p + 2log2 q 3log2 r
r
7x3 y 2
(c) log3 = log37 + 3log3 + 2log3 y log3p 4log3 q
pq 4
3. If log2 = a and log3 = 6, express the following in terms of a and b.

(a) log60 (b) log24 (c) log2.4 (d) log 52 (e) log7 12

Solution:

(a) log60 = log2 3 10 = log2 + log3 + log10 = a + b + 1


(b) log24 = log23 3 = 3log2 + log3 = 3a + b
24
(c) log2.4 = log 10 = log24 log10 = 3a + b 1
(d) log 25 = log 10
4
= log22 log10 = 2log2 log10 2log2
(e) log7 12 = log 15
2
= log 30
4
= log30 log4 = log3 + log10 2log2 = b + 1 2a

More difficult simplification

(a) xlogx a (2) 45log4 2 (3) y logy y

Solution:

(a) Let xlogx a = k logx a = logx k = k = a


Thereforexlogx a = a(log functions is one-to-one.
(b) Let 445log4 2 = ` 5log4 2 = log4 `
log4 32 = log4 ` ` = 32
Therefore 45log4 2 = 32

191
(c) Let y logy y = p logy y = logy p p = y
Therefore y logy y = y

9.10 Change of base

logb x
loga x = b > 0, b 6= 1
lb x

Proof:
Let loga x = p aP = x
logb x
lb aP = logb x so plogb a = logb x p =
logb a
Examples

1. Simplify (Change Base)


log25 2log5
(a) log 1 25 = 1 = = 2
5 log 5 log5
(b)
log2 log49 log7 27
log2 log49 log27
=
log3 log4 log7
log2 2log7 3log3
=
log3 2log2 log7
= 3
(c)
log2 5 log4 3 log2516
log5 log3 4log2
=
log2 2log2 2log5
log3
=
log2
= log2 3
1
2. Prove that loga b = .
logb a
logab = k ak = b
Let
logb ak = logb b klogb a = 1
1
so k = .
logb a
3. Prove that: log4 x = 21 log2x.
1
Let log4 x = p 4p = x; 22p = x 2p = log2x so 2log4 x = log2 x log4 x = 22
x

192
Exercise

1. Write each of the following in exponential form:


1
(a) 22 = 4
(b) 102 = a (c) 34 = y (d) 56 = z

2. Write each of the following in exponential form:


1
(a) log3 81 = 4 (b) log2 ( 16 ) = 4 (c) log3 x = 4 (d) logx p = q

3. Evaluate each of the following:


1
(a) log6 3 6 (b) log2 2 5 (c) log6 216 (d) 10log10 5

(e) aloga 5 (f) log0.00001 (g)log100000 (h) log2 512

9.11 Logarithmic Equations

Examples

1.
log2(x 2) + log2 (x 3) = 1
log2 (x 2)(x 3) = 1
x2>0
(x 2)(x 3) = 2
and x>3
x2 5x + 4 = 0 (x 4)(x 1) = 0
x3>0
x = 4 or x = 1 and x > 3
x = 4 is the only solution.
2.
2log9 x + 2logx 9 = 5 x > 0, x 6= 1
1
2log9 x + 2 = 5 Let log9x = k
log9 x
2k + k2 = 5
2k 2 5k + 2 = 0
(2k 1)(k 2) = 0
k = 12 or k =2
1
So log9 x = 2 or log9 x = 2
1
x = 9 2 or x = 92
x = 3 x = 81

193
3.
xlogx = 1000x2 x>0
loglogx
x = log1000x2
logxlogx = 1000 + 2logx Letlogx = k
k2 = 3 + 2k
k 2 2k 3 = 0 = 0
(k 3)(k + 1) = 0
k = 3 or k = 1
So logx = 3 or logx = 1
1
x = 1000 or x = 10
4.
[log2 (x + 3)]3 = log2 (x + 3)4 x > 3
[log2 (x +3)]3 4log 2(x + 3)
= 0
2
log2 (x + 3) [log2 (x + 3)] 4 = 0
log2(x + 3) {[log2 (x + 3) 2] [log2 (x + 3) + 2]} = 0
log2(x + 3) = 0 or log2 (x + 3) = 2 or log2 (x + 3) = 2
x + 3 = 20 x + 3 = 22 x + 3 = 22
x = 2 x = 1 x = 11/4
5.

log5 x = logx 5 x > 0, x 6= 1
1
log5 x = Let l5 x = p
log5 x
1
p =
p
p2 1 = 0
(p 1)(p + 1) = 0

p = 1 or p = 1
log5x = 1 log5 x = 1
x = 5 x = 15
1
x = 25 x =
25
6.
73logx = 7x2
3logx = log77x2 x>0
logx3 = log77 + log7 x2
logx2
logx3 = 1 +
log7
3logxlog7 = log7 + logx2
3logxlog7 2logx = log7
logx(3log7 2) = log7
log7
So logx =
3log7 2

194
7.
log4 x = (log2x)2 (Hint : loga b = logan bn ) x>0
log4 x = (log22 x2)2
log4 x = (2log22 x)2
log4x (2log22 x)2 = 0
log4 x(4log4 x 1) = 0
log4x = 0 or log4 x = 14
x = 1 x = 41/4 = 2

Exercise
Solve the following equations:

1. logx 3 log3 x = 2

2. logx 9 = log2x3

3. (logx)2 5 = log(10x)

4. log(x2 4) log(x 2) = 2logx

5. log3 (3 x) = log 1 (5 2x)


3

p
6. log2 x log2x = 4

7. 10xlogx = x2

8. 5x+1 + 5x2 = 252

9. 32x 5 3x + 6 = 0

10. 5x 7 + 2 51x

11. loga [log2(log7 x)] = 0


1
12. log4 [log3(log2 x)] = 2

13. log2 x + log8x = 8

14. logx2 (x3 14) = 3

195
9.12 Logarithmic Functions

9.12.1 Definition of Logarithmic Function

y = loga x ay = x a > 0, a 6= 1, x > 0, y R.

We can draw the graph of the logarithmic function by using the graph of the exponential
function.

Now y = loga x will be the mirror image of x = loga y reflected about y = x.


Also, x = loga y y = ax .
We consider two cases: (i) a > 1 and (ii) 0 < a < 1.

y = ax
x
=

a>1
y

1
x
0 1
x = ay or y = loga x

y = ax
x
=

0<a<1
y

1
x
0 1

x = ay or y = loga x

196
We can also draw the graph of y = loga x using a table.

In general we can say that for a function y = loga x

1. D = (0, )

2. R = R

3. x-intercept is (1; 0)

4. y-axis is a vertical asymptote

5. It is a one-to-one function

6. Increasing function if a > 1

Decreasing function is 0 < a < 1

Examples

1. Determine the equation of a graph of the function y = loga x if it passes through the
point ( 14 ; 2).

Soln: y = loga x
2 = loga 14 a2 = 1
4
a2 = 22 a = 2.
y = log2 x

2. Determine the equation of the graph formed if y = log3x is reflected about

(a) the x-axis (b) the y-axis (c) the line y = x.

Soln: (a) y = log3 x or y = log3 x1 or y = log 1 x


3

(b) y = log3 (x)


(c) y = 3x

3. Determine the domain of:

(a) y = log2 (7x 5)


(b) y = log3 [9 (x 4)2 ]
(c) y = log( x2 + x 6)
5
Soln: (a) 7x 5 > 0 x > 7
= D = ( 57 ; ).
(b) 9(x4)2 > 0 (x4)2 9 < 0 (x43)(x4+3) < 0 (x7)(x1) < 0
So we need 1 < x < 7 and thus D = (1; 7).
(c) x2 + x 6 > 0 (x + 3)(x 2) > 0 x < 3 or x > 2.
Thus D = (; 3) (2, ).

197

You might also like